|
<head data-webtasks-id="689717d3-f197-47e5"> |
|
|
|
<title data-webtasks-id="4a659471-b689-4122">where on earth - Are these murals of a real location? Perhaps Bavaria? - Travel Stack Exchange</title> |
|
<link rel="shortcut icon" href="https://cdn.sstatic.net/Sites/travel/Img/favicon.ico?v=d46bb43d5893" data-webtasks-id="8963abf8-2ca0-4a60"> |
|
<link rel="apple-touch-icon" href="https://cdn.sstatic.net/Sites/travel/Img/apple-touch-icon.png?v=0bf351ad5aa0" data-webtasks-id="8e2e037d-5eac-47b2"> |
|
<link rel="image_src" href="https://cdn.sstatic.net/Sites/travel/Img/apple-touch-icon.png?v=0bf351ad5aa0" data-webtasks-id="fde34b4b-9b31-4e69"> |
|
<link rel="search" type="application/opensearchdescription+xml" title="Travel Stack Exchange" href="/opensearch.xml" data-webtasks-id="9fbfdc46-3a77-4c66"> |
|
<link rel="canonical" href="https://travel.stackexchange.com/questions/181865/are-these-murals-of-a-real-location-perhaps-bavaria" data-webtasks-id="278dada7-1bd8-40a7"> |
|
<meta name="viewport" content="width=device-width, height=device-height, initial-scale=1.0, minimum-scale=1.0" data-webtasks-id="9f2ad3fb-32f7-4cbc"> |
|
<meta property="og:type" content="website" data-webtasks-id="da256c31-5da7-43c6"> |
|
<meta property="og:url" content="https://travel.stackexchange.com/questions/181865/are-these-murals-of-a-real-location-perhaps-bavaria" data-webtasks-id="d46c7822-82a6-4d2b"> |
|
<meta property="og:site_name" content="Travel Stack Exchange" data-webtasks-id="110b66ea-e3e1-4581"> |
|
<meta property="og:image" itemprop="image primaryImageOfPage" content="https://cdn.sstatic.net/Sites/travel/Img/[email protected]?v=c2b2aadd2cbd" data-webtasks-id="4bc9c11c-4ff1-40a9"> |
|
<meta name="twitter:card" content="summary" data-webtasks-id="59f99f7c-9ee6-4fd4"> |
|
<meta name="twitter:domain" content="travel.stackexchange.com" data-webtasks-id="125dd507-08c1-4241"> |
|
<meta name="twitter:title" property="og:title" itemprop="name" content="Are these murals of a real location? Perhaps Bavaria?" data-webtasks-id="97ae9437-a0f1-4b10"> |
|
<meta name="twitter:description" property="og:description" itemprop="description" content="In the town of Blackpool, England my partner and I have bought a house which was built around 1952. When we took the old wallpaper off the walls, we discovered amazing things - the lounge, dining r..." data-webtasks-id="1e6bd79f-3075-4d74"> |
|
<script id="webpack-public-path" type="text/uri-list" data-webtasks-id="af2b8b06-b589-4eeb">https:</script> |
|
<script src="https://ajax.googleapis.com/ajax/libs/jquery/1.12.4/jquery.min.js" data-webtasks-id="893c1969-d09b-45d1"></script> |
|
<script defer="" src="https://cdn.sstatic.net/Js/third-party/npm/@stackoverflow/stacks/dist/js/stacks.min.js?v=ad920dba3340" data-webtasks-id="cd0d6781-fe2a-4f26"></script> |
|
<script src="https://cdn.sstatic.net/Js/stub.en.js?v=34b15b21ff80" data-webtasks-id="b93f82bc-a5d8-47f3"></script> |
|
|
|
<link rel="stylesheet" type="text/css" href="https://cdn.sstatic.net/Shared/stacks.css?v=73d389de9f03" data-webtasks-id="a3266029-01b6-4d4e"> |
|
<link rel="stylesheet" type="text/css" href="https://cdn.sstatic.net/Sites/travel/primary.css?v=c1dbec10ac94" data-webtasks-id="3a3508b9-bfaf-45a3"> |
|
|
|
|
|
|
|
<link rel="alternate" type="application/atom+xml" title="Feed for question 'Are these murals of a real location? Perhaps Bavaria?'" href="/feeds/question/181865" data-webtasks-id="1da03a2e-89d0-4311"> |
|
<script data-webtasks-id="bd972829-bd6e-4468"> |
|
StackExchange.ready(function () { |
|
|
|
StackExchange.using("postValidation", function () { |
|
StackExchange.postValidation.initOnBlurAndSubmit($('#post-form'), 2, 'answer'); |
|
}); |
|
|
|
|
|
StackExchange.question.init({votesCast:[],autoShowCommentHelp:true,showAnswerHelp:true,totalCommentCount:0,shownCommentCount:0,enableTables:true,questionId:181865}); |
|
|
|
styleCode(); |
|
|
|
StackExchange.realtime.subscribeToQuestion('273', '181865'); |
|
StackExchange.using("gps", function () { StackExchange.gps.trackOutboundClicks('#content', '.js-post-body'); }); |
|
|
|
|
|
}); |
|
</script> |
|
|
|
|
|
|
|
|
|
<link rel="stylesheet" type="text/css" href="https://cdn.sstatic.net/Shared/Channels/channels.css?v=a4d77abedec3" data-webtasks-id="0e8598ac-60a8-4a8d"> |
|
|
|
|
|
|
|
|
|
|
|
<script data-webtasks-id="81dab132-e4e1-4cbe"> |
|
StackExchange.ready(function () { |
|
StackExchange.realtime.init('wss://qa.sockets.stackexchange.com'); |
|
StackExchange.realtime.subscribeToReputationNotifications('273'); |
|
StackExchange.realtime.subscribeToTopBarNotifications('273'); |
|
}); |
|
</script> |
|
<script type="application/json" data-role="module-args" data-module-name="Shared/options.mod" data-webtasks-id="55a2a144-a4ee-4f08">{"options":{"locale":"en","serverTime":1688971389,"routeName":"Questions/Show","stackAuthUrl":"https://stackauth.com","networkMetaHostname":"meta.stackexchange.com","site":{"name":"Travel Stack Exchange","description":"Q\u0026A for road warriors and seasoned travelers","isNoticesTabEnabled":true,"enableNewTagCreationWarning":false,"insertSpaceAfterNameTabCompletion":false,"id":273,"cookieDomain":".stackexchange.com","childUrl":"https://travel.meta.stackexchange.com","negativeVoteScoreFloor":null,"enableSocialMediaInSharePopup":true,"protocol":"https"},"user":{"fkey":"fe76f3281bad2063ad30f357d65244f2508ca27d494a61fe42e4d6aea23fa56d","tid":"3384c16d-10ea-b1da-8b32-aa877dc15704","rep":1,"isRegistered":true,"userType":3,"userId":136878,"accountId":28364646,"gravatar":"\u003cdiv class=\u0022gravatar-wrapper-32\u0022\u003e\u003cimg src=\u0022https://www.gravatar.com/avatar/64db2cea44c88e309a742529e139f6bd?s=32\u0026amp;d=identicon\u0026amp;r=PG\u0026amp;f=y\u0026amp;so-version=2\u0022 alt=\u0022Webtasks Navigator\u0026#39;s user avatar\u0022 width=\u002232\u0022 height=\u002232\u0022 class=\u0022bar-sm\u0022\u003e\u003c/div\u003e","profileUrl":"https://travel.stackexchange.com/users/136878/webtasks-navigator","canSeeDeletedPosts":false},"realtime":{"newest":true,"active":true,"tagged":true,"staleDisconnectIntervalInHours":0},"events":{"postType":{"question":1},"postEditionSection":{"title":1,"body":2,"tags":3}}}}</script> |
|
<script type="application/json" data-role="module-args" data-module-name="Shared/settings.mod" data-webtasks-id="87d89215-0c7c-47a2">{"settings":{"userMessaging":{},"tags":{},"subscriptions":{"defaultBasicMaxTrueUpSeats":250,"defaultFreemiumMaxTrueUpSeats":50,"defaultMaxTrueUpSeats":1000},"snippets":{"renderDomain":"stacksnippets.net"},"site":{"allowImageUploads":true,"enableImgurHttps":true,"enableUserHovercards":true,"forceHttpsImages":true,"stacksEditorPreviewEnabled":true},"search":{},"questions":{"enableQuestionTitleLengthLiveWarning":true,"enableSavesFeature":true,"maxTitleSize":150,"questionTitleLengthStartLiveWarningChars":50},"intercom":{"appId":"inf0secd","enableJavascriptImplementationFor":true,"hostBaseUrl":"https://stacksnippets.net"},"paths":{"jQueryUICSSPath":"https://ajax.googleapis.com/ajax/libs/jqueryui/1.12.0/themes/smoothness/jquery-ui.css","jQueryUIJSPath":"https://ajax.googleapis.com/ajax/libs/jqueryui/1.12.0/jquery-ui.min.js"},"mentions":{"maxNumUsersInDropdown":50},"markdown":{"enableTables":true},"legal":{"oneTrustConfigId":"cb0f3c87-b769-4e66-bbaa-377f9194216d"},"flags":{"allowRetractingCommentFlags":true,"allowRetractingFlags":true},"elections":{"opaVoteResultsBaseUrl":"https://www.opavote.com/results/"},"comments":{},"accounts":{"currentPasswordRequiredForChangingStackIdPassword":true}}}</script> |
|
<script data-webtasks-id="4ecc8ef4-2c03-4b8e">StackExchange.init();</script> |
|
|
|
<script data-webtasks-id="41eaf09c-d06c-468f"> |
|
StackExchange.using.setCacheBreakers({"Js/adops.en.js":"6da43f5e0a84","Js/ask.en.js":"","Js/begin-edit-event.en.js":"20edbaccceae","Js/copy-transpiled.en.js":"d31dc7eba3bc","Js/events.en.js":"","Js/explore-qlist.en.js":"2b1f34938b8b","Js/full-anon.en.js":"4cfd379baed1","Js/full.en.js":"9ce8d496d04c","Js/highlightjs-loader.en.js":"510e2f94c2bf","Js/inline-tag-editing.en.js":"db4721cf46fa","Js/keyboard-shortcuts.en.js":"107c2ac31497","Js/markdown-it-loader.en.js":"5818ef89ff9d","Js/mentions-transpiled.en.js":"aaeac2684aa8","Js/moderator.en.js":"436d64ed1be0","Js/postCollections-transpiled.en.js":"216ffeda5016","Js/post-validation.en.js":"cce75daddbbd","Js/question-editor.en.js":"","Js/review-v2-transpiled.en.js":"69c0e3530e0b","Js/revisions.en.js":"e6acc03fd5b5","Js/stacks-editor.en.js":"cfd7c1c2abea","Js/tageditor.en.js":"dc13482a67f8","Js/tageditornew.en.js":"cebacb19d403","Js/tagsuggestions.en.js":"bd6ec908f2a7","Js/unlimited-transpiled.en.js":"e776ac1beccb","Js/wmd.en.js":"8c0f9d3b0a66"}); |
|
StackExchange.using("gps", function() { |
|
StackExchange.gps.init(false); |
|
}); |
|
</script> |
|
<noscript id="noscript-css" data-webtasks-id="12233406-b82d-4b90"><style>body,.s-topbar{margin-top:1.9em}</style></noscript> |
|
<script async="" src="https://cdn.sstatic.net/Js/full.en.js?v=9ce8d496d04c" data-webtasks-id="92912133-6b99-440f"></script><meta http-equiv="origin-trial" content="A7CQXglZzTrThjGTBEn1rWTxHOEtkWivwzgea+NjyardrwlieSjVuyG44PkYgIPGs8Q9svD8sF3Yedn0BBBjXAkAAACFeyJvcmlnaW4iOiJodHRwczovL2RvdWJsZWNsaWNrLm5ldDo0NDMiLCJmZWF0dXJlIjoiUHJpdmFjeVNhbmRib3hBZHNBUElzIiwiZXhwaXJ5IjoxNjk1MTY3OTk5LCJpc1N1YmRvbWFpbiI6dHJ1ZSwiaXNUaGlyZFBhcnR5Ijp0cnVlfQ==" data-webtasks-id="4dcab712-e530-4db0"><meta http-equiv="origin-trial" content="A3vKT9yxRPjmXN3DpIiz58f5JykcWHjUo/W7hvmtjgh9jPpQgem9VbADiNovG8NkO6mRmk70Kex8/KUqAYWVWAEAAACLeyJvcmlnaW4iOiJodHRwczovL2dvb2dsZXN5bmRpY2F0aW9uLmNvbTo0NDMiLCJmZWF0dXJlIjoiUHJpdmFjeVNhbmRib3hBZHNBUElzIiwiZXhwaXJ5IjoxNjk1MTY3OTk5LCJpc1N1YmRvbWFpbiI6dHJ1ZSwiaXNUaGlyZFBhcnR5Ijp0cnVlfQ==" data-webtasks-id="ad1b407c-3184-4070"><meta http-equiv="origin-trial" content="A4A26Ymj79UVY7C7JGUS4BG1s7MdcDokAQf/RP0paks+RoTYbXHxceT/5L4iKcsleFCngi75YfNRGW2+SpVv1ggAAACLeyJvcmlnaW4iOiJodHRwczovL2dvb2dsZXRhZ3NlcnZpY2VzLmNvbTo0NDMiLCJmZWF0dXJlIjoiUHJpdmFjeVNhbmRib3hBZHNBUElzIiwiZXhwaXJ5IjoxNjk1MTY3OTk5LCJpc1N1YmRvbWFpbiI6dHJ1ZSwiaXNUaGlyZFBhcnR5Ijp0cnVlfQ==" data-webtasks-id="73bed019-9de1-48a6"><meta http-equiv="origin-trial" content="As0hBNJ8h++fNYlkq8cTye2qDLyom8NddByiVytXGGD0YVE+2CEuTCpqXMDxdhOMILKoaiaYifwEvCRlJ/9GcQ8AAAB8eyJvcmlnaW4iOiJodHRwczovL2RvdWJsZWNsaWNrLm5ldDo0NDMiLCJmZWF0dXJlIjoiV2ViVmlld1hSZXF1ZXN0ZWRXaXRoRGVwcmVjYXRpb24iLCJleHBpcnkiOjE3MTk1MzI3OTksImlzU3ViZG9tYWluIjp0cnVlfQ==" data-webtasks-id="1cb76b6b-7fae-4a14"><meta http-equiv="origin-trial" content="AgRYsXo24ypxC89CJanC+JgEmraCCBebKl8ZmG7Tj5oJNx0cmH0NtNRZs3NB5ubhpbX/bIt7l2zJOSyO64NGmwMAAACCeyJvcmlnaW4iOiJodHRwczovL2dvb2dsZXN5bmRpY2F0aW9uLmNvbTo0NDMiLCJmZWF0dXJlIjoiV2ViVmlld1hSZXF1ZXN0ZWRXaXRoRGVwcmVjYXRpb24iLCJleHBpcnkiOjE3MTk1MzI3OTksImlzU3ViZG9tYWluIjp0cnVlfQ==" data-webtasks-id="f5398601-2e35-4e7d"><script src="https://pagead2.googlesyndication.com/pagead/managed/js/gpt/m202306290101/pubads_impl.js" async="" data-webtasks-id="25e01510-f99c-438b"></script><script async="" src="https://cdn.sstatic.net/Js/post-validation.en.js?v=cce75daddbbd" data-webtasks-id="4bb730b2-2cbe-4c0d"></script><script async="" src="https://cdn.sstatic.net/Js/wmd.en.js?v=8c0f9d3b0a66" data-webtasks-id="8067fdf0-1b23-45e4"></script></head> |
|
<body class="question-page unified-theme" data-webtasks-id="7855e6f5-d484-4f49"> |
|
<div id="notify-container" data-webtasks-id="bfbe13d5-2fa0-46a4"></div> |
|
<div id="custom-header" data-webtasks-id="5fd8123d-de38-4920"></div> |
|
|
|
<header class="s-topbar ps-fixed t0 l0 js-top-bar" data-webtasks-id="d7d3fc44-4607-44dd"> |
|
<div class="s-topbar--container" data-webtasks-id="996a25c1-aba9-48f1"> |
|
<a href="#" class="s-topbar--menu-btn js-left-sidebar-toggle" role="menuitem" aria-haspopup="true" aria-controls="left-sidebar" aria-expanded="false" data-webtasks-id="10504837-e265-4e82"><span data-webtasks-id="fb945df6-2def-4eb9"></span></a> |
|
<div class="topbar-dialog leftnav-dialog js-leftnav-dialog dno" data-webtasks-id="380c9d64-6767-487b"> |
|
<div class="left-sidebar js-unpinned-left-sidebar" data-can-be="left-sidebar" data-is-here-when="sm" data-webtasks-id="2d6730e5-4d7d-4d09"></div> |
|
</div> |
|
<a href="#" class="s-topbar--logo network-logo js-gps-track js-network-logo" data-gps-track="stack_exchange_popup.show" role="menuitem" aria-haspopup="true" aria-controls="topbar-network-logo-dialog" aria-expanded="false" data-webtasks-id="d367e7e6-ef12-4c2b"> |
|
<svg aria-hidden="true" class="native mtn1 svg-icon iconLogoSEAlternativeSm" width="107" height="15" viewBox="0 0 107 15" data-webtasks-id="d2083ea8-84d3-4823"><path d="m48.41 11.93-1.96-3.2-1.04 1.16v2.04h-1.42V2.18h1.42v6.01L48.14 5h1.72l-2.44 2.7 2.74 4.22h-1.75Zm-7.06.08c-1.59 0-3.14-.96-3.14-3.56s1.55-3.54 3.14-3.54c.97 0 1.65.27 2.31.97l-.97.93c-.44-.48-.79-.66-1.34-.66s-1 .22-1.3.62c-.31.38-.42.87-.42 1.68 0 .81.1 1.32.41 1.7.3.4.76.62 1.3.62.56 0 .9-.18 1.35-.66l.97.92c-.66.7-1.34.98-2.31.98Zm-5.66-3.15h-1.65c-.83 0-1.26.37-1.26 1s.4.99 1.3.99c.53 0 .93-.04 1.3-.4.22-.2.31-.53.31-1.03v-.56Zm.03 3.07v-.63c-.51.5-1 .71-1.87.71-.87 0-1.46-.2-1.89-.63a2.1 2.1 0 0 1-.55-1.49c0-1.16.82-2 2.42-2h1.86v-.5c0-.87-.44-1.3-1.54-1.3-.77 0-1.15.18-1.54.68l-.92-.86c.66-.77 1.35-1 2.52-1 1.93 0 2.9.8 2.9 2.38v4.64h-1.39Zm-5.9 0c-1.32 0-1.93-.93-1.93-1.93V6.18h-.8V5.1h.8V3h1.41v2.1h1.36v1.07H29.3v3.75c0 .5.25.81.78.81h.58v1.2h-.85Zm-6.33.08c-1.48 0-2.55-.34-3.49-1.28l1-.98c.72.72 1.51.94 2.52.94 1.3 0 2.04-.55 2.04-1.5 0-.42-.13-.78-.39-1.01-.25-.23-.5-.33-1.08-.41l-1.16-.17a3.4 3.4 0 0 1-1.88-.78 2.41 2.41 0 0 1-.72-1.86c0-1.7 1.25-2.86 3.3-2.86 1.3 0 2.22.33 3.07 1.1l-.96.94a2.92 2.92 0 0 0-2.15-.75c-1.16 0-1.8.65-1.8 1.52 0 .35.1.67.37.9.25.22.65.38 1.11.45l1.13.17c.91.13 1.42.35 1.84.72.54.47.8 1.17.8 2 0 1.8-1.48 2.86-3.55 2.86Z" fill="#FEFEFE" data-webtasks-id="0f65d255-df60-410e"></path><path d="M104.16 7.09c-.2-.42-.6-.74-1.2-.74s-.99.32-1.18.74c-.1.25-.15.44-.16.75h2.7a2 2 0 0 0-.16-.75Zm-2.54 1.96c0 .9.56 1.57 1.55 1.57.78 0 1.16-.21 1.61-.66l1.08 1.04a3.4 3.4 0 0 1-2.7 1.11c-1.68 0-3.29-.76-3.29-3.62 0-2.3 1.26-3.6 3.1-3.6 1.97 0 3.1 1.44 3.1 3.37v.79h-4.45Zm-5.48-2.57C95.1 6.48 95 7.37 95 8.3c0 .94.1 1.85 1.15 1.85 1.05 0 1.18-.91 1.18-1.85 0-.93-.13-1.82-1.18-1.82Zm-.17 8.22c-1.1 0-1.84-.21-2.58-.92l1.1-1.11c.4.38.8.54 1.4.54 1.06 0 1.43-.74 1.43-1.46v-.72c-.47.51-1 .7-1.7.7-.69 0-1.29-.23-1.68-.62-.67-.66-.73-1.57-.73-2.8 0-1.24.06-2.13.73-2.8.4-.39 1-.62 1.7-.62.75 0 1.24.2 1.73.75v-.67h1.72v6.8c0 1.7-1.21 2.93-3.12 2.93Zm-5.76-2.67V7.76c0-.96-.61-1.28-1.17-1.28-.56 0-1.18.32-1.18 1.28v4.27h-1.78V4.97h1.73v.65a2.44 2.44 0 0 1 1.78-.73c.7 0 1.28.23 1.67.62.58.57.73 1.24.73 2v4.52H90.2Zm-7.1-2.98h-1.4c-.64 0-1 .3-1 .8 0 .49.33.81 1.02.81.5 0 .8-.04 1.12-.34.2-.17.26-.46.26-.89v-.38Zm.04 2.98v-.6c-.48.47-.93.67-1.74.67-.8 0-1.4-.2-1.82-.62-.38-.4-.58-.97-.58-1.59 0-1.12.77-2.05 2.42-2.05h1.68V7.5c0-.77-.38-1.11-1.32-1.11-.68 0-1 .16-1.37.58l-1.13-1.1c.7-.75 1.38-.97 2.57-.97 1.99 0 3.02.84 3.02 2.5v4.64h-1.73Zm-6.93 0v-4.3c0-.94-.6-1.25-1.15-1.25-.56 0-1.15.32-1.15 1.24v4.31h-1.77V2.38h1.77v3.24a2.35 2.35 0 0 1 1.7-.73c1.56 0 2.38 1.08 2.38 2.57v4.57h-1.78Zm-6.96.08c-1.42 0-3.18-.76-3.18-3.62 0-2.85 1.76-3.6 3.18-3.6.98 0 1.72.3 2.34.95l-1.2 1.2c-.36-.4-.68-.56-1.14-.56-.42 0-.75.14-1.01.46-.27.33-.4.8-.4 1.55s.13 1.24.4 1.58c.26.3.59.46 1 .46.47 0 .79-.16 1.15-.56l1.2 1.18c-.62.65-1.36.96-2.34.96Zm-5.53-.08-1.3-2.11-1.3 2.11H59l2.45-3.6-2.35-3.46h2.12L62.42 7l1.21-2.02h2.13L63.4 8.43l2.46 3.6h-2.13Zm-11.75 0V2.06h6.6V3.8h-4.65v2.33h3.96v1.74h-3.96v2.42h4.65v1.74h-6.6Z" fill="#2F96E8" data-webtasks-id="7b74329b-036e-4bc0"></path><path d="M0 3c0-1.1.9-2 2-2h8a2 2 0 0 1 2 2H0Z" fill="#8FD8F7" data-webtasks-id="4a8e40f9-55e6-45ea"></path><path d="M12 10H0c0 1.1.9 2 2 2h5v3l3-3a2 2 0 0 0 2-2Z" fill="#155397" data-webtasks-id="59ffd710-9a53-4fa9"></path><path fill="#46A2D9" d="M0 4h12v2H0z" data-webtasks-id="1ab2f8d2-083b-44ae"></path><path fill="#2D6DB5" d="M0 7h12v2H0z" data-webtasks-id="581cebe2-2346-4630"></path></svg> |
|
</a> |
|
<div class="topbar-dialog network-logo-dialog js-network-logo-dialog dno" id="topbar-network-logo-dialog" role="dialog" aria-labelledby="topbar-network-logo-dialog-title" aria-describedby="topbar-network-logo-dialog-body" data-webtasks-id="70ab786b-3909-4e31"> |
|
<div class="dialog-content" data-webtasks-id="15df5874-035c-4308"> |
|
<h4 class="bold" id="topbar-network-logo-dialog-title" data-webtasks-id="18b0c50b-1a96-46a7">Stack Exchange Network</h4> |
|
<p id="topbar-network-logo-dialog-body" data-webtasks-id="f25841ca-1bdc-426f"> |
|
Stack Exchange network consists of 182 Q&A communities including <a href="https://stackoverflow.com" data-webtasks-id="8fa9837f-0ecb-4be5">Stack Overflow</a>, the largest, most trusted online community for developers to learn, share their knowledge, and build their careers. |
|
</p> |
|
<a class="s-btn s-btn__filled" href="https://stackexchange.com" data-gps-track="stack_exchange_popup.click" data-webtasks-id="51fc04ad-d5cd-4ae5">Visit Stack Exchange</a> |
|
<button class="icon-close js-close-button s-btn s-btn__unset" aria-label="Close" data-webtasks-id="479e3900-3329-477b"><svg aria-hidden="true" class="svg-icon iconClear" width="18" height="18" viewBox="0 0 18 18" data-webtasks-id="2d3c41f1-bb38-46b3"><path d="M15 4.41 13.59 3 9 7.59 4.41 3 3 4.41 7.59 9 3 13.59 4.41 15 9 10.41 13.59 15 15 13.59 10.41 9 15 4.41Z" data-webtasks-id="6c2e15ed-4bf6-4f55"></path></svg></button> |
|
</div> |
|
</div> |
|
|
|
|
|
|
|
|
|
|
|
<form id="search" role="search" action="/search" class="s-topbar--searchbar js-searchbar " autocomplete="off" data-webtasks-id="b73deee0-b8f3-40ca"> |
|
<div class="s-topbar--searchbar--input-group" data-webtasks-id="fd0c404a-1b6c-4e35"> |
|
<input name="q" type="text" role="combobox" placeholder="Search on Travel…" value="" autocomplete="off" maxlength="240" class="s-input s-input__search js-search-field " aria-label="Search" aria-controls="top-search" data-controller="s-popover" data-action="focus->s-popover#show" data-s-popover-placement="bottom-start" aria-expanded="false" data-webtasks-id="59aeb526-7a2c-4b6c"> |
|
<svg aria-hidden="true" class="s-input-icon s-input-icon__search svg-icon iconSearch" width="18" height="18" viewBox="0 0 18 18" data-webtasks-id="10c4d985-5774-4e26"><path d="m18 16.5-5.14-5.18h-.35a7 7 0 1 0-1.19 1.19v.35L16.5 18l1.5-1.5ZM12 7A5 5 0 1 1 2 7a5 5 0 0 1 10 0Z" data-webtasks-id="383a7611-0a27-4a99"></path></svg> |
|
<div class="s-popover p0 wmx100 wmn4 sm:wmn-initial js-top-search-popover" id="top-search" role="menu" data-webtasks-id="613dfaa2-c982-4796"> |
|
<div class="s-popover--arrow" data-webtasks-id="8a6163da-cd46-4b92"></div> |
|
<div class="js-spinner p24 d-flex ai-center jc-center d-none" data-webtasks-id="a1caea6c-2110-4844"> |
|
<div class="s-spinner s-spinner__sm fc-orange-400" data-webtasks-id="c957180f-e299-4d75"> |
|
<div class="v-visible-sr" data-webtasks-id="d8cbb937-5cef-4d84">Loading…</div> |
|
</div> |
|
</div> |
|
|
|
<span class="v-visible-sr js-screen-reader-info" data-webtasks-id="341915ec-8fb2-4b1e"></span> |
|
<div class="js-ac-results overflow-y-auto hmx3 d-none" data-webtasks-id="3cbf5e69-6633-4eb2"></div> |
|
|
|
<div class="js-search-hints" aria-describedby="Tips for searching" data-webtasks-id="8de25042-d7d7-4b1a"></div> |
|
</div> |
|
</div> |
|
</form> |
|
|
|
<nav class="h100 ml-auto overflow-x-auto pr12" data-webtasks-id="637a5b41-7f98-40e5"> |
|
<ol class="s-topbar--content" role="menubar" data-webtasks-id="130271fc-816a-45f9"> |
|
<li role="none" data-webtasks-id="1f1e79a8-c7af-4d5e"><button class="s-topbar--item s-btn s-btn__icon s-btn__muted d-none sm:d-inline-flex js-searchbar-trigger" role="menuitem" aria-label="Search" aria-haspopup="true" aria-controls="search" title="Click to show search" data-webtasks-id="80bd9270-4bec-43a8"><svg aria-hidden="true" class="svg-icon iconSearch" width="18" height="18" viewBox="0 0 18 18" data-webtasks-id="04c6e619-951e-46f2"><path d="m18 16.5-5.14-5.18h-.35a7 7 0 1 0-1.19 1.19v.35L16.5 18l1.5-1.5ZM12 7A5 5 0 1 1 2 7a5 5 0 0 1 10 0Z" data-webtasks-id="443cffc5-b249-454a"></path></svg></button></li> |
|
|
|
<li role="none" data-webtasks-id="0a5f8732-e379-4bed"> |
|
<a href="/users/136878/webtasks-navigator" class="s-topbar--item s-user-card s-user-card__small m0 px12 js-gps-track" role="menuitem" data-gps-track="profile_summary.click()" data-webtasks-id="97195da9-264b-446f"> |
|
<div class="s-avatar s-avatar__24 s-user-card--avatar sm:m0" title="Webtasks Navigator" data-webtasks-id="df5d0b9b-b5d7-48ae"><img src="https://www.gravatar.com/avatar/64db2cea44c88e309a742529e139f6bd?s=48&d=identicon&r=PG&f=y&so-version=2" alt="Webtasks Navigator's user avatar" width="24" height="24" class="bar-sm s-avatar--image js-avatar-me" data-webtasks-id="c114e4cb-017f-477f"></div> <span class="v-visible-sr" data-webtasks-id="86caed50-c7aa-43f8">Webtasks Navigator</span> |
|
|
|
<div class="s-user-card--info sm:d-none" data-webtasks-id="0ac778a2-b376-4f12"> |
|
<ul class="s-user-card--awards" data-webtasks-id="3de91f47-fd66-4629"> |
|
<li class="s-user-card--rep js-header-rep" title="your reputation: 1" aria-hidden="true" data-webtasks-id="14ce475c-ecb9-43e3"> |
|
1 |
|
<span class="v-visible-sr" data-webtasks-id="8d608071-7eff-43ba">, 1 reputation</span> |
|
</li> |
|
<li class="-badges mr0" data-webtasks-id="a262c6e4-6c5f-4d4c"> |
|
</li> |
|
</ul> |
|
</div> |
|
</a> |
|
</li> |
|
<li role="none" data-webtasks-id="fa6cc327-e331-4135"> |
|
<a href="https://stackexchange.com/users/28364646?tab=inbox" class="s-topbar--item js-inbox-button" aria-label="Inbox" role="menuitem" title="Recent inbox messages" aria-haspopup="true" aria-expanded="false" data-unread-count="0" data-webtasks-id="9a14ecb0-5887-401d"> |
|
<svg aria-hidden="true" class="svg-icon iconInbox" width="20" height="18" viewBox="0 0 20 18" data-webtasks-id="0a9a1cd8-7bdd-41bb"><path d="M4.63 1h10.56a2 2 0 0 1 1.94 1.35L20 10.79V15a2 2 0 0 1-2 2H2a2 2 0 0 1-2-2v-4.21l2.78-8.44c.25-.8 1-1.36 1.85-1.35Zm8.28 12 2-2h2.95l-2.44-7.32a1 1 0 0 0-.95-.68H5.35a1 1 0 0 0-.95.68L1.96 11h2.95l2 2h6Z" data-webtasks-id="570f555b-96b8-4def"></path></svg> |
|
<span class="s-activity-indicator s-activity-indicator__danger js-unread-count d-none" data-webtasks-id="3231a535-47be-409e">0</span> |
|
</a> |
|
</li> |
|
<li role="none" data-webtasks-id="ea22adcc-0b41-4685"> |
|
<a href="https://stackexchange.com/users/28364646?tab=reputation" class="s-topbar--item js-achievements-button fc-success" data-unread-class="fc-success" aria-label="Achievements (unread)" role="menuitem" title="You have new achievements" aria-haspopup="true" aria-expanded="false" data-unread-count="0" data-lit-up="true" data-webtasks-id="f0ce182f-76d3-4f81"> |
|
<svg aria-hidden="true" class="svg-icon iconAchievements" width="18" height="18" viewBox="0 0 18 18" data-webtasks-id="88ba77a1-f11a-49a4"><path d="M15 2V1H3v1H0v4c0 1.6 1.4 3 3 3v1c.4 1.5 3 2.6 5 3v2H5s-1 1.5-1 2h10c0-.4-1-2-1-2h-3v-2c2-.4 4.6-1.5 5-3V9c1.6-.2 3-1.4 3-3V2h-3ZM3 7c-.5 0-1-.5-1-1V4h1v3Zm8.4 2.5L9 8 6.6 9.4l1-2.7L5 5h3l1-2.7L10 5h2.8l-2.3 1.8 1 2.7h-.1ZM16 6c0 .5-.5 1-1 1V4h1v2Z" data-webtasks-id="1d7bdebd-68f6-4147"></path></svg> |
|
<span class="s-activity-indicator s-activity-indicator__success js-unread-count d-none" data-webtasks-id="f8c5522a-4f48-46c6">+0</span> |
|
</a> |
|
</li> |
|
<li role="none" data-webtasks-id="1f6bd060-1c3a-40d9"> |
|
<a href="/help" class="s-topbar--item js-help-button" role="menuitem" title="Help Center and other resources" aria-haspopup="true" aria-controls="topbar-help-dialog" data-ga="["top navigation","help menu click",null,null,null]" data-webtasks-id="75e4e816-1137-415b" aria-expanded="false"><svg aria-hidden="true" class="svg-icon iconHelp" width="18" height="18" viewBox="0 0 18 18" data-webtasks-id="bff3e929-f276-4f74"><path d="M9 1C4.64 1 1 4.64 1 9c0 4.36 3.64 8 8 8 4.36 0 8-3.64 8-8 0-4.36-3.64-8-8-8Zm.81 12.13c-.02.71-.55 1.15-1.24 1.13-.66-.02-1.17-.49-1.15-1.2.02-.72.56-1.18 1.22-1.16.7.03 1.2.51 1.17 1.23ZM11.77 8c-.59.66-1.78 1.09-2.05 1.97a4 4 0 0 0-.09.75c0 .05-.03.16-.18.16H7.88c-.16 0-.18-.1-.18-.15.06-1.35.66-2.2 1.83-2.88.39-.29.7-.75.7-1.24.01-1.24-1.64-1.82-2.35-.72-.21.33-.18.73-.18 1.1H5.75c0-1.97 1.03-3.26 3.03-3.26 1.75 0 3.47.87 3.47 2.83 0 .57-.2 1.05-.48 1.44Z" data-webtasks-id="b203bfa2-8db3-459e"></path></svg></a> |
|
</li> |
|
<div class="topbar-dialog help-dialog js-help-dialog dno" id="topbar-help-dialog" role="menu" data-webtasks-id="d30c7c58-b891-4440"> |
|
<div class="modal-content" data-webtasks-id="dc76e8a2-ef81-4f06"> |
|
<ul data-webtasks-id="b9e88857-ca12-4d70"> |
|
<li data-webtasks-id="81bbae94-9dd2-4eba"> |
|
<a href="/tour" class="js-gps-track" data-gps-track="help_popup.click({ item_type:1 })" data-ga="["top navigation","tour submenu click",null,null,null]" data-webtasks-id="2e5d7495-4a19-4fda"> |
|
Tour |
|
<span class="item-summary" data-webtasks-id="88c5f782-f4da-4252"> |
|
Start here for a quick overview of the site |
|
</span> |
|
</a> |
|
</li> |
|
<li data-webtasks-id="868e60c5-ee9d-416e"> |
|
<a href="/help" class="js-gps-track" data-gps-track="help_popup.click({ item_type:4 })" data-ga="["top navigation","help center",null,null,null]" data-webtasks-id="0915ccc2-5875-46e3"> |
|
Help Center |
|
<span class="item-summary" data-webtasks-id="a2e9f661-be46-438f"> |
|
Detailed answers to any questions you might have |
|
</span> |
|
</a> |
|
</li> |
|
<li data-webtasks-id="b33cd4e2-e761-4c1a"> |
|
<a href="https://travel.meta.stackexchange.com" class="js-gps-track" data-gps-track="help_popup.click({ item_type:2 })" data-ga="["top navigation","meta submenu click",null,null,null]" data-webtasks-id="3d6970e4-14e0-4345"> |
|
Meta |
|
<span class="item-summary" data-webtasks-id="bede6ba8-199c-4fd7"> |
|
Discuss the workings and policies of this site |
|
</span> |
|
</a> |
|
</li> |
|
<li data-webtasks-id="86ee666d-c888-423b"> |
|
<a href="https://stackoverflow.co/" class="js-gps-track" data-gps-track="help_popup.click({ item_type:6 })" data-ga="["top navigation","about us submenu click",null,null,null]" data-webtasks-id="4865e600-aafb-45e2"> |
|
About Us |
|
<span class="item-summary" data-webtasks-id="ac45587d-6738-4d58"> |
|
Learn more about Stack Overflow the company, and our products. |
|
</span> |
|
</a> |
|
</li> |
|
</ul> |
|
</div> |
|
</div> |
|
<li role="none" data-webtasks-id="475a1b6e-c57e-45fc"> |
|
<a href="https://stackexchange.com" class="s-topbar--item js-site-switcher-button js-gps-track" data-gps-track="site_switcher.show" aria-label="Site switcher" role="menuitem" title="A list of all 182 Stack Exchange sites" aria-haspopup="true" aria-expanded="false" data-ga="["top navigation","stack exchange click",null,null,null]" data-webtasks-id="0adbcef4-3dd6-4f6f"> |
|
<svg aria-hidden="true" class="svg-icon iconStackExchange" width="18" height="18" viewBox="0 0 18 18" data-webtasks-id="666bc866-28d8-4f2d"><path d="M15 1H3a2 2 0 0 0-2 2v2h16V3a2 2 0 0 0-2-2ZM1 13c0 1.1.9 2 2 2h8v3l3-3h1a2 2 0 0 0 2-2v-2H1v2Zm16-7H1v4h16V6Z" data-webtasks-id="a3fdb3df-e17c-4035"></path></svg> |
|
</a> |
|
</li> |
|
|
|
|
|
<li class="js-topbar-dialog-corral" role="presentation" data-webtasks-id="eaaa92ee-95f7-4340"> |
|
|
|
|
|
<div class="topbar-dialog siteSwitcher-dialog dno" role="menu" data-webtasks-id="a0406ffd-81ad-4e3c"> |
|
<div class="header fw-wrap" data-webtasks-id="974ef128-a5f2-418e"> |
|
<h3 class="flex--item" data-webtasks-id="31a00c8e-ac99-42cd"> |
|
<a href="https://travel.stackexchange.com" data-webtasks-id="fdd4f4e7-277c-45ac">current community</a> |
|
</h3> |
|
<div class="flex--item fl1" data-webtasks-id="20885667-c6db-404e"> |
|
<div class="ai-center d-flex jc-end" data-webtasks-id="dd0fcd64-7e62-4946"> |
|
<button class="js-close-button s-btn s-btn__muted p0 ml8 d-none sm:d-block" type="button" aria-label="Close" data-webtasks-id="ad915c91-ddf8-4513"> |
|
<svg aria-hidden="true" class="svg-icon iconClear" width="18" height="18" viewBox="0 0 18 18" data-webtasks-id="fc6db7f3-7d72-4e35"><path d="M15 4.41 13.59 3 9 7.59 4.41 3 3 4.41 7.59 9 3 13.59 4.41 15 9 10.41 13.59 15 15 13.59 10.41 9 15 4.41Z" data-webtasks-id="14884d39-ca67-4bc6"></path></svg> |
|
</button> |
|
</div> |
|
</div> |
|
</div> |
|
<div class="modal-content bg-powder-050 current-site-container" data-webtasks-id="7bafdf13-0c36-4f6d"> |
|
<div class="d-none sm:d-block" data-webtasks-id="d5d14a03-0424-48a1"> |
|
<div class="d-flex ai-end" data-webtasks-id="8378fb92-3371-4bf4"> |
|
<div class="s-user-card flex--item fl1" data-webtasks-id="dfde6756-f158-4764"> |
|
<a href="/users/136878/webtasks-navigator" class="s-avatar s-avatar__32 s-user-card--avatar" data-webtasks-id="f3613c7a-0bd3-4dd7"> |
|
<div class="gravatar-wrapper-32" data-webtasks-id="5ed9defc-2f0b-4f95"><img src="https://www.gravatar.com/avatar/64db2cea44c88e309a742529e139f6bd?s=32&d=identicon&r=PG&f=y&so-version=2" alt="Webtasks Navigator's user avatar" width="32" height="32" class="bar-sm avatar s-avatar--image" data-webtasks-id="afe34047-51d2-4b1c"></div> |
|
</a> |
|
<div class="s-user-card--info" data-webtasks-id="3feefe63-0dff-4aa3"> |
|
<a href="/users/136878/webtasks-navigator" class="s-user-card--link" data-webtasks-id="708525a5-93c6-426e">Webtasks Navigator</a> |
|
<div class="s-user-card--awards lh-unset fc-black-700 fw-bold" data-webtasks-id="edb3eba4-c1a6-4d8b"> |
|
<span class="reputation-score" title="reputation score " dir="ltr" data-webtasks-id="b604a0a3-ed3e-40fb">1</span> |
|
</div> |
|
</div> |
|
</div> |
|
<div class="related-links flex--item p8" data-webtasks-id="55a9fee5-6982-4ac3"> |
|
<a href="https://travel.stackexchange.com/users/logout" data-gps-track="site_switcher.click({ item_type:8 })" data-webtasks-id="268af76e-a50a-4a0f">log out</a> |
|
</div> |
|
</div> |
|
</div> |
|
<ul class="current-site " data-webtasks-id="e328a205-2a1a-4296"> |
|
<li class="d-flex" data-webtasks-id="4dc2984e-39d0-4b64"> |
|
<div class="fl1" data-webtasks-id="82ed7309-b50c-490a"> |
|
<a href="https://travel.stackexchange.com" class="current-site-link site-link js-gps-track d-flex gs8 gsx" data-id="273" data-gps-track="site_switcher.click({ item_type:3 })" data-webtasks-id="0e08a623-b9a5-4d46"> |
|
<div class="favicon favicon-travel site-icon flex--item" title="Travel" data-webtasks-id="7718b80c-5a8c-4388"></div> |
|
<span class="flex--item fl1" data-webtasks-id="3579f48a-0c7f-418c"> |
|
Travel |
|
</span> |
|
</a> |
|
|
|
</div> |
|
<div class="related-links" data-webtasks-id="b93e497e-dd0f-4f0d"> |
|
<a href="https://travel.stackexchange.com/help" class="js-gps-track" data-gps-track="site_switcher.click({ item_type:14 })" data-webtasks-id="3bdc22fa-2da2-407e">help</a> |
|
<a href="https://chat.stackexchange.com?tab=site&host=travel.stackexchange.com" class="js-gps-track" data-gps-track="site_switcher.click({ item_type:6 })" data-webtasks-id="c508f695-f725-497a">chat</a> |
|
<a href="https://travel.stackexchange.com/users/logout" class="js-gps-track" data-gps-track="site_switcher.click({ item_type:8 })" data-webtasks-id="a9981d4a-c19e-4a2b">log out</a> |
|
</div> |
|
|
|
</li> |
|
<li class="related-site d-flex" data-webtasks-id="9b0b7be0-a8c4-467e"> |
|
<div class="L-shaped-icon-container" data-webtasks-id="094d55fb-7edc-4fe6"> |
|
<span class="L-shaped-icon" data-webtasks-id="ad781498-1568-48df"></span> |
|
</div> |
|
|
|
<a href="https://travel.meta.stackexchange.com" class=" site-link js-gps-track d-flex gs8 gsx" data-id="275" data-gps-track="site.switch({ target_site:275, item_type:3 }),site_switcher.click({ item_type:4 })" data-webtasks-id="e6128f98-306e-48bd"> |
|
<div class="favicon favicon-travelmeta site-icon flex--item" title="Travel Meta" data-webtasks-id="a62c49dc-22fe-499c"></div> |
|
<span class="flex--item fl1" data-webtasks-id="485dd0b8-6922-416b"> |
|
Travel Meta |
|
</span> |
|
</a> |
|
|
|
</li> |
|
</ul> |
|
</div> |
|
|
|
<div class="header" id="your-communities-header" data-webtasks-id="c81cf8b3-d619-45ee"> |
|
<h3 data-webtasks-id="51903f19-83a1-480a"> |
|
<a href="https://stackexchange.com/users/28364646/?tab=accounts" data-webtasks-id="bc9252a0-4153-4d3d">your communities</a> |
|
</h3> |
|
|
|
<a href="#" id="edit-pinned-sites" data-webtasks-id="7222d011-207d-4d8f">edit</a> |
|
<a href="#" id="cancel-pinned-sites" style="display: none;" data-webtasks-id="f9930e82-e63e-47ea">cancel</a> |
|
</div> |
|
<div class="modal-content" id="your-communities-section" data-webtasks-id="5fbc0a7b-99fe-4b38"> |
|
|
|
<ul class="my-sites" data-webtasks-id="ea09d81b-fe0d-4efe"> |
|
<li data-webtasks-id="fcf37624-6ecb-4c41"> |
|
<a href="https://stackoverflow.com" class=" site-link js-gps-track d-flex gs8 gsx" data-id="1" data-gps-track="site.switch({ target_site:1, item_type:3 }),site_switcher.click({ item_type:1 })" data-webtasks-id="2a7ea006-c0b5-48e4"> |
|
<div class="favicon favicon-stackoverflow site-icon flex--item" title="Stack Overflow" data-webtasks-id="ad86af26-ff5d-4a68"></div> |
|
<span class="flex--item fl1" data-webtasks-id="367f4490-0ea5-4f4b"> |
|
Stack Overflow |
|
</span> |
|
<span class="rep-score flex--item" data-webtasks-id="8488fd9b-976d-49be">1</span> |
|
</a> |
|
|
|
</li> |
|
<li data-webtasks-id="5df094e9-7d88-431e"> |
|
<a href="https://travel.stackexchange.com" class=" site-link js-gps-track d-flex gs8 gsx" data-id="273" data-gps-track="site.switch({ target_site:273, item_type:3 }),site_switcher.click({ item_type:1 })" data-webtasks-id="f674ffc2-e678-4e65"> |
|
<div class="favicon favicon-travel site-icon flex--item" title="Travel" data-webtasks-id="a3760e0a-4b59-49a2"></div> |
|
<span class="flex--item fl1" data-webtasks-id="24129803-4c31-49b5"> |
|
Travel |
|
</span> |
|
<span class="rep-score flex--item" data-webtasks-id="a12c2014-c165-40be">1</span> |
|
</a> |
|
|
|
</li> |
|
</ul> |
|
<div class="pinned-site-editor-container" style="display: none;" data-webtasks-id="6daf72e1-4aae-49bc"> |
|
<input type="hidden" name="fkey" value="fe76f3281bad2063ad30f357d65244f2508ca27d494a61fe42e4d6aea23fa56d" data-webtasks-id="ff79a44c-68ed-4484"> |
|
<div class="d-flex p12" data-webtasks-id="3ed96f78-a657-443e"> |
|
<div class="flex--item fl1 ps-relative" data-webtasks-id="ecb42836-2ed9-450a"> |
|
<input id="js-site-search-txt" type="text" class="s-input s-input__sm w100 h100 site-filter-input" value="" placeholder="Add a Stack Exchange community" data-webtasks-id="24fe7f3f-6ef5-4093"> |
|
<ul class="js-found-sites found-sites hmx1 l0 r0 ps-absolute overflow-y-auto" data-webtasks-id="e1748446-ae75-4707"></ul> |
|
</div> |
|
<button class="ml8 s-btn s-btn__sm" type="submit" id="pin-site-btn" disabled="disabled" data-webtasks-id="498af096-3bcb-4b81">Add</button> |
|
</div> |
|
<ul class="pinned-site-list sortable" data-custom-list="False" data-webtasks-id="2a38057e-8a65-446b"> |
|
</ul> |
|
<div class="d-flex p12 jc-end" data-webtasks-id="03025767-849f-4e3c"> |
|
<a class="flex--item s-btn s-btn__sm js-reset-pinned-sites" href="#" data-webtasks-id="d3ad261c-dd81-4569">Reset</a> |
|
<button class="ml8 flex--item s-btn s-btn__primary s-btn__sm" type="submit" id="save-pinned-sites-btn" disabled="disabled" data-webtasks-id="de2d8a35-e1d1-4e12">Save</button> |
|
</div> |
|
</div> |
|
</div> |
|
|
|
<div class="header" data-webtasks-id="68fa603e-6688-4f1f"> |
|
<h3 data-webtasks-id="9cd2f11d-bc85-4e5e"><a href="https://stackexchange.com/sites" data-webtasks-id="be2c4733-7818-4a28">more stack exchange communities</a> |
|
</h3> |
|
<a href="https://stackoverflow.blog" class="float-right" data-webtasks-id="bf56062a-1e04-4784">company blog</a> |
|
</div> |
|
<div class="modal-content" data-webtasks-id="5fd7736b-5747-420f"> |
|
<div class="child-content" data-webtasks-id="7c52bdfa-7db5-499c"></div> |
|
</div> |
|
</div> |
|
|
|
</li> |
|
|
|
</ol> |
|
</nav> |
|
|
|
|
|
</div> |
|
</header> |
|
|
|
<script data-webtasks-id="6f7c5030-64a0-4aef"> |
|
StackExchange.ready(function () { StackExchange.topbar.init(); }); |
|
StackExchange.scrollPadding.setPaddingTop(50, 10); |
|
</script> |
|
|
|
|
|
|
|
|
|
<header class="site-header" data-webtasks-id="a7fbeea9-6116-4230"> |
|
<div class="site-header--container" data-webtasks-id="95ba076a-5d38-4f24"> |
|
<a class="site-header--link d-flex ai-center fs-headline1 fw-bold" href="https://travel.stackexchange.com" data-webtasks-id="e7c98ede-429b-4abb"> |
|
<img class="h-auto wmx100" src="https://cdn.sstatic.net/Sites/travel/Img/logo.svg?v=556a1fc4f70a" alt="Travel" data-webtasks-id="46ad7be4-5702-43b4"> |
|
</a> |
|
|
|
</div> |
|
</header> |
|
|
|
<div class="container" data-webtasks-id="af43090c-3ce2-44c7"> |
|
|
|
|
|
<div id="left-sidebar" data-is-here-when="md lg" class="left-sidebar js-pinned-left-sidebar ps-relative" data-webtasks-id="b1a0e8ce-da10-4d24"> |
|
<div class="left-sidebar--sticky-container js-sticky-leftnav" data-webtasks-id="b58d296b-e067-4345"> |
|
<nav role="navigation" data-webtasks-id="8ede3cdc-1bd7-4d2a"> |
|
|
|
<ol class="nav-links" data-webtasks-id="4ac2fdf5-b29f-4b21"> |
|
|
|
|
|
<li class="ps-relative" aria-current="false" data-webtasks-id="031deab7-85b4-4da9"> |
|
|
|
|
|
<a href="/" class="pl8 js-gps-track nav-links--link" data-gps-track="top_nav.click({is_current: false, location:2, destination:8})" aria-controls="" data-controller="" data-s-popover-placement="right" aria-current="false" data-s-popover-auto-show="true" data-s-popover-hide-on-outside-click="never" data-webtasks-id="dc1fe606-f3ce-4d85"> |
|
<div class="d-flex ai-center" data-webtasks-id="c7046ebc-c47b-4e9d"> |
|
<div class="flex--item truncate" data-webtasks-id="3f113361-11e3-4dfd"> |
|
Home |
|
</div> |
|
</div> |
|
</a> |
|
</li> |
|
|
|
|
|
<li data-webtasks-id="0153ad7f-9d74-44ec"> |
|
<ol class="nav-links" data-webtasks-id="8ae5a02a-46d1-4f17"> |
|
|
|
<li class="fs-fine tt-uppercase ml8 mt16 mb4 fc-light" data-webtasks-id="7b91ac92-f035-4147">Public</li> |
|
|
|
|
|
|
|
<li class="ps-relative youarehere" aria-current="true" data-webtasks-id="e61cfce8-cf32-49d3"> |
|
|
|
|
|
<a id="nav-questions" href="/questions" class="pl8 js-gps-track nav-links--link -link__with-icon" data-gps-track="top_nav.click({is_current: true, location:2, destination:1})" aria-controls="" data-controller="" data-s-popover-placement="right" aria-current="false" data-s-popover-auto-show="true" data-s-popover-hide-on-outside-click="never" data-webtasks-id="3bd19069-17ec-41b5"> |
|
<svg aria-hidden="true" class="svg-icon iconGlobe" width="18" height="18" viewBox="0 0 18 18" data-webtasks-id="a37bba9e-ae86-4e3f"><path d="M9 1C4.64 1 1 4.64 1 9c0 4.36 3.64 8 8 8 4.36 0 8-3.64 8-8 0-4.36-3.64-8-8-8ZM8 15.32a6.46 6.46 0 0 1-4.3-2.74 6.46 6.46 0 0 1-.93-5.01L7 11.68v.8c0 .88.12 1.32 1 1.32v1.52Zm5.72-2c-.2-.66-1-1.32-1.72-1.32h-1v-2c0-.44-.56-1-1-1H6V7h1c.44 0 1-.56 1-1V5h2c.88 0 1.4-.72 1.4-1.6v-.33a6.45 6.45 0 0 1 3.83 4.51 6.45 6.45 0 0 1-1.51 5.73v.01Z" data-webtasks-id="874d1af4-41c7-4b1e"></path></svg> <span class="-link--channel-name" data-webtasks-id="711e343f-34fa-4821">Questions</span> |
|
</a> |
|
</li> |
|
|
|
|
|
|
|
|
|
|
|
<li class="ps-relative" aria-current="false" data-webtasks-id="24562cdc-cd52-4d29"> |
|
|
|
|
|
<a id="nav-tags" href="/tags" class=" js-gps-track nav-links--link" data-gps-track="top_nav.click({is_current: false, location:2, destination:2})" aria-controls="" data-controller="" data-s-popover-placement="right" aria-current="false" data-s-popover-auto-show="true" data-s-popover-hide-on-outside-click="never" data-webtasks-id="7a258f04-8f90-4ab5"> |
|
<div class="d-flex ai-center" data-webtasks-id="d63a6e14-ca42-443a"> |
|
<div class="flex--item truncate" data-webtasks-id="aac11021-3190-4377"> |
|
Tags |
|
</div> |
|
</div> |
|
</a> |
|
</li> |
|
|
|
|
|
|
|
|
|
<li class="ps-relative" aria-current="false" data-webtasks-id="a9ce2566-1e20-4c2b"> |
|
|
|
|
|
<a id="nav-users" href="/users" class=" js-gps-track nav-links--link" data-gps-track="top_nav.click({is_current: false, location:2, destination:3})" aria-controls="" data-controller="" data-s-popover-placement="right" aria-current="false" data-s-popover-auto-show="true" data-s-popover-hide-on-outside-click="never" data-webtasks-id="646ec798-af04-42c7"> |
|
<div class="d-flex ai-center" data-webtasks-id="399103cf-c6b6-4cf3"> |
|
<div class="flex--item truncate" data-webtasks-id="c9df3c29-df7c-4c49"> |
|
Users |
|
</div> |
|
</div> |
|
</a> |
|
</li> |
|
|
|
|
|
|
|
|
|
|
|
<li class="ps-relative" aria-current="false" data-webtasks-id="dcc3fa6d-639f-461d"> |
|
|
|
|
|
<a id="nav-unanswered" href="/unanswered" class=" js-gps-track nav-links--link" data-gps-track="top_nav.click({is_current: false, location:2, destination:5})" aria-controls="" data-controller="" data-s-popover-placement="right" aria-current="false" data-s-popover-auto-show="true" data-s-popover-hide-on-outside-click="never" data-webtasks-id="8749b5a9-b9db-4164"> |
|
<div class="d-flex ai-center" data-webtasks-id="b55494b6-f5a4-478e"> |
|
<div class="flex--item truncate" data-webtasks-id="e3b07c75-623f-4ebc"> |
|
Unanswered |
|
</div> |
|
</div> |
|
</a> |
|
</li> |
|
|
|
|
|
|
|
</ol> |
|
</li> |
|
|
|
|
|
|
|
<li class="d-flex ai-center jc-space-between ml8 mt24 mb4 js-create-team-cta" data-webtasks-id="356443cf-c1e6-4fbf"> |
|
<div class="flex--item tt-uppercase fs-fine fc-light" data-webtasks-id="edbb0136-e6b4-4880">Teams</div> |
|
<div class="flex--item" data-webtasks-id="e3ac3174-bf35-4c75"> |
|
<a href="javascript:void(0)" class="s-link p12 fc-black-500 h:fc-black-800 js-gps-track" role="button" aria-controls="popover-teams-create-cta" data-controller="s-popover" data-action="s-popover#toggle" data-s-popover-placement="bottom-start" data-s-popover-toggle-class="is-selected" data-gps-track="teams.create.left-sidenav.click({ Action: ShowInfo })" data-ga="["teams left navigation","left nav show teams info",null,null,null]" aria-expanded="false" data-webtasks-id="b74cbe16-b9e0-4e76"> |
|
<svg aria-hidden="true" class="svg-icon iconInfoSm" width="14" height="14" viewBox="0 0 14 14" data-webtasks-id="0c8096b6-ee36-4519"><path d="M7 1a6 6 0 1 1 0 12A6 6 0 0 1 7 1Zm1 10V6H6v5h2Zm0-6V3H6v2h2Z" data-webtasks-id="007eeb61-2a21-4496"></path></svg> |
|
</a> |
|
|
|
</div> |
|
</li> |
|
<li class="ps-relative js-create-team-cta" data-webtasks-id="413866ec-3243-43de"> |
|
<a href="https://stackoverflowteams.com/teams/create/free/?utm_source=so-owned&utm_medium=side-bar&utm_campaign=campaign-38&utm_content=cta" class="pl8 js-gps-track nav-links--link" title="Stack Overflow for Teams is a private, secure spot for your organization's questions and answers." data-gps-track="teams.create.left-sidenav.click({ Action: FreemiumTeamsCreateClick })" data-ga="["teams left navigation","left nav team click","stackoverflow.com/teams/create/free",null,null]" data-webtasks-id="6ac5f1cb-de21-4150"> |
|
<div class="d-flex ai-center" data-webtasks-id="e7156994-c9a6-4a97"> |
|
<div class="flex--item s-avatar va-middle bg-orange-400" data-webtasks-id="7b324972-af36-46e0"> |
|
<div class="s-avatar--letter mtn1" data-webtasks-id="849f404a-f453-4caf"> |
|
<svg aria-hidden="true" class="svg-icon iconBriefcaseSm" width="14" height="14" viewBox="0 0 14 14" data-webtasks-id="5a48f0e3-1c5c-4064"><path d="M4 3a1 1 0 0 1 1-1h4a1 1 0 0 1 1 1v1h.5c.83 0 1.5.67 1.5 1.5v5c0 .83-.67 1.5-1.5 1.5h-7A1.5 1.5 0 0 1 2 10.5v-5C2 4.67 2.67 4 3.5 4H4V3Zm5 1V3H5v1h4Z" data-webtasks-id="da0bcc1a-e983-4f57"></path></svg> |
|
</div> |
|
<svg aria-hidden="true" class="native s-avatar--badge svg-icon iconShieldXSm" width="9" height="10" viewBox="0 0 9 10" data-webtasks-id="e7e76ae1-b80f-49cd"><path fill="var(--white)" d="M0 1.84 4.5 0 9 1.84v3.17C9 7.53 6.3 10 4.5 10 2.7 10 0 7.53 0 5.01V1.84Z" data-webtasks-id="de6fa1a5-3926-473e"></path><path fill="var(--black-500)" d="M1 2.5 4.5 1 8 2.5v2.51C8 7.34 5.34 9 4.5 9 3.65 9 1 7.34 1 5.01V2.5Zm2.98 3.02L3.2 7h2.6l-.78-1.48a.4.4 0 0 1 .15-.38c.34-.24.73-.7.73-1.14 0-.71-.5-1.23-1.41-1.23-.92 0-1.39.52-1.39 1.23 0 .44.4.9.73 1.14.12.08.18.23.15.38Z" data-webtasks-id="8faab64b-4ad6-4ff4"></path></svg> |
|
</div> |
|
<div class="flex--item pl6" data-webtasks-id="2814936d-127e-4742"> |
|
Create free Team |
|
</div> |
|
</div> |
|
</a> |
|
</li> |
|
<li class="pl1 pr8" data-webtasks-id="ae125168-59bb-4194"> |
|
<button class="s-btn s-btn__link w100 fc-blue-700 h:fc-blue-900 p8 fs-caption mr8 mt8 d-block bar-md bg-blue-050 lh-md js-gps-track" aria-controls="teams-were-moved-popover" aria-expanded="false" data-controller="s-popover" data-action="s-popover#toggle" data-s-popover-placement="bottom-start" data-gps-track="teams.sot_migration_notification_teams_moved.click()" data-webtasks-id="295cafad-33b0-4130"> |
|
Looking for your Teams? |
|
</button> |
|
</li> |
|
|
|
|
|
</ol> |
|
</nav> |
|
</div> |
|
|
|
<div class="s-popover mt4 ws3" id="teams-were-moved-popover" role="menu" data-webtasks-id="4ca7d613-61e0-4163"> |
|
<div class="s-popover--arrow" data-webtasks-id="0b13a49d-983a-4650"></div> |
|
<p data-webtasks-id="d889f6f9-8d2c-4620">Stack Overflow for Teams has its own domain!</p> |
|
<p data-webtasks-id="40415868-12be-463b">You can now access your Teams at <a href="https://stackoverflowteams.com" class="s-link fs-italic" data-webtasks-id="4b6c1252-1c47-45f5">stackoverflowteams.com</a>. Teams no longer appear in the left sidebar on <span class="fs-italic" data-webtasks-id="12c6dd31-f3c0-489c">stackoverflow.com</span>. Check your email to learn more about these changes.</p> |
|
</div> |
|
|
|
|
|
<div class="s-popover ws2" id="popover-teams-create-cta" role="menu" aria-hidden="true" data-webtasks-id="d0680090-13f5-42a7"> |
|
<div class="s-popover--arrow" data-webtasks-id="379c5829-01ee-4e14"></div> |
|
|
|
<div class="ps-relative overflow-hidden" data-webtasks-id="d2f7c86c-18ac-4802"> |
|
<p class="mb2" data-webtasks-id="cf90ed52-12b5-4094"><strong data-webtasks-id="512b7999-c203-4607">Teams</strong></p> |
|
<p class="mb12 fs-caption fc-black-400" data-webtasks-id="eacb3b11-bc26-4a51">Q&A for work</p> |
|
<p class="mb12 fs-caption fc-medium" data-webtasks-id="443bd247-543f-47cb">Connect and share knowledge within a single location that is structured and easy to search.</p> |
|
<a href="https://stackoverflow.co/teams/" class="js-gps-track s-btn s-btn__primary s-btn__xs" data-gps-track="teams.create.left-sidenav.click({ Action: CtaClick })" data-ga="["teams left navigation","left nav cta","stackoverflow.com/teams",null,null]" data-webtasks-id="c0332c8b-4275-4db5"> |
|
Learn more about Teams |
|
</a> |
|
</div> |
|
|
|
<div class="ps-absolute t8 r8" data-webtasks-id="3639fcb2-74a4-4b0d"> |
|
<svg aria-hidden="true" class="fc-orange-500 svg-spot spotPeople" width="48" height="48" viewBox="0 0 48 48" data-webtasks-id="b46f23df-48c7-4253"><path d="M13.5 28a4.5 4.5 0 1 0 0-9 4.5 4.5 0 0 0 0 9ZM7 30a1 1 0 0 1 1-1h11a1 1 0 0 1 1 1v5h11v-5a1 1 0 0 1 1-1h12a1 1 0 0 1 1 1v10a2 2 0 0 1-2 2H33v5a1 1 0 0 1-1 1H20a1 1 0 0 1-1-1v-5H8a1 1 0 0 1-1-1V30Zm25-6.5a4.5 4.5 0 1 0 9 0 4.5 4.5 0 0 0-9 0ZM24.5 34a4.5 4.5 0 1 0 0-9 4.5 4.5 0 0 0 0 9Z" opacity=".2" data-webtasks-id="6b6d09a3-46bf-4d0d"></path><path d="M16.4 26.08A6 6 0 1 0 7.53 26C5.64 26.06 4 27.52 4 29.45V40a1 1 0 0 0 1 1h9a1 1 0 1 0 0-2h-4v-7a1 1 0 1 0-2 0v7H6v-9.55c0-.73.67-1.45 1.64-1.45H16a1 1 0 0 0 .4-1.92ZM12 18a4 4 0 1 1 0 8 4 4 0 0 1 0-8Zm16.47 14a6 6 0 1 0-8.94 0A3.6 3.6 0 0 0 16 35.5V46a1 1 0 0 0 1 1h14a1 1 0 0 0 1-1V35.5c0-1.94-1.64-3.42-3.53-3.5ZM20 28a4 4 0 1 1 8 0 4 4 0 0 1-8 0Zm-.3 6h8.6c1 0 1.7.75 1.7 1.5V45h-2v-7a1 1 0 1 0-2 0v7h-4v-7a1 1 0 1 0-2 0v7h-2v-9.5c0-.75.7-1.5 1.7-1.5ZM42 22c0 1.54-.58 2.94-1.53 4A3.5 3.5 0 0 1 44 29.45V40a1 1 0 0 1-1 1h-9a1 1 0 1 1 0-2h4v-7a1 1 0 1 1 2 0v7h2v-9.55A1.5 1.5 0 0 0 40.48 28H32a1 1 0 0 1-.4-1.92A6 6 0 1 1 42 22Zm-2 0a4 4 0 1 0-8 0 4 4 0 0 0 8 0Z" data-webtasks-id="da006bcb-f35a-4d7e"></path><g opacity=".35" data-webtasks-id="6f49fcc0-21fb-416c"><path d="M17 10a1 1 0 011-1h12a1 1 0 110 2H18a1 1 0 01-1-1Zm1-5a1 1 0 100 2h12a1 1 0 100-2H18ZM14 1a1 1 0 00-1 1v12a1 1 0 001 1h5.09l4.2 4.2a1 1 0 001.46-.04l3.7-4.16H34a1 1 0 001-1V2a1 1 0 00-1-1H14Zm1 12V3h18v10h-5a1 1 0 00-.75.34l-3.3 3.7-3.74-3.75a1 1 0 00-.71-.29H15Z" data-webtasks-id="5d4b5eef-883d-4f12"></path></g></svg> |
|
</div> |
|
</div> |
|
|
|
|
|
</div> |
|
|
|
|
|
|
|
<div id="content" class="" data-webtasks-id="9a554a83-ca07-476b"> |
|
|
|
|
|
|
|
<div itemprop="mainEntity" itemscope="" itemtype="https://schema.org/Question" data-webtasks-id="4d290581-43fc-4911"> |
|
<link itemprop="image" href="https://cdn.sstatic.net/Sites/travel/Img/apple-touch-icon.png?v=0bf351ad5aa0" data-webtasks-id="8a97d751-d42f-46aa"> |
|
|
|
<div class="inner-content clearfix" data-webtasks-id="520a09dd-6938-43fe"> |
|
|
|
|
|
|
|
<div id="question-header" class="d-flex sm:fd-column" data-webtasks-id="59c9dcc7-04b8-480f"> |
|
<h1 itemprop="name" class="fs-headline1 ow-break-word mb8 flex--item fl1" data-webtasks-id="0d197248-ce19-4505"><a href="/questions/181865/are-these-murals-of-a-real-location-perhaps-bavaria" class="question-hyperlink" data-webtasks-id="916d9b35-b35f-445f">Are these murals of a real location? Perhaps Bavaria?</a></h1> |
|
<div class="ml12 aside-cta flex--item print:d-none sm:ml0 sm:mb12 sm:order-first sm:as-end" data-webtasks-id="d474a2d8-c595-46b3"> |
|
<a href="/questions/ask" class="ws-nowrap s-btn s-btn__primary" data-webtasks-id="f1644e36-7a9f-40ed"> |
|
Ask Question |
|
</a> |
|
|
|
</div> |
|
</div> |
|
<div class="d-flex fw-wrap pb8 mb16 bb bc-black-075" data-webtasks-id="8f217507-5d1a-4556"> |
|
<div class="flex--item ws-nowrap mr16 mb8" title="2023-06-19 22:58:14Z" data-webtasks-id="940cb6f3-c64b-4c56"> |
|
<span class="fc-light mr2" data-webtasks-id="821b3ed2-609d-4915">Asked</span> |
|
<time itemprop="dateCreated" datetime="2023-06-19T22:58:14" data-webtasks-id="fcee136b-8364-44a8">20 days ago</time> |
|
</div> |
|
<div class="flex--item ws-nowrap mr16 mb8" data-webtasks-id="7ef9bad1-fcbf-414c"> |
|
<span class="fc-light mr2" data-webtasks-id="df686e73-28ec-4319">Modified</span> |
|
<a href="?lastactivity" class="s-link s-link__inherit" title="2023-06-22 07:36:04Z" data-webtasks-id="bde5c14a-9a1c-4dc5">17 days ago</a> |
|
</div> |
|
<div class="flex--item ws-nowrap mb8" title="Viewed 6,700 times" data-webtasks-id="5bc1f265-a307-474e"> |
|
<span class="fc-light mr2" data-webtasks-id="9b4d7542-ede2-4c67">Viewed</span> |
|
7k times |
|
</div> |
|
</div> |
|
<div id="mainbar" role="main" aria-label="question and answers" data-webtasks-id="cf0b1127-09a3-4031"> |
|
|
|
|
|
<div class="question js-question" data-questionid="181865" data-position-on-page="0" data-score="26" id="question" data-webtasks-id="53f8ef09-452b-4f25"> |
|
<style data-webtasks-id="c6ffdcb9-1297-4f98"> |
|
</style> |
|
<div class="js-zone-container zone-container-main" data-webtasks-id="086ee1e6-046e-4cde"> |
|
<div id="dfp-tlb" class="everyonelovesstackoverflow everyoneloves__top-leaderboard everyoneloves__leaderboard" data-dfp-zone="true" data-webtasks-id="b9c8315c-79d1-4c33" style="min-height: auto; height: auto; display: none;" data-google-query-id="CPu30d7Eg4ADFQrXcwEdE3gF6w"><div id="google_ads_iframe_/248424177/travel.stackexchange.com/lb/question-pages_0__container__" data-webtasks-id="9f93b3c8-c0b3-4e79" style="border: 0pt none; width: 728px; height: 0px;"></div></div> |
|
<div class="js-report-ad-button-container " style="width: 728px" data-webtasks-id="6054f18e-e46c-4f67"></div> |
|
</div> |
|
|
|
<div class="post-layout " data-webtasks-id="4c8f800e-63d4-438e"> |
|
<div class="votecell post-layout--left" data-webtasks-id="ff4a15bb-fa39-4f56"> |
|
<div class="js-voting-container d-flex jc-center fd-column ai-stretch gs4 fc-black-200" data-post-id="181865" data-webtasks-id="277f2eb1-c346-4e12"> |
|
<button class="js-vote-up-btn flex--item s-btn ba bar-pill c-pointer as-center bc-black-100 fc-black-700 h:bg-theme-primary-100" data-controller="s-tooltip" data-s-tooltip-placement="right" aria-pressed="false" aria-label="Up vote" data-selected-classes="fc-theme-primary bc-theme-primary bg-theme-primary-025" data-unselected-classes="bc-black-100 fc-black-700 h:bg-theme-primary-100" aria-describedby="--stacks-s-tooltip-tk556jeb" data-webtasks-id="90994013-0d66-4a90"> |
|
<svg aria-hidden="true" class="svg-icon iconArrowUp" width="18" height="18" viewBox="0 0 18 18" data-webtasks-id="1b4226f3-e1e9-4cda"><path d="M1 12h16L9 4l-8 8Z" data-webtasks-id="4413f8b3-1f95-40d6"></path></svg> |
|
</button><div id="--stacks-s-tooltip-tk556jeb" class="s-popover s-popover__tooltip" role="tooltip" data-webtasks-id="f888a045-4b8b-4a40">This question shows research effort; it is useful and clear<div class="s-popover--arrow" data-webtasks-id="a511f343-fa3a-4756"></div></div> |
|
<div class="js-vote-count flex--item d-flex fd-column ai-center fc-theme-body-font fw-bold fs-subheading py4" itemprop="upvoteCount" data-value="26" data-webtasks-id="a3b0a45e-de19-4b12"> |
|
26 |
|
</div> |
|
<button class="js-vote-down-btn flex--item mb8 s-btn ba bar-pill c-pointer as-center bc-black-100 fc-black-700 h:bg-theme-primary-100" data-controller="s-tooltip" data-s-tooltip-placement="right" aria-pressed="false" aria-label="Down vote" data-selected-classes="fc-theme-primary bc-theme-primary bg-theme-primary-025" data-unselected-classes="bc-black-100 fc-black-700 h:bg-theme-primary-100" aria-describedby="--stacks-s-tooltip-pe2yl3te" data-webtasks-id="1b477a0e-5715-4c42"> |
|
<svg aria-hidden="true" class="svg-icon iconArrowDown" width="18" height="18" viewBox="0 0 18 18" data-webtasks-id="1d8c0aeb-f168-4215"><path d="M1 6h16l-8 8-8-8Z" data-webtasks-id="0a4e2ff1-7c5f-4216"></path></svg> |
|
</button><div id="--stacks-s-tooltip-pe2yl3te" class="s-popover s-popover__tooltip" role="tooltip" data-webtasks-id="e4298c08-3770-4b9c">This question does not show any research effort; it is unclear or not useful<div class="s-popover--arrow" data-webtasks-id="10e8275a-ebbd-43ed"></div></div> |
|
|
|
|
|
|
|
<button class="js-saves-btn s-btn s-btn__unset c-pointer py4" type="button" id="saves-btn-181865" data-controller="s-tooltip" data-s-tooltip-placement="right" data-s-popover-placement="" aria-pressed="false" data-post-id="181865" data-post-type-id="1" data-user-privilege-for-post-click="-1" aria-controls="" data-s-popover-auto-show="false" aria-describedby="--stacks-s-tooltip-gc6ikpoy" data-webtasks-id="d199b08e-27e8-4858"> |
|
<svg aria-hidden="true" class="fc-theme-primary-500 js-saves-btn-selected d-none svg-icon iconBookmark" width="18" height="18" viewBox="0 0 18 18" data-webtasks-id="5e1a6dde-d7bb-41e5"><path d="M3 17V3c0-1.1.9-2 2-2h8a2 2 0 0 1 2 2v14l-6-4-6 4Z" data-webtasks-id="1fceeb0f-b519-4a08"></path></svg> |
|
<svg aria-hidden="true" class="js-saves-btn-unselected svg-icon iconBookmarkAlt" width="18" height="18" viewBox="0 0 18 18" data-webtasks-id="de526be4-cf82-47ee"><path d="m9 10.6 4 2.66V3H5v10.26l4-2.66ZM3 17V3c0-1.1.9-2 2-2h8a2 2 0 0 1 2 2v14l-6-4-6 4Z" data-webtasks-id="bc931b3f-d5cf-4c4f"></path></svg> |
|
</button><div id="--stacks-s-tooltip-gc6ikpoy" class="s-popover s-popover__tooltip" role="tooltip" data-webtasks-id="0de7630f-15d4-48d9">Save this question.<div class="s-popover--arrow" data-webtasks-id="04951ff0-e9ae-48d8"></div></div> |
|
|
|
|
|
|
|
|
|
|
|
|
|
|
|
|
|
|
|
<a class="js-post-issue flex--item s-btn s-btn__unset c-pointer py6 mx-auto" href="/posts/181865/timeline" data-shortcut="T" data-ks-title="timeline" data-controller="s-tooltip" data-s-tooltip-placement="right" aria-label="Timeline" aria-describedby="--stacks-s-tooltip-hlxjs8mb" data-webtasks-id="faa46d18-1d0f-4615"><svg aria-hidden="true" class="mln2 mr0 svg-icon iconHistory" width="19" height="18" viewBox="0 0 19 18" data-webtasks-id="598d12a4-2c41-48d8"><path d="M3 9a8 8 0 1 1 3.73 6.77L8.2 14.3A6 6 0 1 0 5 9l3.01-.01-4 4-4-4h3L3 9Zm7-4h1.01L11 9.36l3.22 2.1-.6.93L10 10V5Z" data-webtasks-id="e3f04581-dc6b-4adc"></path></svg></a><div id="--stacks-s-tooltip-hlxjs8mb" class="s-popover s-popover__tooltip" role="tooltip" data-webtasks-id="16ee2913-5727-4c33">Show activity on this post.<div class="s-popover--arrow" data-webtasks-id="7722c7c3-5dd2-4251"></div></div> |
|
|
|
</div> |
|
|
|
</div> |
|
|
|
|
|
|
|
<div class="postcell post-layout--right" data-webtasks-id="a4c8f850-d456-4e07"> |
|
|
|
<div class="s-prose js-post-body" itemprop="text" data-webtasks-id="54ba264a-296f-4958"> |
|
|
|
<p data-webtasks-id="79a512a2-9f82-4ef4">In the town of Blackpool, England my partner and I have bought a house which was built around 1952. When we took the old wallpaper off the walls, we discovered amazing things - the lounge, dining room and bedrooms were all covered with full size, hand-painted murals.</p> |
|
<p data-webtasks-id="f079b7c9-8161-4ea9"><a href="https://i.stack.imgur.com/6fNjZ.jpg" rel="nofollow noreferrer" data-webtasks-id="02f46cbf-59ee-49f9"><img src="https://i.stack.imgur.com/6fNjZ.jpg" alt="enter image description here" data-webtasks-id="c6d0d74c-57cd-4ac9"></a></p> |
|
<p data-webtasks-id="c375ee64-138b-4299"><a href="https://i.stack.imgur.com/wLKgk.jpg" rel="nofollow noreferrer" data-webtasks-id="40a5ef4d-f389-4838"><img src="https://i.stack.imgur.com/wLKgk.jpg" alt="enter image description here" data-webtasks-id="b3893a00-fa1e-4682"></a></p> |
|
<p data-webtasks-id="171e1e8a-1869-40de"><a href="https://i.stack.imgur.com/y2CFO.jpg" rel="nofollow noreferrer" data-webtasks-id="7f339ff1-dc20-433b"><img src="https://i.stack.imgur.com/y2CFO.jpg" alt="enter image description here" data-webtasks-id="bc6ecba2-36bf-48f6"></a></p> |
|
<p data-webtasks-id="68019669-0100-4bd1"><a href="https://i.stack.imgur.com/AYmTe.jpg" rel="nofollow noreferrer" data-webtasks-id="a4ee6234-e153-4705"><img src="https://i.stack.imgur.com/AYmTe.jpg" alt="enter image description here" data-webtasks-id="9a4afa13-ef7c-429d"></a></p> |
|
<p data-webtasks-id="d4bdab03-f09e-47a4"><a href="https://i.stack.imgur.com/MiO8a.jpg" rel="nofollow noreferrer" data-webtasks-id="e5bad92d-0c08-4b91"><img src="https://i.stack.imgur.com/MiO8a.jpg" alt="enter image description here" data-webtasks-id="b990e2f6-608f-47d6"></a></p> |
|
<p data-webtasks-id="c84c6973-67a9-4a06"><a href="https://i.stack.imgur.com/ggavT.jpg" rel="nofollow noreferrer" data-webtasks-id="70a7148a-f1e7-47ae"><img src="https://i.stack.imgur.com/ggavT.jpg" alt="enter image description here" data-webtasks-id="b7b67c0c-25f1-463c"></a></p> |
|
<p data-webtasks-id="616ef4e2-64f4-4b4b"><a href="https://i.stack.imgur.com/vXlm1.jpg" rel="nofollow noreferrer" data-webtasks-id="8e8f74fd-7917-4909"><img src="https://i.stack.imgur.com/vXlm1.jpg" alt="enter image description here" data-webtasks-id="ec6dfbe1-00b7-4275"></a></p> |
|
<p data-webtasks-id="eb6ecebe-5ff5-402b">We were told that there was also a full working fountain in the dining room.</p> |
|
<p data-webtasks-id="45e8f25a-7867-4188">Somehow, I think they look like Bavarian scenes. We are told that a German woman lived there in 1950s/1960s. She operated a grand cafe in our town called 'Black Forest Cafe'. Does anyone recognise any of the locations?</p> |
|
<p data-webtasks-id="6c39729c-8910-489c">I feel these are inspired by real places, not just imagination, as the details are so intricate, such as two random palm trees next to an observatory or folly. Or perhaps the random hillside house near a viaduct/bridge, or the lakeside jetties, and the detail in the lakeside 'lighthouse'. We would love to know where the scenes are from, if anyone knows if it is a real location.</p> |
|
</div> |
|
|
|
<div class="mt24 mb12" data-webtasks-id="6b2b4fe0-8ce5-4818"> |
|
<div class="post-taglist d-flex gs4 gsy fd-column" data-webtasks-id="c9103b05-01b1-4cca"> |
|
<div class="d-flex ps-relative fw-wrap" data-webtasks-id="14e3d2c7-04bd-4f8a"> |
|
|
|
<ul class="ml0 list-ls-none js-post-tag-list-wrapper d-inline" data-webtasks-id="38d18477-ff92-4a6b"><li class="d-inline mr4 js-post-tag-list-item" data-webtasks-id="da36cf53-4533-40f8"><a href="/questions/tagged/where-on-earth" class="post-tag" title="" aria-label="show questions tagged 'where-on-earth'" rel="tag" aria-labelledby="tag-where-on-earth-tooltip-container" data-webtasks-id="c885930c-e212-44a3">where-on-earth</a></li><li class="d-inline mr4 js-post-tag-list-item" data-webtasks-id="42763373-d177-4c6b"><a href="/questions/tagged/bavaria" class="post-tag" title="" aria-label="show questions tagged 'bavaria'" rel="tag" aria-labelledby="tag-bavaria-tooltip-container" data-webtasks-id="afcd7494-1d05-4d0b">bavaria</a></li></ul> |
|
</div> |
|
</div> |
|
</div> |
|
|
|
<div class="mb0 " data-webtasks-id="e4bfc12f-fa10-4c78"> |
|
<div class="mt16 d-flex gs8 gsy fw-wrap jc-end ai-start pt4 mb16" data-webtasks-id="1857060f-f491-4f24"> |
|
<div class="flex--item mr16 fl1 w96" data-webtasks-id="5ca04dd0-023b-4abe"> |
|
|
|
|
|
|
|
<div class="js-post-menu pt2" data-post-id="181865" data-post-type-id="1" data-webtasks-id="7bd505c7-5fc2-4aff"> |
|
|
|
<div class="d-flex gs8 s-anchors s-anchors__muted fw-wrap" data-webtasks-id="3331d56d-94ed-4f35"> |
|
|
|
<div class="flex--item" data-webtasks-id="fc12e4f5-a968-4a49"> |
|
<a href="/q/181865/136878" rel="nofollow" itemprop="url" class="js-share-link js-gps-track" title="Short permalink to this question" data-gps-track="post.click({ item: 2, priv: -1, post_type: 1 })" data-controller="se-share-sheet s-popover" data-se-share-sheet-title="Share a link to this question" data-se-share-sheet-subtitle="(Includes your user id)" data-se-share-sheet-post-type="question" data-se-share-sheet-social="facebook twitter " data-se-share-sheet-location="1" data-se-share-sheet-license-url="https%3a%2f%2fcreativecommons.org%2flicenses%2fby-sa%2f4.0%2f" data-se-share-sheet-license-name="CC BY-SA 4.0" data-s-popover-placement="bottom-start" aria-controls="se-share-sheet-0" data-action=" s-popover#toggle se-share-sheet#preventNavigation s-popover:show->se-share-sheet#willShow s-popover:shown->se-share-sheet#didShow" aria-expanded="false" data-webtasks-id="f1bd1c10-0d97-4595">Share</a><div class="s-popover z-dropdown s-anchors s-anchors__default" style="width: unset; max-width: 28em;" id="se-share-sheet-0" data-webtasks-id="7fa364b9-56d7-4f53"><div class="s-popover--arrow" data-webtasks-id="78472699-11c9-4ecd"></div><div data-webtasks-id="c0e31f9e-c37d-4321"><label for="share-sheet-input-se-share-sheet-0" data-webtasks-id="757b6ab4-ef45-4415"><span class="js-title fw-bold" data-webtasks-id="e8aa8e71-ee15-482f">Share a link to this question</span> <span class="js-subtitle" data-webtasks-id="cb2d7f94-bb5d-4cc9">(Includes your user id)</span></label></div><div class="my8" data-webtasks-id="58534171-7c28-4e3d"><input type="text" id="share-sheet-input-se-share-sheet-0" class="js-input s-input wmn3 sm:wmn-initial bc-black-200 bg-white fc-dark" readonly="" data-webtasks-id="32bc9aa5-de00-4aed"></div><div class="d-flex jc-space-between ai-center mbn4" data-webtasks-id="86ba8fdb-cf8d-4590"><button class="js-copy-link-btn s-btn s-btn__link js-gps-track" data-gps-track="" data-webtasks-id="2fa43b20-9d50-4314">Copy link</button><a href="https://creativecommons.org/licenses/by-sa/4.0/" rel="license" class="js-license s-block-link w-auto" target="_blank" title="The current license for this post: CC BY-SA 4.0" data-webtasks-id="8d81c789-c6f7-426a">CC BY-SA 4.0</a><div class="js-social-container d-none" data-webtasks-id="27e1e942-8fa8-47a8"></div></div></div> |
|
</div> |
|
|
|
|
|
<div class="flex--item" data-webtasks-id="bf451cfb-0e48-4205"> |
|
<a href="/posts/181865/edit" class="js-suggest-edit-post js-gps-track" data-gps-track="post.click({ item: 6, priv: -1, post_type: 1 })" title="Revise and improve this post" data-webtasks-id="b31b3fe5-aa66-49c4">Edit</a> |
|
</div> |
|
|
|
<div class="flex--item" data-webtasks-id="45ddb52b-596a-4b3d"> |
|
<button type="button" id="btnFollowPost-181865" class="s-btn s-btn__link js-follow-post js-follow-question js-gps-track" data-gps-track="post.click({ item: 14, priv: -1, post_type: 1 })" data-controller="s-tooltip " data-s-tooltip-placement="bottom" data-s-popover-placement="bottom" aria-controls="" aria-describedby="--stacks-s-tooltip-bo5r8bz4" data-webtasks-id="650145da-e756-4a4a"> |
|
Follow |
|
</button><div id="--stacks-s-tooltip-bo5r8bz4" class="s-popover s-popover__tooltip" role="tooltip" data-webtasks-id="407a3ce5-4239-4492">Follow this question to receive notifications<div class="s-popover--arrow" data-webtasks-id="ccfe2455-2bd2-4ba0"></div></div> |
|
</div> |
|
|
|
|
|
|
|
|
|
|
|
|
|
</div> |
|
<div class="js-menu-popup-container" data-webtasks-id="009736f0-9164-46c3"></div> |
|
</div> |
|
</div> |
|
|
|
<div class="post-signature flex--item" data-webtasks-id="0677ba79-1041-4b2b"> |
|
<div class="user-info user-hover" data-webtasks-id="14d3b9b1-14cc-48fe"> |
|
<div class="user-action-time" data-webtasks-id="0a65e0b0-f7d5-419c"> |
|
<a href="/posts/181865/revisions" title="show all edits to this post" class="js-gps-track" data-gps-track="post.click({ item: 4, priv: -1, post_type: 1 })" data-webtasks-id="4f48a6bf-69d7-42c5">edited <span title="2023-06-22 07:36:04Z" class="relativetime" data-webtasks-id="f8fd91ff-5d5a-4a64">Jun 22 at 7:36</span></a> |
|
</div> |
|
<div class="user-gravatar32" data-webtasks-id="73618821-cba4-4d29"> |
|
<a href="/users/2509/gerrit" data-webtasks-id="901c47ee-1be7-44ed"><div class="gravatar-wrapper-32" data-webtasks-id="fcace875-ee2d-45b6"><img src="https://i.stack.imgur.com/vAAS0.png?s=64&g=1" alt="gerrit's user avatar" width="32" height="32" class="bar-sm" data-webtasks-id="cd6237f9-416e-4ebe"></div></a> |
|
</div> |
|
<div class="user-details" data-webtasks-id="6f1b75de-fb52-4e72"> |
|
<a href="/users/2509/gerrit" data-webtasks-id="b6143195-1d96-4aff">gerrit</a> |
|
<div class="-flair" data-webtasks-id="829fa5e9-7504-4e66"> |
|
<span class="reputation-score" title="reputation score 54,172" dir="ltr" data-webtasks-id="669a4881-f5db-44ee">54.2k</span><span title="19 gold badges" aria-hidden="true" data-webtasks-id="809d0980-5f66-499b"><span class="badge1" data-webtasks-id="3a4e8f47-de5e-4c0e"></span><span class="badgecount" data-webtasks-id="679b50a0-5048-4c6d">19</span></span><span class="v-visible-sr" data-webtasks-id="3aa4871c-c219-486f">19 gold badges</span><span title="159 silver badges" aria-hidden="true" data-webtasks-id="1a123876-c4ca-4dd9"><span class="badge2" data-webtasks-id="a81d596c-6af8-406a"></span><span class="badgecount" data-webtasks-id="3df5592c-f6ba-4bb3">159</span></span><span class="v-visible-sr" data-webtasks-id="1ded87ea-3a03-41ad">159 silver badges</span><span title="309 bronze badges" aria-hidden="true" data-webtasks-id="07774c27-115b-4638"><span class="badge3" data-webtasks-id="65ea8c7d-ef62-47d4"></span><span class="badgecount" data-webtasks-id="4e501d87-5a40-491b">309</span></span><span class="v-visible-sr" data-webtasks-id="29dc9879-f5d4-44b5">309 bronze badges</span> |
|
</div> |
|
</div> |
|
</div> |
|
</div> |
|
<div class="post-signature owner flex--item" data-webtasks-id="f12b1160-93f9-48bc"> |
|
<div class="user-info user-hover" data-webtasks-id="55380eaa-9261-403f"> |
|
<div class="user-action-time" data-webtasks-id="2ad4c302-8e03-4988"> |
|
asked <span title="2023-06-19 22:58:14Z" class="relativetime" data-webtasks-id="0d4fc43f-d713-47e8">Jun 19 at 22:58</span> |
|
</div> |
|
<div class="user-gravatar32" data-webtasks-id="08bb6565-4388-4669"> |
|
<a href="/users/136688/tony" data-webtasks-id="d1df9c6a-97a4-4bf3"><div class="gravatar-wrapper-32" data-webtasks-id="a18a867a-b46a-4eef"><img src="https://www.gravatar.com/avatar/6b0285e7517ace2c302d526c5f6239bc?s=64&d=identicon&r=PG&f=y&so-version=2" alt="Tony's user avatar" width="32" height="32" class="bar-sm" data-webtasks-id="a9bb9e5d-4153-451c"></div></a> |
|
</div> |
|
<div class="user-details" itemprop="author" itemscope="" itemtype="http://schema.org/Person" data-webtasks-id="a33b5510-8b08-4a13"> |
|
<a href="/users/136688/tony" data-webtasks-id="3a43a2f2-4d50-4ec0">Tony</a><span class="d-none" itemprop="name" data-webtasks-id="6b82cf47-c871-4b0c">Tony</span> |
|
<div class="-flair" data-webtasks-id="f90155e1-7637-4e77"> |
|
<span class="reputation-score" title="reputation score " dir="ltr" data-webtasks-id="7fd3402e-3d9e-4b32">269</span><span title="2 silver badges" aria-hidden="true" data-webtasks-id="0262195f-70ad-4bf8"><span class="badge2" data-webtasks-id="4e049d96-2f93-4c60"></span><span class="badgecount" data-webtasks-id="95e5e447-7d96-462d">2</span></span><span class="v-visible-sr" data-webtasks-id="e58cb67f-f77c-4fdb">2 silver badges</span><span title="6 bronze badges" aria-hidden="true" data-webtasks-id="9c4b3bed-287b-4fb0"><span class="badge3" data-webtasks-id="08e3ec9e-7e30-4020"></span><span class="badgecount" data-webtasks-id="9176574c-f320-450d">6</span></span><span class="v-visible-sr" data-webtasks-id="e2eca04e-3ddd-435b">6 bronze badges</span> |
|
</div> |
|
</div> |
|
</div> |
|
|
|
|
|
</div> |
|
</div> |
|
</div> |
|
|
|
</div> |
|
|
|
|
|
|
|
|
|
<span class="d-none" itemprop="commentCount" data-webtasks-id="79cd1891-5300-421e">0</span> |
|
<div class="post-layout--right js-post-comments-component" data-webtasks-id="ad60e2d2-020c-4b7f"> |
|
<div id="comments-181865" class="comments js-comments-container bt bc-black-075 mt12 dno" data-post-id="181865" data-min-length="15" data-webtasks-id="7b6801ce-1e3c-4205"> |
|
<ul class="comments-list js-comments-list" data-remaining-comments-count="0" data-canpost="false" data-cansee="true" data-comments-unavailable="false" data-addlink-disabled="true" data-webtasks-id="5c36b5e3-1940-4615"> |
|
|
|
</ul> |
|
</div> |
|
|
|
<div id="comments-link-181865" data-rep="50" data-reg="true" data-webtasks-id="d818a4e9-4981-414f"> |
|
<a class="js-add-link comments-link disabled-link" title="Use comments to ask for more information or suggest improvements. Avoid answering questions in comments." href="#" role="button" data-webtasks-id="9c295197-17dd-465a">Add a comment</a> |
|
<span class="js-link-separator dno" data-webtasks-id="dbcc9250-4328-4e04"> | </span> |
|
<a class="js-show-link comments-link dno" title="Expand to show all comments on this post" href="#" onclick="" role="button" data-webtasks-id="8374a28e-d87c-4b8a"></a> |
|
</div> |
|
</div> |
|
</div> |
|
|
|
</div> |
|
|
|
|
|
|
|
|
|
|
|
<div id="answers" data-webtasks-id="deb8a970-efc5-4d23"> |
|
<a name="tab-top" data-webtasks-id="9b67de71-6e14-47eb"></a> |
|
<div id="answers-header" data-webtasks-id="a5ff7cd9-6961-407a"> |
|
<div class="answers-subheader d-flex ai-center mb8" data-webtasks-id="4faf619e-f245-43bd"> |
|
<div class="flex--item fl1" data-webtasks-id="83e40193-527f-41c5"> |
|
<h2 class="mb0" data-answercount="3" data-webtasks-id="e4e16a97-a7f9-4126"> |
|
3 Answers |
|
<span style="display:none;" itemprop="answerCount" data-webtasks-id="30cc12d2-ba4f-45f7">3</span> |
|
</h2> |
|
</div> |
|
<div class="flex--item" data-webtasks-id="1c166ea6-4cb1-4fc0"> |
|
|
|
|
|
<div class="d-flex g4 gsx ai-center sm:fd-column sm:ai-start" data-webtasks-id="07af1b45-b0b8-46c3"> |
|
<div class="d-flex fd-column ai-end sm:ai-start" data-webtasks-id="d7ea672f-2fea-4bca"> |
|
<label class="flex--item fs-caption" for="answer-sort-dropdown-select-menu" data-webtasks-id="754adb0d-bf7c-4b7c"> |
|
Sorted by: |
|
</label> |
|
<a class="js-sort-preference-change s-link flex--item fs-fine d-none" data-value="ScoreDesc" href="/questions/181865/are-these-murals-of-a-real-location-perhaps-bavaria?answertab=scoredesc#tab-top" data-webtasks-id="9d6b5af0-bdc2-4c48"> |
|
Reset to default |
|
</a> |
|
</div> |
|
<div class="flex--item s-select" data-webtasks-id="9fecccbf-4aea-486d"> |
|
<select id="answer-sort-dropdown-select-menu" data-webtasks-id="a75963ee-747a-4735"> |
|
<option value="scoredesc" selected="selected" data-webtasks-id="686dbd2f-233e-4b3a"> |
|
Highest score (default) |
|
</option> |
|
<option value="modifieddesc" data-webtasks-id="7e524729-9610-4baf"> |
|
Date modified (newest first) |
|
</option> |
|
<option value="createdasc" data-webtasks-id="68b6e6a8-f23e-4c40"> |
|
Date created (oldest first) |
|
</option> |
|
</select> |
|
</div> |
|
</div> |
|
|
|
|
|
</div> |
|
</div> |
|
</div> |
|
|
|
|
|
|
|
<a name="181868" data-webtasks-id="5551043c-11a1-4a6b"></a> |
|
<div id="answer-181868" class="answer js-answer" data-answerid="181868" data-parentid="181865" data-score="32" data-position-on-page="1" data-highest-scored="1" data-question-has-accepted-highest-score="0" itemprop="suggestedAnswer" itemscope="" itemtype="https://schema.org/Answer" data-webtasks-id="bd6a66de-b14a-43c1"> |
|
<div class="post-layout" data-webtasks-id="24598beb-42b5-4d30"> |
|
<div class="votecell post-layout--left" data-webtasks-id="05762aa2-d10d-4750"> |
|
<div class="js-voting-container d-flex jc-center fd-column ai-stretch gs4 fc-black-200" data-post-id="181868" data-webtasks-id="12ae40cb-96ea-43c1"> |
|
<button class="js-vote-up-btn flex--item s-btn ba bar-pill c-pointer as-center bc-black-100 fc-black-700 h:bg-theme-primary-100" data-controller="s-tooltip" data-s-tooltip-placement="right" aria-pressed="false" aria-label="Up vote" data-selected-classes="fc-theme-primary bc-theme-primary bg-theme-primary-025" data-unselected-classes="bc-black-100 fc-black-700 h:bg-theme-primary-100" aria-describedby="--stacks-s-tooltip-zxokdi0p" data-webtasks-id="d559cf54-515a-49e1"> |
|
<svg aria-hidden="true" class="svg-icon iconArrowUp" width="18" height="18" viewBox="0 0 18 18" data-webtasks-id="778bbb0d-21ad-4341"><path d="M1 12h16L9 4l-8 8Z" data-webtasks-id="67dccb7e-1f8c-4b49"></path></svg> |
|
</button><div id="--stacks-s-tooltip-zxokdi0p" class="s-popover s-popover__tooltip" role="tooltip" data-webtasks-id="c079b2df-d863-40e4">This answer is useful<div class="s-popover--arrow" data-webtasks-id="960b009e-b156-4f34"></div></div> |
|
<div class="js-vote-count flex--item d-flex fd-column ai-center fc-theme-body-font fw-bold fs-subheading py4" itemprop="upvoteCount" data-value="32" data-webtasks-id="399286d9-e558-4ba3"> |
|
32 |
|
</div> |
|
<button class="js-vote-down-btn flex--item mb8 s-btn ba bar-pill c-pointer as-center bc-black-100 fc-black-700 h:bg-theme-primary-100" data-controller="s-tooltip" data-s-tooltip-placement="right" aria-pressed="false" aria-label="Down vote" data-selected-classes="fc-theme-primary bc-theme-primary bg-theme-primary-025" data-unselected-classes="bc-black-100 fc-black-700 h:bg-theme-primary-100" aria-describedby="--stacks-s-tooltip-lf6b38xp" data-webtasks-id="3b47e6f5-a1c6-49a3"> |
|
<svg aria-hidden="true" class="svg-icon iconArrowDown" width="18" height="18" viewBox="0 0 18 18" data-webtasks-id="6666c555-8b29-4903"><path d="M1 6h16l-8 8-8-8Z" data-webtasks-id="daad7b55-a861-4ce8"></path></svg> |
|
</button><div id="--stacks-s-tooltip-lf6b38xp" class="s-popover s-popover__tooltip" role="tooltip" data-webtasks-id="df7a3784-bd84-45e9">This answer is not useful<div class="s-popover--arrow" data-webtasks-id="ce9e5f9a-06c5-4525"></div></div> |
|
|
|
|
|
|
|
<button class="js-saves-btn s-btn s-btn__unset c-pointer py4" type="button" id="saves-btn-181868" data-controller="s-tooltip" data-s-tooltip-placement="right" data-s-popover-placement="" aria-pressed="false" data-post-id="181868" data-post-type-id="2" data-user-privilege-for-post-click="-1" aria-controls="" data-s-popover-auto-show="false" aria-describedby="--stacks-s-tooltip-apg21v3e" data-webtasks-id="f320136b-ee1e-4fda"> |
|
<svg aria-hidden="true" class="fc-theme-primary-500 js-saves-btn-selected d-none svg-icon iconBookmark" width="18" height="18" viewBox="0 0 18 18" data-webtasks-id="29ff9369-b6e3-4708"><path d="M3 17V3c0-1.1.9-2 2-2h8a2 2 0 0 1 2 2v14l-6-4-6 4Z" data-webtasks-id="0a775621-5d04-456d"></path></svg> |
|
<svg aria-hidden="true" class="js-saves-btn-unselected svg-icon iconBookmarkAlt" width="18" height="18" viewBox="0 0 18 18" data-webtasks-id="851c3e6c-332a-4ece"><path d="m9 10.6 4 2.66V3H5v10.26l4-2.66ZM3 17V3c0-1.1.9-2 2-2h8a2 2 0 0 1 2 2v14l-6-4-6 4Z" data-webtasks-id="c21ac1fe-843d-4c00"></path></svg> |
|
</button><div id="--stacks-s-tooltip-apg21v3e" class="s-popover s-popover__tooltip" role="tooltip" data-webtasks-id="3c45c2fe-369d-4f89">Save this answer.<div class="s-popover--arrow" data-webtasks-id="43f9d80b-dd8c-46e4"></div></div> |
|
|
|
|
|
|
|
|
|
|
|
|
|
|
|
<div class="js-accepted-answer-indicator flex--item fc-green-700 py6 mtn8 d-none" data-s-tooltip-placement="right" title="Loading when this answer was accepted…" tabindex="0" role="note" aria-label="Accepted" data-webtasks-id="2ce81ef8-414c-42cf"> |
|
<div class="ta-center" data-webtasks-id="f432a015-c794-4978"> |
|
<svg aria-hidden="true" class="svg-icon iconCheckmarkLg" width="36" height="36" viewBox="0 0 36 36" data-webtasks-id="423db7ae-f02f-478a"><path d="m6 14 8 8L30 6v8L14 30l-8-8v-8Z" data-webtasks-id="c4c3f7e3-a273-449a"></path></svg> |
|
</div> |
|
</div> |
|
|
|
|
|
<a class="js-post-issue flex--item s-btn s-btn__unset c-pointer py6 mx-auto" href="/posts/181868/timeline" data-shortcut="T" data-ks-title="timeline" data-controller="s-tooltip" data-s-tooltip-placement="right" aria-label="Timeline" aria-describedby="--stacks-s-tooltip-ir4qx811" data-webtasks-id="cb5f331e-d14b-4608"><svg aria-hidden="true" class="mln2 mr0 svg-icon iconHistory" width="19" height="18" viewBox="0 0 19 18" data-webtasks-id="548bf7ea-bac6-4c56"><path d="M3 9a8 8 0 1 1 3.73 6.77L8.2 14.3A6 6 0 1 0 5 9l3.01-.01-4 4-4-4h3L3 9Zm7-4h1.01L11 9.36l3.22 2.1-.6.93L10 10V5Z" data-webtasks-id="46556014-e59b-40ce"></path></svg></a><div id="--stacks-s-tooltip-ir4qx811" class="s-popover s-popover__tooltip" role="tooltip" data-webtasks-id="50f251a7-aa44-4e9e">Show activity on this post.<div class="s-popover--arrow" data-webtasks-id="3275ebb3-83e3-46fc"></div></div> |
|
|
|
</div> |
|
|
|
</div> |
|
|
|
|
|
|
|
<div class="answercell post-layout--right" data-webtasks-id="cb92d62e-90ba-4b1e"> |
|
|
|
<div class="s-prose js-post-body" itemprop="text" data-webtasks-id="fec29c58-1b94-4e18"> |
|
<p data-webtasks-id="464fc224-ca24-4a6a">I'm putting in an answer, as I've got too much for a comment.</p> |
|
<p data-webtasks-id="a762ab82-b873-4efc">This doesn't strike me as a 'Bavarian scene'. The peaks are a bit too pointy, the palm tree (I only see one) is not quite Bavarian, and the square sail on the sailboat feels more eastern Mediterranean to me.</p> |
|
<p data-webtasks-id="f4fc6b75-6518-40ba">I found <a href="https://www.belgian-art-gallery.be/en/belgian-artists/jules-pierre-van-biesbroeck-jr-le-pont-st-louis" rel="noreferrer" data-webtasks-id="734d2987-3db6-4115">the following painting</a>, which includes a slightly tilted bridge, not dissimilar from the one in your painting. The website is slow. It's "Le Pont St Louis Nice" by <a href="https://en.wikipedia.org/wiki/Jules_Pierre_van_Biesbroeck" rel="noreferrer" data-webtasks-id="533c0cbf-7bf1-40e8">Jules Pierre van Biesbroeck (Jr.)</a>.</p> |
|
<p data-webtasks-id="ce866b16-61b9-4110">A <a href="https://duckduckgo.com/?q=le%20pont%20st.%20louis%20nice&t=osx&iax=images&ia=images&iai=https%3A%2F%2Fi.pinimg.com%2F736x%2F85%2F6e%2Fc0%2F856ec0ceff3ccee62ab4c701e6845943--in-the-middle-anne.jpg" rel="noreferrer" data-webtasks-id="1f5ed92e-5773-452d">search for that bridge</a> returns modern Nice, but also a few old images, like <a href="https://external-content.duckduckgo.com/iu/?u=https%3A%2F%2Ftse1.mm.bing.net%2Fth%3Fid%3DOIP.U6r1an7ZPaAxND7QqX3xMgHaKn%26pid%3DApi&f=1&ipt=b2d3fe36e7f3900dd5da6175eac2e05a8822e0fb04248ae0f777fb696155cc74&ipo=images" rel="noreferrer" data-webtasks-id="07ca5714-a8c7-46df">this one</a>, which seem like a spitting image of the bridge. Notice, also, the two small houses on either side of the bridge itself, which seem to be replicated in your painting.</p> |
|
<p data-webtasks-id="9504ccc3-1fed-442a">However, <a href="https://external-content.duckduckgo.com/iu/?u=https%3A%2F%2Ftse3.mm.bing.net%2Fth%3Fid%3DOIP.djB0hafuc5s1qb1kXqK85gHaEw%26pid%3DApi&f=1&ipt=47bbfd5a34d40577ff9386a85645008d743b7d510a8f842d3158bf882a74471b&ipo=images" rel="noreferrer" data-webtasks-id="85575e85-5988-46c7">another view of the same bridge</a> shows that the wider area around the bridge is quite different from your painting, which would rule out this particular setting as the real-life version of your painting.</p> |
|
<p data-webtasks-id="b5556009-0f9d-4a60">The bridge in the images is in the town of Menton, in southern France. And some sleuthing around Menton results in the discovery of <a href="https://external-content.duckduckgo.com/iu/?u=https%3A%2F%2Ftse1.mm.bing.net%2Fth%3Fid%3DOIP.iHxUpPnJSRIy83MzhesYSQHaE6%26pid%3DApi&f=1&ipt=e16fdf57387c76dd2593fa97f0af16487a4f0e93c00a64e7864363eec27cd514&ipo=images" rel="noreferrer" data-webtasks-id="26f1504e-8c4f-41db">a picture of the bridge, taken from the sea</a> (included below).</p> |
|
<p data-webtasks-id="99eec8dc-e9d0-471d">This is still not quite the same image as your painting, but, notice the slant of the mountain to the left of the bridge, and compare that with the actual slant in the image. This, and the scene in general, is close enough, for me, to consider that your painting could be an artist's impression of the "Pont St. Louis" in Menton, France.</p> |
|
<p data-webtasks-id="bf51e705-4631-4bea">For one, <a href="https://imgc.allpostersimages.com/img/posters/menton-tropical-france-palm-tree_u-L-F8TEWL0.jpg?p=0" rel="noreferrer" data-webtasks-id="66bdc60f-992c-4998">Menton has palm trees</a> :)</p> |
|
<p data-webtasks-id="35bc53e1-264e-4a36"><a href="https://i.stack.imgur.com/ZKCUn.jpg" rel="noreferrer" data-webtasks-id="2903d199-9814-42c1"><img src="https://i.stack.imgur.com/ZKCUn.jpg" alt="enter image description here" data-webtasks-id="77082a7e-1197-444e"></a></p> |
|
<p data-webtasks-id="aba93674-6a14-4365">Edit (after OP added a second image): The second image feels quite generic to me, while the right side of the image is hard to make out. But, the church on the left and the setting have a passing resemblance to <a href="https://wall.alphacoders.com/big.php?i=1094879&lang=Italian" rel="noreferrer" data-webtasks-id="11c1ba4c-ba0d-4c99">The Santuario della Madonna del Sasso</a>, in Locarno.</p> |
|
</div> |
|
<div class="mt24" data-webtasks-id="db3c0aa9-ffc7-4ac3"> |
|
<div class="d-flex fw-wrap ai-start jc-end gs8 gsy" data-webtasks-id="22a3451e-0017-4f6e"> |
|
<time itemprop="dateCreated" datetime="2023-06-20T00:19:47" data-webtasks-id="9e22381c-5015-462f"></time> |
|
<div class="flex--item mr16" style="flex: 1 1 100px;" data-webtasks-id="5b25ce41-315b-4700"> |
|
|
|
|
|
|
|
<div class="js-post-menu pt2" data-post-id="181868" data-post-type-id="2" data-webtasks-id="146fa23f-6a3a-465f"> |
|
|
|
<div class="d-flex gs8 s-anchors s-anchors__muted fw-wrap" data-webtasks-id="d7e0a1fd-0156-482e"> |
|
|
|
<div class="flex--item" data-webtasks-id="37ad1c7b-8074-446e"> |
|
<a href="/a/181868/136878" rel="nofollow" itemprop="url" class="js-share-link js-gps-track" title="Short permalink to this answer" data-gps-track="post.click({ item: 2, priv: -1, post_type: 2 })" data-controller="se-share-sheet s-popover" data-se-share-sheet-title="Share a link to this answer" data-se-share-sheet-subtitle="(Includes your user id)" data-se-share-sheet-post-type="answer" data-se-share-sheet-social="facebook twitter " data-se-share-sheet-location="2" data-se-share-sheet-license-url="https%3a%2f%2fcreativecommons.org%2flicenses%2fby-sa%2f4.0%2f" data-se-share-sheet-license-name="CC BY-SA 4.0" data-s-popover-placement="bottom-start" aria-controls="se-share-sheet-1" data-action=" s-popover#toggle se-share-sheet#preventNavigation s-popover:show->se-share-sheet#willShow s-popover:shown->se-share-sheet#didShow" aria-expanded="false" data-webtasks-id="8af0924e-3823-4655">Share</a><div class="s-popover z-dropdown s-anchors s-anchors__default" style="width: unset; max-width: 28em;" id="se-share-sheet-1" data-webtasks-id="8fb3fabd-07e4-4e4c"><div class="s-popover--arrow" data-webtasks-id="234548c4-b94e-4539"></div><div data-webtasks-id="4d0e5a27-d483-47a0"><label for="share-sheet-input-se-share-sheet-1" data-webtasks-id="75c1c2ae-6c1b-40c3"><span class="js-title fw-bold" data-webtasks-id="e94908fe-d017-4127">Share a link to this answer</span> <span class="js-subtitle" data-webtasks-id="1e0c3592-234c-41a4">(Includes your user id)</span></label></div><div class="my8" data-webtasks-id="c23f67b1-c02f-4f00"><input type="text" id="share-sheet-input-se-share-sheet-1" class="js-input s-input wmn3 sm:wmn-initial bc-black-200 bg-white fc-dark" readonly="" data-webtasks-id="aadfcd43-b24c-4808"></div><div class="d-flex jc-space-between ai-center mbn4" data-webtasks-id="1390caa5-5be9-48e7"><button class="js-copy-link-btn s-btn s-btn__link js-gps-track" data-gps-track="" data-webtasks-id="7038934f-20ed-45ac">Copy link</button><a href="https://creativecommons.org/licenses/by-sa/4.0/" rel="license" class="js-license s-block-link w-auto" target="_blank" title="The current license for this post: CC BY-SA 4.0" data-webtasks-id="79954ed6-c99a-4210">CC BY-SA 4.0</a><div class="js-social-container d-none" data-webtasks-id="d3723604-9e95-472c"></div></div></div> |
|
</div> |
|
|
|
|
|
<div class="flex--item" data-webtasks-id="884bff9b-cd6a-4ff1"> |
|
<a href="/posts/181868/edit" class="js-suggest-edit-post js-gps-track" data-gps-track="post.click({ item: 6, priv: -1, post_type: 2 })" title="Revise and improve this post" data-webtasks-id="6ab480c8-f699-4be3">Edit</a> |
|
</div> |
|
|
|
<div class="flex--item" data-webtasks-id="03eba246-e5b0-472d"> |
|
<button type="button" id="btnFollowPost-181868" class="s-btn s-btn__link js-follow-post js-follow-answer js-gps-track" data-gps-track="post.click({ item: 14, priv: -1, post_type: 2 })" data-controller="s-tooltip " data-s-tooltip-placement="bottom" data-s-popover-placement="bottom" aria-controls="" aria-describedby="--stacks-s-tooltip-xdf5s2ew" data-webtasks-id="3f15e385-eddf-40df"> |
|
Follow |
|
</button><div id="--stacks-s-tooltip-xdf5s2ew" class="s-popover s-popover__tooltip" role="tooltip" data-webtasks-id="a841880e-c9bd-4801">Follow this answer to receive notifications<div class="s-popover--arrow" data-webtasks-id="3de5ffbd-6226-49a3"></div></div> |
|
</div> |
|
|
|
|
|
|
|
|
|
|
|
|
|
</div> |
|
<div class="js-menu-popup-container" data-webtasks-id="8a96d420-0edf-4636"></div> |
|
</div> |
|
</div> |
|
<div class="post-signature flex--item fl0" data-webtasks-id="0ae42dc1-791a-46bf"> |
|
<div class="user-info " data-webtasks-id="29f8f167-9ef5-42e0"> |
|
<div class="user-action-time" data-webtasks-id="94631d6d-4e09-45fb"> |
|
<a href="/posts/181868/revisions" title="show all edits to this post" class="js-gps-track" data-gps-track="post.click({ item: 4, priv: -1, post_type: 2 })" data-webtasks-id="feabf04d-7def-472d">edited <span title="2023-06-21 11:49:35Z" class="relativetime" data-webtasks-id="3a1c72f4-69ba-4d39">Jun 21 at 11:49</span></a> |
|
</div> |
|
<div class="user-gravatar32" data-webtasks-id="45aac50e-c7e7-419d"> |
|
|
|
</div> |
|
<div class="user-details" data-webtasks-id="5f7b8466-9717-405b"> |
|
|
|
<div class="-flair" data-webtasks-id="41b6eb62-c1a4-4376"> |
|
|
|
</div> |
|
</div> |
|
</div> |
|
</div> |
|
|
|
|
|
<div class="post-signature flex--item fl0" data-webtasks-id="af7ef29d-48ac-4e2c"> |
|
<div class="user-info user-hover" data-webtasks-id="c59c03e5-8ba8-462c"> |
|
<div class="user-action-time" data-webtasks-id="f2e33881-696b-4fb9"> |
|
answered <span title="2023-06-20 00:19:47Z" class="relativetime" data-webtasks-id="185ffa4d-9091-415a">Jun 20 at 0:19</span> |
|
</div> |
|
<div class="user-gravatar32" data-webtasks-id="40906694-7472-4427"> |
|
<a href="/users/2271/mastababa" data-webtasks-id="33ec13ea-0bf3-4c7d"><div class="gravatar-wrapper-32" data-webtasks-id="993ef2f5-e14b-41e1"><img src="https://www.gravatar.com/avatar/f51c78e32ba94d51b371bf8b829aac2b?s=64&d=identicon&r=PG" alt="MastaBaba's user avatar" width="32" height="32" class="bar-sm" data-webtasks-id="9742edc6-d525-422b"></div></a> |
|
</div> |
|
<div class="user-details" itemprop="author" itemscope="" itemtype="http://schema.org/Person" data-webtasks-id="553440fd-b01f-45fa"> |
|
<a href="/users/2271/mastababa" data-webtasks-id="45af5a5f-7d82-447c">MastaBaba</a><span class="d-none" itemprop="name" data-webtasks-id="3520555d-11ee-45eb">MastaBaba</span> |
|
<div class="-flair" data-webtasks-id="be5119c5-d1ae-4fed"> |
|
<span class="reputation-score" title="reputation score 23,861" dir="ltr" data-webtasks-id="4c6adf0a-96ae-4135">23.9k</span><span title="1 gold badge" aria-hidden="true" data-webtasks-id="f387f02f-e191-4f2e"><span class="badge1" data-webtasks-id="8592e4cc-e8d7-4111"></span><span class="badgecount" data-webtasks-id="a06f8208-e1ba-46c1">1</span></span><span class="v-visible-sr" data-webtasks-id="3e9c13d3-9f2b-4ba9">1 gold badge</span><span title="61 silver badges" aria-hidden="true" data-webtasks-id="968bcfb5-0c24-4ca5"><span class="badge2" data-webtasks-id="62b7ac73-0575-4727"></span><span class="badgecount" data-webtasks-id="a3d486a2-3033-4324">61</span></span><span class="v-visible-sr" data-webtasks-id="8ff5655d-2f77-4788">61 silver badges</span><span title="86 bronze badges" aria-hidden="true" data-webtasks-id="7512d163-c6dc-4220"><span class="badge3" data-webtasks-id="a6da9952-0085-4abb"></span><span class="badgecount" data-webtasks-id="6cecc138-3c96-4e7f">86</span></span><span class="v-visible-sr" data-webtasks-id="aa7b64ef-8039-4c80">86 bronze badges</span> |
|
</div> |
|
</div> |
|
</div> |
|
|
|
|
|
</div> |
|
</div> |
|
|
|
|
|
</div> |
|
|
|
</div> |
|
|
|
|
|
|
|
|
|
<span class="d-none" itemprop="commentCount" data-webtasks-id="ffa28dd1-5f04-4ee0">3</span> |
|
<div class="post-layout--right js-post-comments-component" data-webtasks-id="ee11bc31-0e2a-4110"> |
|
<div id="comments-181868" class="comments js-comments-container bt bc-black-075 mt12 " data-post-id="181868" data-min-length="15" data-webtasks-id="0a95db56-a0e6-4212"> |
|
<ul class="comments-list js-comments-list" data-remaining-comments-count="0" data-canpost="false" data-cansee="true" data-comments-unavailable="false" data-addlink-disabled="true" data-webtasks-id="ca550efa-8cdf-468e"> |
|
|
|
<li id="comment-462814" class="comment js-comment " data-comment-id="462814" data-comment-owner-id="95267" data-comment-score="3" data-webtasks-id="a19ba576-18ef-414d"> |
|
<div class="js-comment-actions comment-actions" data-webtasks-id="d1580f26-93a8-4e3e"> |
|
<div class="comment-score js-comment-score js-comment-edit-hide" data-webtasks-id="cdbc4a28-7001-4b92"> |
|
<span title="number of 'useful comment' votes received" class="cool" data-webtasks-id="3a9f5c21-94fe-4fcf">3</span> |
|
</div> |
|
</div> |
|
<div class="comment-text js-comment-text-and-form" data-webtasks-id="e0ccb3d5-b7f7-4bf1"> |
|
<div class="comment-body js-comment-edit-hide" data-webtasks-id="eeb26994-7ba7-410d"> |
|
|
|
<span class="comment-copy" data-webtasks-id="b22ef73d-77a9-48b8">Also the houses are not typical of Bavaria (nor the Black Forest in Baden-Württemberg), but look more Mediterranean, so Italy/France is more likely.</span> |
|
|
|
<div class="d-inline-flex ai-center" data-webtasks-id="09571da6-776f-49f5"> |
|
– <a href="/users/95267/mark-johnson" title="22,559 reputation" class="comment-user" data-webtasks-id="ebf6c9eb-db52-462e">Mark Johnson</a> |
|
</div> |
|
<span class="comment-date" dir="ltr" data-webtasks-id="89bc84d9-a03d-42d8"><a class="comment-link" href="#comment462814_181868" aria-label="Link to comment" data-webtasks-id="9a254c6d-bac7-450c"><span title="2023-06-20 03:30:18Z, License: CC BY-SA 4.0" class="relativetime-clean" data-webtasks-id="38b89361-270e-4f98">Jun 20 at 3:30</span></a></span> |
|
<span title="this comment was edited 1 time" data-webtasks-id="e200f051-654d-43f8"> |
|
<svg aria-hidden="true" class="va-text-bottom o50 svg-icon iconPencilSm" width="14" height="14" viewBox="0 0 14 14" data-webtasks-id="8c79665b-6ed0-405e"><path d="m11.1 1.71 1.13 1.12c.2.2.2.51 0 .71L11.1 4.7 9.21 2.86l1.17-1.15c.2-.2.51-.2.71 0ZM2 10.12l6.37-6.43 1.88 1.88L3.88 12H2v-1.88Z" data-webtasks-id="e7cc99bc-deca-423d"></path></svg> |
|
</span> |
|
</div> |
|
</div> |
|
</li> |
|
<li id="comment-462868" class="comment js-comment " data-comment-id="462868" data-comment-owner-id="8261" data-comment-score="0" data-webtasks-id="8c4080be-feee-48b6"> |
|
<div class="js-comment-actions comment-actions" data-webtasks-id="9dfa3e0b-6d07-4ef8"> |
|
<div class="comment-score js-comment-score js-comment-edit-hide" data-webtasks-id="4e669a51-8f54-48f8"> |
|
</div> |
|
</div> |
|
<div class="comment-text js-comment-text-and-form" data-webtasks-id="a649b28c-e818-4a85"> |
|
<div class="comment-body js-comment-edit-hide" data-webtasks-id="ff3518c3-8810-4b5c"> |
|
|
|
<span class="comment-copy" data-webtasks-id="2a3aed06-69c6-4a7f">The bridge in the painting is the border between France and Italy, so half the painting is of Italy.</span> |
|
|
|
<div class="d-inline-flex ai-center" data-webtasks-id="70e5a23c-3080-4b36"> |
|
– <a href="/users/8261/jack-aidley" title="2,898 reputation" class="comment-user" data-webtasks-id="8da9e998-1214-432d">Jack Aidley</a> |
|
</div> |
|
<span class="comment-date" dir="ltr" data-webtasks-id="7aea0e9b-4288-4235"><a class="comment-link" href="#comment462868_181868" aria-label="Link to comment" data-webtasks-id="cc06026b-51ea-4336"><span title="2023-06-20 14:02:01Z, License: CC BY-SA 4.0" class="relativetime-clean" data-webtasks-id="37c7d738-fdd5-4abb">Jun 20 at 14:02</span></a></span> |
|
</div> |
|
</div> |
|
</li> |
|
<li id="comment-462876" class="comment js-comment " data-comment-id="462876" data-comment-owner-id="115444" data-comment-score="1" data-webtasks-id="54e25938-5e79-4cf9"> |
|
<div class="js-comment-actions comment-actions" data-webtasks-id="cee01b80-8e21-4c50"> |
|
<div class="comment-score js-comment-score js-comment-edit-hide" data-webtasks-id="535c4ab0-db3f-47ae"> |
|
<span title="number of 'useful comment' votes received" class="cool" data-webtasks-id="19acfa36-0bc3-41f2">1</span> |
|
</div> |
|
</div> |
|
<div class="comment-text js-comment-text-and-form" data-webtasks-id="9ed5dfec-0c84-4252"> |
|
<div class="comment-body js-comment-edit-hide" data-webtasks-id="0a9d8ff5-c38f-44c7"> |
|
|
|
<span class="comment-copy" data-webtasks-id="f24c1b24-64f2-4479">Notwithstanding your excellent answer, the buildings immediately reminded me of those shown on the cover of a Smetana CD representing Bohemia (not Bavaria). The bridge, wedged between living rock rather than with engineered approaches, immediately reminded me of the Mostar bridge that was rebuilt.. or possibly the (old) Teufelsbrücke on the St Gotthard Pass. And the boat (which /surely/ isn't a Thames Barge) looks more like something you'd see on a river or lake than on the Mediterranean. In short, I think we need to be cautious lest it turn out to be a fanciful (albeit fascinating) composite.</span> |
|
|
|
<div class="d-inline-flex ai-center" data-webtasks-id="96e781f1-03cd-4196"> |
|
– <a href="/users/115444/mark-morgan-lloyd" title="101 reputation" class="comment-user" data-webtasks-id="c6817831-434a-41ce">Mark Morgan Lloyd</a> |
|
</div> |
|
<span class="comment-date" dir="ltr" data-webtasks-id="3e3ee154-c13e-4a9c"><a class="comment-link" href="#comment462876_181868" aria-label="Link to comment" data-webtasks-id="fbfc169b-10d3-491d"><span title="2023-06-20 14:59:21Z, License: CC BY-SA 4.0" class="relativetime-clean" data-webtasks-id="7cdb3716-f4a9-4a89">Jun 20 at 14:59</span></a></span> |
|
<span title="this comment was edited 1 time" data-webtasks-id="b1495e92-fa1d-4911"> |
|
<svg aria-hidden="true" class="va-text-bottom o50 svg-icon iconPencilSm" width="14" height="14" viewBox="0 0 14 14" data-webtasks-id="e7addf78-79d4-4db9"><path d="m11.1 1.71 1.13 1.12c.2.2.2.51 0 .71L11.1 4.7 9.21 2.86l1.17-1.15c.2-.2.51-.2.71 0ZM2 10.12l6.37-6.43 1.88 1.88L3.88 12H2v-1.88Z" data-webtasks-id="61962c6d-a848-4cf9"></path></svg> |
|
</span> |
|
</div> |
|
</div> |
|
</li> |
|
|
|
</ul> |
|
</div> |
|
|
|
<div id="comments-link-181868" data-rep="50" data-reg="true" data-webtasks-id="09d56c2d-a68f-4a16"> |
|
<a class="js-add-link comments-link disabled-link" title="Use comments to ask for more information or suggest improvements. Avoid comments like “+1” or “thanks”." href="#" role="button" data-webtasks-id="9b1c085b-28af-4415">Add a comment</a> |
|
<span class="js-link-separator dno" data-webtasks-id="b1d16fe2-734e-491c"> | </span> |
|
<a class="js-show-link comments-link dno" title="Expand to show all comments on this post" href="#" onclick="" role="button" data-webtasks-id="dfb9126d-2cb1-4aba"></a> |
|
</div> |
|
</div> |
|
</div> |
|
</div> |
|
|
|
<div class="js-zone-container zone-container-main" data-webtasks-id="3840de46-ddf4-49eb"> |
|
<div id="dfp-tlb" class="everyonelovesstackoverflow everyoneloves__top-leaderboard everyoneloves__leaderboard" data-webtasks-id="6f6159cb-5d7c-4e82"></div> |
|
<div class="js-report-ad-button-container " style="width: 728px" data-webtasks-id="276da904-031b-43b5"></div> |
|
</div> |
|
|
|
<a name="181892" data-webtasks-id="0cc45273-9f73-4d6e"></a> |
|
<div id="answer-181892" class="answer js-answer" data-answerid="181892" data-parentid="181865" data-score="14" data-position-on-page="2" data-highest-scored="0" data-question-has-accepted-highest-score="0" itemprop="suggestedAnswer" itemscope="" itemtype="https://schema.org/Answer" data-webtasks-id="aab9bdc7-3741-49cf"> |
|
<div class="post-layout" data-webtasks-id="388ded2b-7d64-4674"> |
|
<div class="votecell post-layout--left" data-webtasks-id="d433cd5d-79c9-4960"> |
|
<div class="js-voting-container d-flex jc-center fd-column ai-stretch gs4 fc-black-200" data-post-id="181892" data-webtasks-id="2b5be4b9-cc6f-4b39"> |
|
<button class="js-vote-up-btn flex--item s-btn ba bar-pill c-pointer as-center bc-black-100 fc-black-700 h:bg-theme-primary-100" data-controller="s-tooltip" data-s-tooltip-placement="right" aria-pressed="false" aria-label="Up vote" data-selected-classes="fc-theme-primary bc-theme-primary bg-theme-primary-025" data-unselected-classes="bc-black-100 fc-black-700 h:bg-theme-primary-100" aria-describedby="--stacks-s-tooltip-6siihd6d" data-webtasks-id="bffd0392-1c8d-498c"> |
|
<svg aria-hidden="true" class="svg-icon iconArrowUp" width="18" height="18" viewBox="0 0 18 18" data-webtasks-id="9d3687e5-b28c-422c"><path d="M1 12h16L9 4l-8 8Z" data-webtasks-id="a03823e9-1e0d-4695"></path></svg> |
|
</button><div id="--stacks-s-tooltip-6siihd6d" class="s-popover s-popover__tooltip" role="tooltip" data-webtasks-id="362f8b68-a1de-4673">This answer is useful<div class="s-popover--arrow" data-webtasks-id="840ad8fd-b976-428c"></div></div> |
|
<div class="js-vote-count flex--item d-flex fd-column ai-center fc-theme-body-font fw-bold fs-subheading py4" itemprop="upvoteCount" data-value="14" data-webtasks-id="2e0666e3-498a-4763"> |
|
14 |
|
</div> |
|
<button class="js-vote-down-btn flex--item mb8 s-btn ba bar-pill c-pointer as-center bc-black-100 fc-black-700 h:bg-theme-primary-100" data-controller="s-tooltip" data-s-tooltip-placement="right" aria-pressed="false" aria-label="Down vote" data-selected-classes="fc-theme-primary bc-theme-primary bg-theme-primary-025" data-unselected-classes="bc-black-100 fc-black-700 h:bg-theme-primary-100" aria-describedby="--stacks-s-tooltip-o1hvm4lz" data-webtasks-id="a94cffc5-825c-4cc7"> |
|
<svg aria-hidden="true" class="svg-icon iconArrowDown" width="18" height="18" viewBox="0 0 18 18" data-webtasks-id="9279f47e-ded3-4434"><path d="M1 6h16l-8 8-8-8Z" data-webtasks-id="5f24bcd4-fce0-4fbd"></path></svg> |
|
</button><div id="--stacks-s-tooltip-o1hvm4lz" class="s-popover s-popover__tooltip" role="tooltip" data-webtasks-id="9b91b126-4b6c-48fb">This answer is not useful<div class="s-popover--arrow" data-webtasks-id="ebaf04ff-0774-4afa"></div></div> |
|
|
|
|
|
|
|
<button class="js-saves-btn s-btn s-btn__unset c-pointer py4" type="button" id="saves-btn-181892" data-controller="s-tooltip" data-s-tooltip-placement="right" data-s-popover-placement="" aria-pressed="false" data-post-id="181892" data-post-type-id="2" data-user-privilege-for-post-click="-1" aria-controls="" data-s-popover-auto-show="false" aria-describedby="--stacks-s-tooltip-x6ti701o" data-webtasks-id="be95f514-631b-446b"> |
|
<svg aria-hidden="true" class="fc-theme-primary-500 js-saves-btn-selected d-none svg-icon iconBookmark" width="18" height="18" viewBox="0 0 18 18" data-webtasks-id="08cc7daa-0d27-47da"><path d="M3 17V3c0-1.1.9-2 2-2h8a2 2 0 0 1 2 2v14l-6-4-6 4Z" data-webtasks-id="9a44713e-5fdb-4a52"></path></svg> |
|
<svg aria-hidden="true" class="js-saves-btn-unselected svg-icon iconBookmarkAlt" width="18" height="18" viewBox="0 0 18 18" data-webtasks-id="80f53a7d-5b80-4278"><path d="m9 10.6 4 2.66V3H5v10.26l4-2.66ZM3 17V3c0-1.1.9-2 2-2h8a2 2 0 0 1 2 2v14l-6-4-6 4Z" data-webtasks-id="56c92beb-7360-4b07"></path></svg> |
|
</button><div id="--stacks-s-tooltip-x6ti701o" class="s-popover s-popover__tooltip" role="tooltip" data-webtasks-id="d6ec0e95-20c9-40e0">Save this answer.<div class="s-popover--arrow" data-webtasks-id="5ae872e4-6700-4e57"></div></div> |
|
|
|
|
|
|
|
|
|
|
|
|
|
|
|
<div class="js-accepted-answer-indicator flex--item fc-green-700 py6 mtn8 d-none" data-s-tooltip-placement="right" title="Loading when this answer was accepted…" tabindex="0" role="note" aria-label="Accepted" data-webtasks-id="76b0ea6f-7fb3-4759"> |
|
<div class="ta-center" data-webtasks-id="9f8e0e84-b90e-4a1a"> |
|
<svg aria-hidden="true" class="svg-icon iconCheckmarkLg" width="36" height="36" viewBox="0 0 36 36" data-webtasks-id="03bffaf3-610c-4fbd"><path d="m6 14 8 8L30 6v8L14 30l-8-8v-8Z" data-webtasks-id="bc642f96-fbfd-46a2"></path></svg> |
|
</div> |
|
</div> |
|
|
|
|
|
<a class="js-post-issue flex--item s-btn s-btn__unset c-pointer py6 mx-auto" href="/posts/181892/timeline" data-shortcut="T" data-ks-title="timeline" data-controller="s-tooltip" data-s-tooltip-placement="right" aria-label="Timeline" aria-describedby="--stacks-s-tooltip-f0r79sxd" data-webtasks-id="4a02dfad-77b6-475b"><svg aria-hidden="true" class="mln2 mr0 svg-icon iconHistory" width="19" height="18" viewBox="0 0 19 18" data-webtasks-id="b62e1608-394b-490a"><path d="M3 9a8 8 0 1 1 3.73 6.77L8.2 14.3A6 6 0 1 0 5 9l3.01-.01-4 4-4-4h3L3 9Zm7-4h1.01L11 9.36l3.22 2.1-.6.93L10 10V5Z" data-webtasks-id="e9af4e71-4c4e-4d8e"></path></svg></a><div id="--stacks-s-tooltip-f0r79sxd" class="s-popover s-popover__tooltip" role="tooltip" data-webtasks-id="41811098-09cb-4bc0">Show activity on this post.<div class="s-popover--arrow" data-webtasks-id="a8a6284b-cd1f-496b"></div></div> |
|
|
|
</div> |
|
|
|
</div> |
|
|
|
|
|
|
|
<div class="answercell post-layout--right" data-webtasks-id="ecad2636-79ec-4669"> |
|
|
|
<div class="s-prose js-post-body" itemprop="text" data-webtasks-id="b64ae356-506b-442f"> |
|
<p data-webtasks-id="fd19d2db-be0c-43f1">Based on MastaBabas answer. You can clearly see from Google Maps that the location is on the border between France and Italy (in fact the border runs through that bridge), in a place called Menton Saint Louis. See link to the below image <a href="https://www.google.com/maps/place/Menton+Saint+Louis/@43.7762962,7.5170085,267a,35y,43.12h,78.97t/data=!3m1!1e3!4m6!3m5!1s0x12cdec0964476ee7:0x1403197ea5776521!8m2!3d43.7859516!4d7.5284511!16s%2Fg%2F11c6rgjw18?entry=ttu" rel="noreferrer" data-webtasks-id="32472bce-4f79-4f8d">here</a> including the mountains, bridge, and style of trees around the costal area.</p> |
|
<p data-webtasks-id="8c2e9651-351a-41b1"><a href="https://i.stack.imgur.com/mS25r.jpg" rel="noreferrer" data-webtasks-id="45b6920f-f241-4b47"><img src="https://i.stack.imgur.com/mS25r.jpg" alt="Google Maps Image of Location" data-webtasks-id="794330af-c533-4ccc"></a></p> |
|
<ul data-webtasks-id="10bcae65-b5ed-43bf"> |
|
<li data-webtasks-id="69264590-e6ec-42fa"><a href="https://www.google.com/maps?ll=43.786120,7.529919&q=43.786120,7.529919&hl=en&t=k&z=19" rel="noreferrer" data-webtasks-id="3edf7735-1b74-4644">Menton - Pont Saint Louis (bridge), 43.786120 ,7.529919</a> (Google Maps) |
|
<ul data-webtasks-id="1e4a89da-7ebb-4e5f"> |
|
<li data-webtasks-id="8cc31959-9f22-4dab">with Street View, the view from the bridge can be seen</li> |
|
</ul> |
|
</li> |
|
</ul> |
|
<p data-webtasks-id="e0016298-6d02-4b1e"><a href="https://i.stack.imgur.com/e66tV.jpg" rel="noreferrer" data-webtasks-id="7cf50025-d7f0-4381"><img src="https://i.stack.imgur.com/e66tV.jpg" alt="ca. 1920 Menton - Pont Saint Louis (bridge) - Italy" data-webtasks-id="d09765f1-cba8-46d9"></a> |
|
<a href="https://i.stack.imgur.com/PDzwj.jpg" rel="noreferrer" data-webtasks-id="a13c98dd-31fa-4ded"><img src="https://i.stack.imgur.com/PDzwj.jpg" alt="enter image description here" data-webtasks-id="27ed5821-14c1-4397"></a></p> |
|
<ul data-webtasks-id="153192f8-6f81-4ecb"> |
|
<li data-webtasks-id="4c128b9a-0e3f-4572">around 1920</li> |
|
</ul> |
|
</div> |
|
<div class="mt24" data-webtasks-id="8389d1da-ecd6-4292"> |
|
<div class="d-flex fw-wrap ai-start jc-end gs8 gsy" data-webtasks-id="bf460afa-b74f-43fb"> |
|
<time itemprop="dateCreated" datetime="2023-06-20T11:23:04" data-webtasks-id="f4c5e188-d567-4c4f"></time> |
|
<div class="flex--item mr16" style="flex: 1 1 100px;" data-webtasks-id="a23ab6f5-15da-4c40"> |
|
|
|
|
|
|
|
<div class="js-post-menu pt2" data-post-id="181892" data-post-type-id="2" data-webtasks-id="acd27b4b-d4f7-4d6b"> |
|
|
|
<div class="d-flex gs8 s-anchors s-anchors__muted fw-wrap" data-webtasks-id="b6004c64-c197-4c2d"> |
|
|
|
<div class="flex--item" data-webtasks-id="8e13bc86-874f-4de2"> |
|
<a href="/a/181892/136878" rel="nofollow" itemprop="url" class="js-share-link js-gps-track" title="Short permalink to this answer" data-gps-track="post.click({ item: 2, priv: -1, post_type: 2 })" data-controller="se-share-sheet s-popover" data-se-share-sheet-title="Share a link to this answer" data-se-share-sheet-subtitle="(Includes your user id)" data-se-share-sheet-post-type="answer" data-se-share-sheet-social="facebook twitter " data-se-share-sheet-location="2" data-se-share-sheet-license-url="https%3a%2f%2fcreativecommons.org%2flicenses%2fby-sa%2f4.0%2f" data-se-share-sheet-license-name="CC BY-SA 4.0" data-s-popover-placement="bottom-start" aria-controls="se-share-sheet-2" data-action=" s-popover#toggle se-share-sheet#preventNavigation s-popover:show->se-share-sheet#willShow s-popover:shown->se-share-sheet#didShow" aria-expanded="false" data-webtasks-id="9161b071-3c67-4f13">Share</a><div class="s-popover z-dropdown s-anchors s-anchors__default" style="width: unset; max-width: 28em;" id="se-share-sheet-2" data-webtasks-id="57ea9532-8293-464e"><div class="s-popover--arrow" data-webtasks-id="3a625dc2-4c8f-4657"></div><div data-webtasks-id="02fbdb80-f74e-414f"><label for="share-sheet-input-se-share-sheet-2" data-webtasks-id="aa86b883-3957-4ae0"><span class="js-title fw-bold" data-webtasks-id="fb401b13-35dd-4914">Share a link to this answer</span> <span class="js-subtitle" data-webtasks-id="46c3899f-f342-449e">(Includes your user id)</span></label></div><div class="my8" data-webtasks-id="d9afe053-5f9d-4ee0"><input type="text" id="share-sheet-input-se-share-sheet-2" class="js-input s-input wmn3 sm:wmn-initial bc-black-200 bg-white fc-dark" readonly="" data-webtasks-id="388ec346-221d-4bfc"></div><div class="d-flex jc-space-between ai-center mbn4" data-webtasks-id="57c93a0c-5f82-4aa9"><button class="js-copy-link-btn s-btn s-btn__link js-gps-track" data-gps-track="" data-webtasks-id="04910cf0-42eb-41e3">Copy link</button><a href="https://creativecommons.org/licenses/by-sa/4.0/" rel="license" class="js-license s-block-link w-auto" target="_blank" title="The current license for this post: CC BY-SA 4.0" data-webtasks-id="36bf2c3b-20c6-48b4">CC BY-SA 4.0</a><div class="js-social-container d-none" data-webtasks-id="0bf6a151-e607-4ad0"></div></div></div> |
|
</div> |
|
|
|
|
|
<div class="flex--item" data-webtasks-id="1d91ad73-cca5-4935"> |
|
<a href="/posts/181892/edit" class="js-suggest-edit-post js-gps-track" data-gps-track="post.click({ item: 6, priv: -1, post_type: 2 })" title="Revise and improve this post" data-webtasks-id="a4fd6ba8-f2f1-4c15">Edit</a> |
|
</div> |
|
|
|
<div class="flex--item" data-webtasks-id="c9e771fb-fa80-463f"> |
|
<button type="button" id="btnFollowPost-181892" class="s-btn s-btn__link js-follow-post js-follow-answer js-gps-track" data-gps-track="post.click({ item: 14, priv: -1, post_type: 2 })" data-controller="s-tooltip " data-s-tooltip-placement="bottom" data-s-popover-placement="bottom" aria-controls="" aria-describedby="--stacks-s-tooltip-ssjxf19s" data-webtasks-id="080f0a1b-2bf9-4b8f"> |
|
Follow |
|
</button><div id="--stacks-s-tooltip-ssjxf19s" class="s-popover s-popover__tooltip" role="tooltip" data-webtasks-id="91f7c276-d99e-4409">Follow this answer to receive notifications<div class="s-popover--arrow" data-webtasks-id="331142c4-3e3f-4fa2"></div></div> |
|
</div> |
|
|
|
|
|
|
|
|
|
|
|
|
|
</div> |
|
<div class="js-menu-popup-container" data-webtasks-id="1acf2f99-437a-443d"></div> |
|
</div> |
|
</div> |
|
<div class="post-signature flex--item fl0" data-webtasks-id="bc920069-2572-4cf0"> |
|
<div class="user-info " data-webtasks-id="d1103d4a-bae2-432e"> |
|
<div class="user-action-time" data-webtasks-id="8c9c913b-1c9c-4c0f"> |
|
<a href="/posts/181892/revisions" title="show all edits to this post" class="js-gps-track" data-gps-track="post.click({ item: 4, priv: -1, post_type: 2 })" data-webtasks-id="71dd596d-040e-4b01">edited <span title="2023-06-20 19:41:50Z" class="relativetime" data-webtasks-id="b6ad104b-41c6-4982">Jun 20 at 19:41</span></a> |
|
</div> |
|
<div class="user-gravatar32" data-webtasks-id="18fbc583-27f7-44c3"> |
|
<a href="/users/95267/mark-johnson" data-webtasks-id="ce633895-06da-426b"><div class="gravatar-wrapper-32" data-webtasks-id="9402e25d-0f5f-4d57"><img src="https://lh4.googleusercontent.com/-Sz39icOHOnU/AAAAAAAAAAI/AAAAAAAACd8/UeNRBD3O2KY/photo.jpg?sz=64" alt="Mark Johnson's user avatar" width="32" height="32" class="bar-sm" data-webtasks-id="92dca06c-94ef-44b8"></div></a> |
|
</div> |
|
<div class="user-details" data-webtasks-id="056e2ec6-c209-403f"> |
|
<a href="/users/95267/mark-johnson" data-webtasks-id="bd47c3c8-a42b-4195">Mark Johnson</a> |
|
<div class="-flair" data-webtasks-id="007d9728-20ff-4ce2"> |
|
<span class="reputation-score" title="reputation score 22,559" dir="ltr" data-webtasks-id="447ced8c-90f6-4e7b">22.6k</span><span title="2 gold badges" aria-hidden="true" data-webtasks-id="0a0f2457-31df-4af6"><span class="badge1" data-webtasks-id="10b3195f-4f9e-445e"></span><span class="badgecount" data-webtasks-id="3ac2f6f9-12f8-42cf">2</span></span><span class="v-visible-sr" data-webtasks-id="7ac7f267-65fc-49af">2 gold badges</span><span title="38 silver badges" aria-hidden="true" data-webtasks-id="fd54e1e9-38ed-4f56"><span class="badge2" data-webtasks-id="91224fe6-3feb-4522"></span><span class="badgecount" data-webtasks-id="aa37978f-7db4-47d0">38</span></span><span class="v-visible-sr" data-webtasks-id="cb34bb2f-d2ef-4d15">38 silver badges</span><span title="93 bronze badges" aria-hidden="true" data-webtasks-id="daa346a4-bd30-4621"><span class="badge3" data-webtasks-id="97e6cbab-0dc8-4949"></span><span class="badgecount" data-webtasks-id="e39046cd-685e-4c1a">93</span></span><span class="v-visible-sr" data-webtasks-id="7eecd504-a2f5-40da">93 bronze badges</span> |
|
</div> |
|
</div> |
|
</div> |
|
</div> |
|
|
|
|
|
<div class="post-signature flex--item fl0" data-webtasks-id="8db524c4-974a-48c9"> |
|
<div class="user-info user-hover" data-webtasks-id="c2dcd085-6318-46c9"> |
|
<div class="user-action-time" data-webtasks-id="ff21d221-42b3-4d9b"> |
|
answered <span title="2023-06-20 11:23:04Z" class="relativetime" data-webtasks-id="e0b9589f-31e7-4d9a">Jun 20 at 11:23</span> |
|
</div> |
|
<div class="user-gravatar32" data-webtasks-id="f24c8f5a-8964-4bfb"> |
|
<a href="/users/13535/dean-meehan" data-webtasks-id="8695cf71-c68f-49e1"><div class="gravatar-wrapper-32" data-webtasks-id="8bcaaa3d-40dd-497f"><img src="https://www.gravatar.com/avatar/2191f177d8fa8b98113039e097964f33?s=64&d=identicon&r=PG" alt="Dean Meehan's user avatar" width="32" height="32" class="bar-sm" data-webtasks-id="17e455fa-5dc6-49ae"></div></a> |
|
</div> |
|
<div class="user-details" itemprop="author" itemscope="" itemtype="http://schema.org/Person" data-webtasks-id="d4c33616-0fb4-4561"> |
|
<a href="/users/13535/dean-meehan" data-webtasks-id="37e72247-df46-4834">Dean Meehan</a><span class="d-none" itemprop="name" data-webtasks-id="4494a3aa-ce17-46dc">Dean Meehan</span> |
|
<div class="-flair" data-webtasks-id="5c139833-9eb5-4d92"> |
|
<span class="reputation-score" title="reputation score " dir="ltr" data-webtasks-id="978a4b0b-957f-4a35">633</span><span title="1 gold badge" aria-hidden="true" data-webtasks-id="20e57a3f-bc17-42ae"><span class="badge1" data-webtasks-id="2a69d713-d105-400f"></span><span class="badgecount" data-webtasks-id="6c82ba6b-7611-47ad">1</span></span><span class="v-visible-sr" data-webtasks-id="e149a3b1-8104-4a57">1 gold badge</span><span title="5 silver badges" aria-hidden="true" data-webtasks-id="bb2ad6c9-4a5f-4c1d"><span class="badge2" data-webtasks-id="4ada94df-c770-46b6"></span><span class="badgecount" data-webtasks-id="b084cd15-1615-4d7a">5</span></span><span class="v-visible-sr" data-webtasks-id="15ba5ab2-e180-47ed">5 silver badges</span><span title="12 bronze badges" aria-hidden="true" data-webtasks-id="47e331f3-93f9-4ff5"><span class="badge3" data-webtasks-id="7e5b2c4e-08dc-4a42"></span><span class="badgecount" data-webtasks-id="ecf70203-2067-42e7">12</span></span><span class="v-visible-sr" data-webtasks-id="8d56ae57-b78c-4480">12 bronze badges</span> |
|
</div> |
|
</div> |
|
</div> |
|
|
|
|
|
</div> |
|
</div> |
|
|
|
|
|
</div> |
|
|
|
</div> |
|
|
|
|
|
|
|
|
|
<span class="d-none" itemprop="commentCount" data-webtasks-id="2046b883-91c3-4e9d">9</span> |
|
<div class="post-layout--right js-post-comments-component" data-webtasks-id="e00d8260-1e19-49cc"> |
|
<div id="comments-181892" class="comments js-comments-container bt bc-black-075 mt12 " data-post-id="181892" data-min-length="15" data-webtasks-id="bbaae975-1d42-4942"> |
|
<ul class="comments-list js-comments-list" data-remaining-comments-count="4" data-canpost="false" data-cansee="true" data-comments-unavailable="false" data-addlink-disabled="true" data-webtasks-id="5a4a1b75-433c-433e"> |
|
|
|
<li id="comment-462852" class="comment js-comment " data-comment-id="462852" data-comment-owner-id="27650" data-comment-score="4" data-webtasks-id="8aada808-3786-47e0"> |
|
<div class="js-comment-actions comment-actions" data-webtasks-id="b71574fe-2804-4e92"> |
|
<div class="comment-score js-comment-score js-comment-edit-hide" data-webtasks-id="a33fc630-e605-4118"> |
|
<span title="number of 'useful comment' votes received" class="cool" data-webtasks-id="2cc9966d-704d-40a5">4</span> |
|
</div> |
|
</div> |
|
<div class="comment-text js-comment-text-and-form" data-webtasks-id="ab4dc88b-bc02-4586"> |
|
<div class="comment-body js-comment-edit-hide" data-webtasks-id="2f28e0d2-0666-4498"> |
|
|
|
<span class="comment-copy" data-webtasks-id="71255bd7-a536-4ef3">Can you please edit in the location in words, not just a link? No need to be exact but a country and region will help those that read the answer.</span> |
|
|
|
<div class="d-inline-flex ai-center" data-webtasks-id="269ff44e-ab1d-41d2"> |
|
– <a href="/users/27650/willeke" title="57,603 reputation" class="comment-user" data-webtasks-id="9c923134-c31d-49bd">Willeke</a> |
|
<span class="mod-flair " title="Moderator" data-webtasks-id="537f074a-cc30-48f7">♦</span> </div> |
|
<span class="comment-date" dir="ltr" data-webtasks-id="61902cfb-29ed-4790"><a class="comment-link" href="#comment462852_181892" aria-label="Link to comment" data-webtasks-id="ba2d22ea-57cd-49a6"><span title="2023-06-20 12:05:18Z, License: CC BY-SA 4.0" class="relativetime-clean" data-webtasks-id="90c5c3ce-c387-4145">Jun 20 at 12:05</span></a></span> |
|
</div> |
|
</div> |
|
</li> |
|
<li id="comment-462855" class="comment js-comment " data-comment-id="462855" data-comment-owner-id="42899" data-comment-score="6" data-webtasks-id="258872c5-1421-4898"> |
|
<div class="js-comment-actions comment-actions" data-webtasks-id="66e973da-06ee-4493"> |
|
<div class="comment-score js-comment-score js-comment-edit-hide" data-webtasks-id="001e3d1e-8936-480b"> |
|
<span title="number of 'useful comment' votes received" class="warm" data-webtasks-id="8ba40c05-ae20-4914">6</span> |
|
</div> |
|
</div> |
|
<div class="comment-text js-comment-text-and-form" data-webtasks-id="ab1a70b9-7854-4035"> |
|
<div class="comment-body js-comment-edit-hide" data-webtasks-id="435720ac-0dd3-45f6"> |
|
|
|
<span class="comment-copy" data-webtasks-id="71e5fee9-3eb4-485c">When (not if) Google's map linking formats change, the link will die and "here" will be as useful as the original painting...</span> |
|
|
|
<div class="d-inline-flex ai-center" data-webtasks-id="ef0c9a5c-abc4-49d8"> |
|
– <a href="/users/42899/freeman" title="1,059 reputation" class="comment-user" data-webtasks-id="535daac4-83b7-4d36">FreeMan</a> |
|
</div> |
|
<span class="comment-date" dir="ltr" data-webtasks-id="9c4efa0f-1872-45a9"><a class="comment-link" href="#comment462855_181892" aria-label="Link to comment" data-webtasks-id="6cce3777-b93a-4d7c"><span title="2023-06-20 12:20:19Z, License: CC BY-SA 4.0" class="relativetime-clean" data-webtasks-id="e266c1b1-37e4-452f">Jun 20 at 12:20</span></a></span> |
|
</div> |
|
</div> |
|
</li> |
|
<li id="comment-462864" class="comment js-comment " data-comment-id="462864" data-comment-owner-id="22731" data-comment-score="2" data-webtasks-id="153bf497-e693-4121"> |
|
<div class="js-comment-actions comment-actions" data-webtasks-id="aaf22f4a-1181-431b"> |
|
<div class="comment-score js-comment-score js-comment-edit-hide" data-webtasks-id="5f378fd6-2a87-455d"> |
|
<span title="number of 'useful comment' votes received" class="cool" data-webtasks-id="85cd6ef4-17d7-4304">2</span> |
|
</div> |
|
</div> |
|
<div class="comment-text js-comment-text-and-form" data-webtasks-id="e55ebc07-2f90-443b"> |
|
<div class="comment-body js-comment-edit-hide" data-webtasks-id="ca46b147-3360-43b7"> |
|
|
|
<span class="comment-copy" data-webtasks-id="e328b8ba-eb3b-4d4b">@FreeMan thankfully, Google Maps URLs aren't completely opaque, and the <code data-webtasks-id="8213832c-ff9a-4fdd">@43.7762962,7.5170085</code> it contains can actually be helpful even if Google Maps disappears (even if it's not <i data-webtasks-id="d643ca6a-9a1e-4b3a">exactly</i> where the link points to). (Just in case it's not obvious, these are latitude and longitude coordinates.)</span> |
|
|
|
<div class="d-inline-flex ai-center" data-webtasks-id="b2a963cb-87ac-48e6"> |
|
– <a href="/users/22731/bruno" title="101 reputation" class="comment-user" data-webtasks-id="bc7e9582-6bbd-4db5">Bruno</a> |
|
</div> |
|
<span class="comment-date" dir="ltr" data-webtasks-id="7c5a0051-19d0-455d"><a class="comment-link" href="#comment462864_181892" aria-label="Link to comment" data-webtasks-id="e2b21b42-9059-440a"><span title="2023-06-20 13:18:28Z, License: CC BY-SA 4.0" class="relativetime-clean" data-webtasks-id="7109c867-6a23-42f3">Jun 20 at 13:18</span></a></span> |
|
</div> |
|
</div> |
|
</li> |
|
<li id="comment-462919" class="comment js-comment " data-comment-id="462919" data-comment-owner-id="27650" data-comment-score="2" data-webtasks-id="a948b0c3-0afe-4582"> |
|
<div class="js-comment-actions comment-actions" data-webtasks-id="3c711543-669e-40e7"> |
|
<div class="comment-score js-comment-score js-comment-edit-hide" data-webtasks-id="0c66a200-8028-45bd"> |
|
<span title="number of 'useful comment' votes received" class="cool" data-webtasks-id="de4c38dc-fbbd-415b">2</span> |
|
</div> |
|
</div> |
|
<div class="comment-text js-comment-text-and-form" data-webtasks-id="5a652016-fd7b-4aa3"> |
|
<div class="comment-body js-comment-edit-hide" data-webtasks-id="c07708ad-1f28-4c8c"> |
|
|
|
<span class="comment-copy" data-webtasks-id="fc2aa668-aac2-48bb">@Bruno my point, as in my first comment, you should not need to follow a link to understand the answer.</span> |
|
|
|
<div class="d-inline-flex ai-center" data-webtasks-id="395dd7e9-1397-49b7"> |
|
– <a href="/users/27650/willeke" title="57,603 reputation" class="comment-user" data-webtasks-id="26db3fe0-a536-4005">Willeke</a> |
|
<span class="mod-flair " title="Moderator" data-webtasks-id="894e3df1-4135-4806">♦</span> </div> |
|
<span class="comment-date" dir="ltr" data-webtasks-id="bea683ea-dc49-4109"><a class="comment-link" href="#comment462919_181892" aria-label="Link to comment" data-webtasks-id="218077ae-e162-45ae"><span title="2023-06-20 19:47:37Z, License: CC BY-SA 4.0" class="relativetime-clean" data-webtasks-id="5eb1f9a7-9ac9-4f46">Jun 20 at 19:47</span></a></span> |
|
<span title="this comment was edited 1 time" data-webtasks-id="acb6f305-5cbc-470a"> |
|
<svg aria-hidden="true" class="va-text-bottom o50 svg-icon iconPencilSm" width="14" height="14" viewBox="0 0 14 14" data-webtasks-id="802e4e25-7170-4ef5"><path d="m11.1 1.71 1.13 1.12c.2.2.2.51 0 .71L11.1 4.7 9.21 2.86l1.17-1.15c.2-.2.51-.2.71 0ZM2 10.12l6.37-6.43 1.88 1.88L3.88 12H2v-1.88Z" data-webtasks-id="73ee05a7-d169-498e"></path></svg> |
|
</span> |
|
</div> |
|
</div> |
|
</li> |
|
<li id="comment-462986" class="comment js-comment " data-comment-id="462986" data-comment-owner-id="13535" data-comment-score="2" data-webtasks-id="870cab88-fded-4077"> |
|
<div class="js-comment-actions comment-actions" data-webtasks-id="f95dd672-2517-45ae"> |
|
<div class="comment-score js-comment-score js-comment-edit-hide" data-webtasks-id="8f2e0de2-c213-43ae"> |
|
<span title="number of 'useful comment' votes received" class="cool" data-webtasks-id="16ef66c3-caf6-4c32">2</span> |
|
</div> |
|
</div> |
|
<div class="comment-text js-comment-text-and-form" data-webtasks-id="cfaa9189-7e01-451f"> |
|
<div class="comment-body js-comment-edit-hide" data-webtasks-id="9f2c1797-8a10-4fc2"> |
|
|
|
<span class="comment-copy" data-webtasks-id="280245ff-02bd-4760">Thanks for the edits to make the answer better. @ToivoSäwén The question has been edited since I created this answer which is why it looks different.</span> |
|
|
|
<div class="d-inline-flex ai-center" data-webtasks-id="ed050215-186b-479d"> |
|
– <a href="/users/13535/dean-meehan" title="633 reputation" class="comment-user" data-webtasks-id="88561b3d-a327-48c3">Dean Meehan</a> |
|
</div> |
|
<span class="comment-date" dir="ltr" data-webtasks-id="6252b40a-46ba-4a92"><a class="comment-link" href="#comment462986_181892" aria-label="Link to comment" data-webtasks-id="1da3b600-bb4c-44c4"><span title="2023-06-21 15:14:02Z, License: CC BY-SA 4.0" class="relativetime-clean" data-webtasks-id="52fac78c-472c-44ac">Jun 21 at 15:14</span></a></span> |
|
</div> |
|
</div> |
|
</li> |
|
|
|
</ul> |
|
</div> |
|
|
|
<div id="comments-link-181892" data-rep="50" data-reg="true" data-webtasks-id="89f63fd2-60dc-4849"> |
|
<a class="js-add-link comments-link dno" title="Use comments to ask for more information or suggest improvements. Avoid comments like “+1” or “thanks”." href="#" role="button" data-webtasks-id="1af4aa95-88ea-426f"></a> |
|
<span class="js-link-separator dno" data-webtasks-id="e9a38d64-114d-4439"> | </span> |
|
<a class="js-show-link comments-link " title="Expand to show all comments on this post" href="#" onclick="" role="button" data-webtasks-id="a599f36a-8e2c-4659">Show <b data-webtasks-id="04340127-947c-4aa5">4</b> more comments</a> |
|
</div> |
|
</div> |
|
</div> |
|
</div> |
|
|
|
|
|
<a name="181924" data-webtasks-id="f39be0c3-f053-40c4"></a> |
|
<div id="answer-181924" class="answer js-answer" data-answerid="181924" data-parentid="181865" data-score="3" data-position-on-page="3" data-highest-scored="0" data-question-has-accepted-highest-score="0" itemprop="suggestedAnswer" itemscope="" itemtype="https://schema.org/Answer" data-webtasks-id="dc41499d-5c7b-4316"> |
|
<div class="post-layout" data-webtasks-id="bc680e96-fd07-4ce9"> |
|
<div class="votecell post-layout--left" data-webtasks-id="dd895879-dbab-47b1"> |
|
<div class="js-voting-container d-flex jc-center fd-column ai-stretch gs4 fc-black-200" data-post-id="181924" data-webtasks-id="50b837be-516e-4625"> |
|
<button class="js-vote-up-btn flex--item s-btn ba bar-pill c-pointer as-center bc-black-100 fc-black-700 h:bg-theme-primary-100" data-controller="s-tooltip" data-s-tooltip-placement="right" aria-pressed="false" aria-label="Up vote" data-selected-classes="fc-theme-primary bc-theme-primary bg-theme-primary-025" data-unselected-classes="bc-black-100 fc-black-700 h:bg-theme-primary-100" aria-describedby="--stacks-s-tooltip-yjax1bvy" data-webtasks-id="c5432426-b803-4255"> |
|
<svg aria-hidden="true" class="svg-icon iconArrowUp" width="18" height="18" viewBox="0 0 18 18" data-webtasks-id="be1adf80-131b-4a5e"><path d="M1 12h16L9 4l-8 8Z" data-webtasks-id="17f76542-7370-4797"></path></svg> |
|
</button><div id="--stacks-s-tooltip-yjax1bvy" class="s-popover s-popover__tooltip" role="tooltip" data-webtasks-id="46e22506-0cff-4c93">This answer is useful<div class="s-popover--arrow" data-webtasks-id="486bfd92-cb89-4342"></div></div> |
|
<div class="js-vote-count flex--item d-flex fd-column ai-center fc-theme-body-font fw-bold fs-subheading py4" itemprop="upvoteCount" data-value="3" data-webtasks-id="67160ac2-7052-4284"> |
|
3 |
|
</div> |
|
<button class="js-vote-down-btn flex--item mb8 s-btn ba bar-pill c-pointer as-center bc-black-100 fc-black-700 h:bg-theme-primary-100" data-controller="s-tooltip" data-s-tooltip-placement="right" aria-pressed="false" aria-label="Down vote" data-selected-classes="fc-theme-primary bc-theme-primary bg-theme-primary-025" data-unselected-classes="bc-black-100 fc-black-700 h:bg-theme-primary-100" aria-describedby="--stacks-s-tooltip-48p38d2i" data-webtasks-id="3f4a4bfd-0587-43ea"> |
|
<svg aria-hidden="true" class="svg-icon iconArrowDown" width="18" height="18" viewBox="0 0 18 18" data-webtasks-id="05434996-bae5-42c9"><path d="M1 6h16l-8 8-8-8Z" data-webtasks-id="0e34d8c0-04f2-4f32"></path></svg> |
|
</button><div id="--stacks-s-tooltip-48p38d2i" class="s-popover s-popover__tooltip" role="tooltip" data-webtasks-id="bd437548-1e11-42c7">This answer is not useful<div class="s-popover--arrow" data-webtasks-id="04166657-cf4a-4f43"></div></div> |
|
|
|
|
|
|
|
<button class="js-saves-btn s-btn s-btn__unset c-pointer py4" type="button" id="saves-btn-181924" data-controller="s-tooltip" data-s-tooltip-placement="right" data-s-popover-placement="" aria-pressed="false" data-post-id="181924" data-post-type-id="2" data-user-privilege-for-post-click="-1" aria-controls="" data-s-popover-auto-show="false" aria-describedby="--stacks-s-tooltip-j8wnbhtb" data-webtasks-id="4e399683-294e-4803"> |
|
<svg aria-hidden="true" class="fc-theme-primary-500 js-saves-btn-selected d-none svg-icon iconBookmark" width="18" height="18" viewBox="0 0 18 18" data-webtasks-id="a20ecde9-768c-4b5a"><path d="M3 17V3c0-1.1.9-2 2-2h8a2 2 0 0 1 2 2v14l-6-4-6 4Z" data-webtasks-id="b222c046-0070-4eef"></path></svg> |
|
<svg aria-hidden="true" class="js-saves-btn-unselected svg-icon iconBookmarkAlt" width="18" height="18" viewBox="0 0 18 18" data-webtasks-id="3a3c0caf-1e3c-4eae"><path d="m9 10.6 4 2.66V3H5v10.26l4-2.66ZM3 17V3c0-1.1.9-2 2-2h8a2 2 0 0 1 2 2v14l-6-4-6 4Z" data-webtasks-id="a728a094-ccbb-4377"></path></svg> |
|
</button><div id="--stacks-s-tooltip-j8wnbhtb" class="s-popover s-popover__tooltip" role="tooltip" data-webtasks-id="497046c0-2b2d-4481">Save this answer.<div class="s-popover--arrow" data-webtasks-id="d8fcabe4-1df3-4524"></div></div> |
|
|
|
|
|
|
|
|
|
|
|
|
|
|
|
<div class="js-accepted-answer-indicator flex--item fc-green-700 py6 mtn8 d-none" data-s-tooltip-placement="right" title="Loading when this answer was accepted…" tabindex="0" role="note" aria-label="Accepted" data-webtasks-id="5002c2a9-7f01-4552"> |
|
<div class="ta-center" data-webtasks-id="9dd0b585-fadb-4c37"> |
|
<svg aria-hidden="true" class="svg-icon iconCheckmarkLg" width="36" height="36" viewBox="0 0 36 36" data-webtasks-id="dcbb490e-0d88-4e81"><path d="m6 14 8 8L30 6v8L14 30l-8-8v-8Z" data-webtasks-id="e6f8ef3e-2893-446f"></path></svg> |
|
</div> |
|
</div> |
|
|
|
|
|
<a class="js-post-issue flex--item s-btn s-btn__unset c-pointer py6 mx-auto" href="/posts/181924/timeline" data-shortcut="T" data-ks-title="timeline" data-controller="s-tooltip" data-s-tooltip-placement="right" aria-label="Timeline" aria-describedby="--stacks-s-tooltip-av5c4gvu" data-webtasks-id="8837b96a-421a-497b"><svg aria-hidden="true" class="mln2 mr0 svg-icon iconHistory" width="19" height="18" viewBox="0 0 19 18" data-webtasks-id="1623a235-b27f-48ac"><path d="M3 9a8 8 0 1 1 3.73 6.77L8.2 14.3A6 6 0 1 0 5 9l3.01-.01-4 4-4-4h3L3 9Zm7-4h1.01L11 9.36l3.22 2.1-.6.93L10 10V5Z" data-webtasks-id="0f28fa5b-782a-4bf2"></path></svg></a><div id="--stacks-s-tooltip-av5c4gvu" class="s-popover s-popover__tooltip" role="tooltip" data-webtasks-id="a153a812-efb4-4ace">Show activity on this post.<div class="s-popover--arrow" data-webtasks-id="f43a6829-253a-49dc"></div></div> |
|
|
|
</div> |
|
|
|
</div> |
|
|
|
|
|
|
|
<div class="answercell post-layout--right" data-webtasks-id="b49038c9-fa77-4fdd"> |
|
|
|
<div class="s-prose js-post-body" itemprop="text" data-webtasks-id="18f634f8-ab10-45fc"> |
|
<p data-webtasks-id="9bddf6f8-62da-4caa">People from southern Germany just love northern Italy, especially in Bavaria. It is easy to reach, cheap (used to be...), and folks speak German ("Südtirol"). |
|
From the mountains, the style of houses, and the vegetation I think this could be southern Tirol region, very likely around Lago di Garda.</p> |
|
</div> |
|
<div class="mt24" data-webtasks-id="3819d7e5-e3a3-45fe"> |
|
<div class="d-flex fw-wrap ai-start jc-end gs8 gsy" data-webtasks-id="35607dc2-5c92-419d"> |
|
<time itemprop="dateCreated" datetime="2023-06-21T19:05:38" data-webtasks-id="f10b6bfd-3c05-4d19"></time> |
|
<div class="flex--item mr16" style="flex: 1 1 100px;" data-webtasks-id="1ae53f18-e897-4927"> |
|
|
|
|
|
|
|
<div class="js-post-menu pt2" data-post-id="181924" data-post-type-id="2" data-webtasks-id="f4e73458-3a98-43a2"> |
|
|
|
<div class="d-flex gs8 s-anchors s-anchors__muted fw-wrap" data-webtasks-id="fbf005fd-8796-4ddf"> |
|
|
|
<div class="flex--item" data-webtasks-id="9c6f825f-52ce-43a8"> |
|
<a href="/a/181924/136878" rel="nofollow" itemprop="url" class="js-share-link js-gps-track" title="Short permalink to this answer" data-gps-track="post.click({ item: 2, priv: -1, post_type: 2 })" data-controller="se-share-sheet s-popover" data-se-share-sheet-title="Share a link to this answer" data-se-share-sheet-subtitle="(Includes your user id)" data-se-share-sheet-post-type="answer" data-se-share-sheet-social="facebook twitter " data-se-share-sheet-location="2" data-se-share-sheet-license-url="https%3a%2f%2fcreativecommons.org%2flicenses%2fby-sa%2f4.0%2f" data-se-share-sheet-license-name="CC BY-SA 4.0" data-s-popover-placement="bottom-start" aria-controls="se-share-sheet-3" data-action=" s-popover#toggle se-share-sheet#preventNavigation s-popover:show->se-share-sheet#willShow s-popover:shown->se-share-sheet#didShow" aria-expanded="false" data-webtasks-id="43da049b-4c11-428a">Share</a><div class="s-popover z-dropdown s-anchors s-anchors__default" style="width: unset; max-width: 28em;" id="se-share-sheet-3" data-webtasks-id="8ba2b96b-e249-4f28"><div class="s-popover--arrow" data-webtasks-id="b9021705-0858-492e"></div><div data-webtasks-id="dfa5aca6-b54c-4715"><label for="share-sheet-input-se-share-sheet-3" data-webtasks-id="95687565-c801-49d3"><span class="js-title fw-bold" data-webtasks-id="0af59679-1514-4b33">Share a link to this answer</span> <span class="js-subtitle" data-webtasks-id="5d2108b0-f8b3-47a3">(Includes your user id)</span></label></div><div class="my8" data-webtasks-id="c2735528-a551-4181"><input type="text" id="share-sheet-input-se-share-sheet-3" class="js-input s-input wmn3 sm:wmn-initial bc-black-200 bg-white fc-dark" readonly="" data-webtasks-id="051a7a63-b5f7-45cc"></div><div class="d-flex jc-space-between ai-center mbn4" data-webtasks-id="ff14c3e4-a3da-4541"><button class="js-copy-link-btn s-btn s-btn__link js-gps-track" data-gps-track="" data-webtasks-id="5f66e580-ec1c-45bc">Copy link</button><a href="https://creativecommons.org/licenses/by-sa/4.0/" rel="license" class="js-license s-block-link w-auto" target="_blank" title="The current license for this post: CC BY-SA 4.0" data-webtasks-id="b456fd66-9240-44c9">CC BY-SA 4.0</a><div class="js-social-container d-none" data-webtasks-id="e676a146-fdf1-47ec"></div></div></div> |
|
</div> |
|
|
|
|
|
<div class="flex--item" data-webtasks-id="b7ee181b-211a-49f5"> |
|
<a href="/posts/181924/edit" class="js-suggest-edit-post js-gps-track" data-gps-track="post.click({ item: 6, priv: -1, post_type: 2 })" title="Revise and improve this post" data-webtasks-id="59e4abf1-bb59-4457">Edit</a> |
|
</div> |
|
|
|
<div class="flex--item" data-webtasks-id="b7d605b3-b19b-4839"> |
|
<button type="button" id="btnFollowPost-181924" class="s-btn s-btn__link js-follow-post js-follow-answer js-gps-track" data-gps-track="post.click({ item: 14, priv: -1, post_type: 2 })" data-controller="s-tooltip " data-s-tooltip-placement="bottom" data-s-popover-placement="bottom" aria-controls="" aria-describedby="--stacks-s-tooltip-x8qtjrgy" data-webtasks-id="ae25d654-bd01-4967"> |
|
Follow |
|
</button><div id="--stacks-s-tooltip-x8qtjrgy" class="s-popover s-popover__tooltip" role="tooltip" data-webtasks-id="32f3e54a-2010-4a07">Follow this answer to receive notifications<div class="s-popover--arrow" data-webtasks-id="90367249-c40a-4876"></div></div> |
|
</div> |
|
|
|
|
|
|
|
|
|
|
|
|
|
</div> |
|
<div class="js-menu-popup-container" data-webtasks-id="e750a6e9-6d66-44eb"></div> |
|
</div> |
|
</div> |
|
|
|
|
|
<div class="post-signature flex--item fl0" data-webtasks-id="706c1008-673c-4c83"> |
|
<div class="user-info " data-webtasks-id="778c2041-cf67-432a"> |
|
<div class="user-action-time" data-webtasks-id="db820389-338a-4f8f"> |
|
answered <span title="2023-06-21 19:05:38Z" class="relativetime" data-webtasks-id="135709a0-bcae-424d">Jun 21 at 19:05</span> |
|
</div> |
|
<div class="user-gravatar32" data-webtasks-id="468f9c73-5b91-4d77"> |
|
<a href="/users/136748/r-c" data-webtasks-id="0adcab8d-2e6c-4cc2"><div class="gravatar-wrapper-32" data-webtasks-id="eaeffa5f-63ff-4b83"><img src="https://www.gravatar.com/avatar/1490f8cf8859f7f09a1fc4820c2eb62a?s=64&d=identicon&r=PG" alt="R. C.'s user avatar" width="32" height="32" class="bar-sm" data-webtasks-id="421379d7-fded-4ec5"></div></a> |
|
</div> |
|
<div class="user-details" itemprop="author" itemscope="" itemtype="http://schema.org/Person" data-webtasks-id="27894ce5-f798-4c33"> |
|
<a href="/users/136748/r-c" data-webtasks-id="d13ebae8-16af-4853">R. C.</a><span class="d-none" itemprop="name" data-webtasks-id="118cc4c0-59f9-4d0f">R. C.</span> |
|
<div class="-flair" data-webtasks-id="9a86b65c-65d9-40b0"> |
|
<span class="reputation-score" title="reputation score " dir="ltr" data-webtasks-id="39343af3-efc3-4de8">131</span><span title="1 bronze badge" aria-hidden="true" data-webtasks-id="630ac3ce-0ae8-4de9"><span class="badge3" data-webtasks-id="bdc4bd61-05d0-4c1e"></span><span class="badgecount" data-webtasks-id="d1bf7ade-4965-4a6e">1</span></span><span class="v-visible-sr" data-webtasks-id="9496d74b-f1ee-421f">1 bronze badge</span> |
|
</div> |
|
</div> |
|
</div> |
|
|
|
|
|
</div> |
|
</div> |
|
|
|
|
|
</div> |
|
|
|
</div> |
|
|
|
|
|
|
|
|
|
<span class="d-none" itemprop="commentCount" data-webtasks-id="d0d32877-9dee-4e2e">2</span> |
|
<div class="post-layout--right js-post-comments-component" data-webtasks-id="05dbcc9c-af56-417c"> |
|
<div id="comments-181924" class="comments js-comments-container bt bc-black-075 mt12 " data-post-id="181924" data-min-length="15" data-webtasks-id="ed8d7af7-a3b7-489e"> |
|
<ul class="comments-list js-comments-list" data-remaining-comments-count="0" data-canpost="false" data-cansee="true" data-comments-unavailable="false" data-addlink-disabled="true" data-webtasks-id="0f23448f-611b-4f12"> |
|
|
|
<li id="comment-463031" class="comment js-comment " data-comment-id="463031" data-comment-owner-id="2509" data-comment-score="1" data-webtasks-id="2190b39b-e804-4ff4"> |
|
<div class="js-comment-actions comment-actions" data-webtasks-id="b0d71ce2-cd33-4cf3"> |
|
<div class="comment-score js-comment-score js-comment-edit-hide" data-webtasks-id="f9e51364-ab95-43a8"> |
|
<span title="number of 'useful comment' votes received" class="cool" data-webtasks-id="b99f03cb-8dd0-4f86">1</span> |
|
</div> |
|
</div> |
|
<div class="comment-text js-comment-text-and-form" data-webtasks-id="9b32ecda-2654-4837"> |
|
<div class="comment-body js-comment-edit-hide" data-webtasks-id="777ca67e-f6b8-4c2c"> |
|
|
|
<span class="comment-copy" data-webtasks-id="c82843e3-778b-46d6">Lago di Garda is not in southern Tyrol.</span> |
|
|
|
<div class="d-inline-flex ai-center" data-webtasks-id="8877c124-dff8-45c9"> |
|
– <a href="/users/2509/gerrit" title="54,172 reputation" class="comment-user" data-webtasks-id="095734a6-389a-473e">gerrit</a> |
|
</div> |
|
<span class="comment-date" dir="ltr" data-webtasks-id="8edc10a2-e323-4e11"><a class="comment-link" href="#comment463031_181924" aria-label="Link to comment" data-webtasks-id="c100ae99-1260-4bbc"><span title="2023-06-22 07:07:17Z, License: CC BY-SA 4.0" class="relativetime-clean" data-webtasks-id="e351371c-d60c-43a9">Jun 22 at 7:07</span></a></span> |
|
</div> |
|
</div> |
|
</li> |
|
<li id="comment-463036" class="comment js-comment " data-comment-id="463036" data-comment-owner-id="27650" data-comment-score="0" data-webtasks-id="3dd1ff74-f75d-402c"> |
|
<div class="js-comment-actions comment-actions" data-webtasks-id="80f14c70-9c79-4e0f"> |
|
<div class="comment-score js-comment-score js-comment-edit-hide" data-webtasks-id="6d13cf51-86e4-4145"> |
|
</div> |
|
</div> |
|
<div class="comment-text js-comment-text-and-form" data-webtasks-id="477401ce-6857-414e"> |
|
<div class="comment-body js-comment-edit-hide" data-webtasks-id="d691cf66-8190-4c15"> |
|
|
|
<span class="comment-copy" data-webtasks-id="99a09e7e-80b6-4bc2">It is not far south of South Tyrol.</span> |
|
|
|
<div class="d-inline-flex ai-center" data-webtasks-id="991efd9d-b99a-46f7"> |
|
– <a href="/users/27650/willeke" title="57,603 reputation" class="comment-user" data-webtasks-id="0fff465e-3dc0-4fdf">Willeke</a> |
|
<span class="mod-flair " title="Moderator" data-webtasks-id="152870d5-b074-40bd">♦</span> </div> |
|
<span class="comment-date" dir="ltr" data-webtasks-id="ca76afe1-5e80-4f33"><a class="comment-link" href="#comment463036_181924" aria-label="Link to comment" data-webtasks-id="e3e1ac03-db5f-4c3f"><span title="2023-06-22 08:09:18Z, License: CC BY-SA 4.0" class="relativetime-clean" data-webtasks-id="9357181a-3ecc-4c7b">Jun 22 at 8:09</span></a></span> |
|
</div> |
|
</div> |
|
</li> |
|
|
|
</ul> |
|
</div> |
|
|
|
<div id="comments-link-181924" data-rep="50" data-reg="true" data-webtasks-id="04f04afd-ce41-4b82"> |
|
<a class="js-add-link comments-link disabled-link" title="Use comments to ask for more information or suggest improvements. Avoid comments like “+1” or “thanks”." href="#" role="button" data-webtasks-id="2a3d3b71-35fb-41ac">Add a comment</a> |
|
<span class="js-link-separator dno" data-webtasks-id="e705738b-30fd-4309"> | </span> |
|
<a class="js-show-link comments-link dno" title="Expand to show all comments on this post" href="#" onclick="" role="button" data-webtasks-id="948bd690-4eb4-45f6"></a> |
|
</div> |
|
</div> |
|
</div> |
|
</div> |
|
|
|
|
|
<a name="new-answer" data-webtasks-id="3b3d9bd3-3e22-481a"></a> |
|
<form id="post-form" action="/questions/181865/answer/submit" method="post" class="js-add-answer-component post-form" data-webtasks-id="646497be-1e65-4543"> |
|
<input type="hidden" id="post-id" value="181865" data-webtasks-id="0209cff7-40aa-4ddd"> |
|
<input type="hidden" id="qualityBanWarningShown" name="qualityBanWarningShown" value="false" data-webtasks-id="ccdcf50c-668c-4008"> |
|
<input type="hidden" name="referrer" value="https://travel.stackexchange.com/?tab=month" data-webtasks-id="a94501de-c8c7-4c67"> |
|
<h2 class="space" id="your-answer-header" data-webtasks-id="a402804c-7532-4565"> |
|
Your Answer |
|
</h2> |
|
|
|
|
|
|
|
|
|
<script data-webtasks-id="b32cd721-2734-42d1"> |
|
StackExchange.ready(function() { |
|
var channelOptions = { |
|
tags: "".split(" "), |
|
id: "273" |
|
}; |
|
initTagRenderer("".split(" "), "".split(" "), channelOptions); |
|
|
|
StackExchange.using("externalEditor", function() { |
|
|
|
if (StackExchange.settings.snippets.snippetsEnabled) { |
|
StackExchange.using("snippets", function() { |
|
createEditor(); |
|
}); |
|
} |
|
else { |
|
createEditor(); |
|
} |
|
}); |
|
|
|
function createEditor() { |
|
StackExchange.prepareEditor({ |
|
useStacksEditor: false, |
|
heartbeatType: 'answer', |
|
autoActivateHeartbeat: false, |
|
convertImagesToLinks: false, |
|
noModals: true, |
|
showLowRepImageUploadWarning: true, |
|
reputationToPostImages: null, |
|
bindNavPrevention: true, |
|
postfix: "", |
|
imageUploader: { |
|
brandingHtml: "Powered by \u003ca href=\"https://imgur.com/\"\u003e\u003csvg class=\"svg-icon\" width=\"50\" height=\"18\" viewBox=\"0 0 50 18\" fill=\"none\" xmlns=\"http://www.w3.org/2000/svg\"\u003e\u003ctitle\u003eImgur Logo\u003c/title\u003e\u003cpath d=\"M46.1709 9.17788C46.1709 8.26454 46.2665 7.94324 47.1084 7.58816C47.4091 7.46349 47.7169 7.36433 48.0099 7.26993C48.9099 6.97997 49.672 6.73443 49.672 5.93063C49.672 5.22043 48.9832 4.61182 48.1414 4.61182C47.4335 4.61182 46.7256 4.91628 46.0943 5.50789C45.7307 4.9328 45.2525 4.66231 44.6595 4.66231C43.6264 4.66231 43.1481 5.28821 43.1481 6.59048V11.9512C43.1481 13.2535 43.6264 13.8962 44.6595 13.8962C45.6924 13.8962 46.1709 13.2535 46.1709 11.9512V9.17788Z\"/\u003e\u003cpath d=\"M32.492 10.1419C32.492 12.6954 34.1182 14.0484 37.0451 14.0484C39.9723 14.0484 41.5985 12.6954 41.5985 10.1419V6.59049C41.5985 5.28821 41.1394 4.66232 40.1061 4.66232C39.0732 4.66232 38.5948 5.28821 38.5948 6.59049V9.60062C38.5948 10.8521 38.2696 11.5455 37.0451 11.5455C35.8209 11.5455 35.4954 10.8521 35.4954 9.60062V6.59049C35.4954 5.28821 35.0173 4.66232 34.0034 4.66232C32.9703 4.66232 32.492 5.28821 32.492 6.59049V10.1419Z\" /\u003e\u003cpath fill-rule=\"evenodd\" clip-rule=\"evenodd\" d=\"M25.6622 17.6335C27.8049 17.6335 29.3739 16.9402 30.2537 15.6379C30.8468 14.7755 30.9615 13.5579 30.9615 11.9512V6.59049C30.9615 5.28821 30.4833 4.66231 29.4502 4.66231C28.9913 4.66231 28.4555 4.94978 28.1109 5.50789C27.499 4.86533 26.7335 4.56087 25.7005 4.56087C23.1369 4.56087 21.0134 6.57349 21.0134 9.27932C21.0134 11.9852 23.003 13.913 25.3754 13.913C26.5612 13.913 27.4607 13.4902 28.1109 12.6616C28.1109 12.7229 28.1161 12.7799 28.121 12.8346C28.1256 12.8854 28.1301 12.9342 28.1301 12.983C28.1301 14.4373 27.2502 15.2321 25.777 15.2321C24.8349 15.2321 24.1352 14.9821 23.5661 14.7787C23.176 14.6393 22.8472 14.5218 22.5437 14.5218C21.7977 14.5218 21.2429 15.0123 21.2429 15.6887C21.2429 16.7375 22.9072 17.6335 25.6622 17.6335ZM24.1317 9.27932C24.1317 7.94324 24.9928 7.09766 26.1024 7.09766C27.2119 7.09766 28.0918 7.94324 28.0918 9.27932C28.0918 10.6321 27.2311 11.5116 26.1024 11.5116C24.9737 11.5116 24.1317 10.6491 24.1317 9.27932Z\"/\u003e\u003cpath d=\"M16.8045 11.9512C16.8045 13.2535 17.2637 13.8962 18.2965 13.8962C19.3298 13.8962 19.8079 13.2535 19.8079 11.9512V8.12928C19.8079 5.82936 18.4879 4.62866 16.4027 4.62866C15.1594 4.62866 14.279 4.98375 13.3609 5.88013C12.653 5.05154 11.6581 4.62866 10.3573 4.62866C9.34336 4.62866 8.57809 4.89931 7.9466 5.5079C7.58314 4.9328 7.10506 4.66232 6.51203 4.66232C5.47873 4.66232 5.00066 5.28821 5.00066 6.59049V11.9512C5.00066 13.2535 5.47873 13.8962 6.51203 13.8962C7.54479 13.8962 8.0232 13.2535 8.0232 11.9512V8.90741C8.0232 7.58817 8.44431 6.91179 9.53458 6.91179C10.5104 6.91179 10.893 7.58817 10.893 8.94108V11.9512C10.893 13.2535 11.3711 13.8962 12.4044 13.8962C13.4375 13.8962 13.9157 13.2535 13.9157 11.9512V8.90741C13.9157 7.58817 14.3365 6.91179 15.4269 6.91179C16.4027 6.91179 16.8045 7.58817 16.8045 8.94108V11.9512Z\"/\u003e\u003cpath d=\"M3.31675 6.59049C3.31675 5.28821 2.83866 4.66232 1.82471 4.66232C0.791758 4.66232 0.313354 5.28821 0.313354 6.59049V11.9512C0.313354 13.2535 0.791758 13.8962 1.82471 13.8962C2.85798 13.8962 3.31675 13.2535 3.31675 11.9512V6.59049Z\" /\u003e\u003cpath d=\"M1.87209 0.400291C0.843612 0.400291 0 1.1159 0 1.98861C0 2.87869 0.822846 3.57676 1.87209 3.57676C2.90056 3.57676 3.7234 2.87869 3.7234 1.98861C3.7234 1.1159 2.90056 0.400291 1.87209 0.400291Z\" fill=\"#1BB76E\"/\u003e\u003c/svg\u003e\u003c/a\u003e", |
|
contentPolicyHtml: "User contributions licensed under \u003ca href=\"https://stackoverflow.com/help/licensing\"\u003eCC BY-SA\u003c/a\u003e \u003ca href=\"https://stackoverflow.com/legal/content-policy\"\u003e(content policy)\u003c/a\u003e", |
|
allowUrls: true |
|
}, |
|
noCode: true, onDemand: false, |
|
discardSelector: ".discard-answer", |
|
enableTables: true, |
|
isStacksEditorPreviewEnabled: false |
|
,immediatelyShowMarkdownHelp:true,userId:136878,enableTables:true |
|
}); |
|
} |
|
}); |
|
</script> |
|
<div id="post-editor" class="post-editor js-post-editor" data-webtasks-id="1aa1b40b-e142-4915"> |
|
|
|
|
|
<div class="ps-relative" data-webtasks-id="3f335e5d-1879-4a1b"> |
|
<div class="wmd-container mb8" data-webtasks-id="9633cb65-17c5-4443"> |
|
<div id="wmd-button-bar" class="wmd-button-bar btr-sm" data-webtasks-id="8c310915-e751-4887"><ul id="wmd-button-row" class="wmd-button-row" data-webtasks-id="ec10cdb0-cf60-4543"><li class="wmd-button" id="wmd-bold-button" title="Strong <strong> Ctrl+B" data-webtasks-id="9e99480d-3055-4f31"><span data-webtasks-id="f37ab6e8-1e1d-4475" style="background-position: 0px 0px;"></span></li><li class="wmd-button" id="wmd-italic-button" title="Emphasis <em> Ctrl+I" data-webtasks-id="d0715f3e-86ac-40a8"><span data-webtasks-id="2082094c-29f4-4353" style="background-position: -20px 0px;"></span></li><li class="wmd-spacer wmd-spacer1" id="wmd-spacer1" data-webtasks-id="103abcb8-6ce1-447d"></li><li class="wmd-button" id="wmd-link-button" title="Hyperlink <a> Ctrl+L" data-webtasks-id="e26ee090-00d5-491c"><span data-webtasks-id="93a694c2-30ac-4b81" style="background-position: -40px 0px;"></span></li><li class="wmd-button" id="wmd-quote-button" title="Blockquote <blockquote> Ctrl+Q" data-webtasks-id="d37d11ee-f4f9-4e21"><span data-webtasks-id="33dd6c18-6df9-4e25" style="background-position: -60px 0px;"></span></li><li class="wmd-button" id="wmd-code-button" title="Preformatted text <pre><code> Ctrl-K" data-webtasks-id="adbbfdfb-0a42-4034"><span data-webtasks-id="32d54dbe-0160-41a1" style="background-position: -80px 0px;"></span></li><li class="wmd-button" id="wmd-image-button" title="Image <img> Ctrl+G" data-webtasks-id="fa9d5fb1-14fb-48fd"><span data-webtasks-id="c558254f-34c2-4830" style="background-position: -100px 0px;"></span></li><li class="wmd-spacer wmd-spacer2" id="wmd-spacer2" data-webtasks-id="65697c6a-ac2a-4235"></li><li class="wmd-button" id="wmd-olist-button" title="Numbered List <ol> Ctrl+O" data-webtasks-id="c5c8ba3a-005c-4e50"><span data-webtasks-id="c8119c45-6d5f-4e35" style="background-position: -120px 0px;"></span></li><li class="wmd-button" id="wmd-ulist-button" title="Bulleted List <ul> Ctrl+U" data-webtasks-id="b35f6372-50ad-45a6"><span data-webtasks-id="800622a1-a787-420d" style="background-position: -140px 0px;"></span></li><li class="wmd-button" id="wmd-heading-button" title="Heading <h1>/<h2> Ctrl+H" data-webtasks-id="50a7baa6-bf30-4415"><span data-webtasks-id="9f9b563c-a250-4fdc" style="background-position: -160px 0px;"></span></li><li class="wmd-button" id="wmd-hr-button" title="Horizontal Rule <hr> Ctrl+R" data-webtasks-id="baa8914b-eb23-4adb"><span data-webtasks-id="292e23df-6261-442d" style="background-position: -180px 0px;"></span></li><li class="wmd-spacer wmd-spacer3" id="wmd-spacer3" data-webtasks-id="1ef8e169-b81e-4930"></li><li class="wmd-button" id="wmd-undo-button" title="Undo - Ctrl+Z" data-webtasks-id="07dd7259-942a-4462"><span data-webtasks-id="7ba4d5b6-4ce1-46f7" style="background-position: -200px -20px;"></span></li><li class="wmd-button" id="wmd-redo-button" title="Redo - Ctrl+Y" data-webtasks-id="c1256f37-25fe-4be9"><span data-webtasks-id="47f7ffe9-4e58-4d1d" style="background-position: -220px -20px;"></span></li><li class="wmd-spacer wmd-spacer-max" data-webtasks-id="fde5598e-94b2-4119"></li><li class="wmd-button wmd-help-button active-help" id="wmd-help-button" title="Markdown Editing Help" data-webtasks-id="5d2d1f98-7748-46d3" style="right: 0px;"><span style="background-position: -240px 0px;" data-webtasks-id="25324c6d-b32b-442d"></span></li></ul></div><div id="mdhelp" class="mdhelp" style="" data-webtasks-id="bde8d889-3aad-4ba4"> |
|
<ul id="mdhelp-tabs" class="mdhelp-tabs" data-webtasks-id="f0e16a5e-a3b0-40c2"> |
|
<li data-tab="mdhelp-links" href="..." class="js-gps-track" data-gps-track="mdhelp.click({ help_type: 0 })" data-buttons="link" data-webtasks-id="f45cce26-39eb-456d">Links</li> |
|
<li data-tab="mdhelp-images" data-gps-track="mdhelp.click({ help_type: 1 })" data-buttons="image" data-webtasks-id="e3930cb6-be9c-4af8">Images</li> |
|
<li data-tab="mdhelp-styles" data-gps-track="mdhelp.click({ help_type: 2 })" data-buttons="bold,italic,heading" data-webtasks-id="84a26886-c1cf-4efb">Styling/Headers</li> |
|
<li data-tab="mdhelp-lists" data-gps-track="mdhelp.click({ help_type: 3 })" data-buttons="olist,ulist" data-webtasks-id="5e7ae648-ffe8-4047">Lists</li> |
|
<li data-tab="mdhelp-blockquotes" data-gps-track="mdhelp.click({ help_type: 4 })" data-buttons="quote" data-webtasks-id="16859c91-172d-456e">Blockquotes</li> |
|
<li data-tab="mdhelp-code" data-gps-track="mdhelp.click({ help_type: 5 })" data-buttons="code" data-webtasks-id="a5090b67-e3e2-4565">Preformatted</li> |
|
<li data-tab="mdhelp-html" data-gps-track="mdhelp.click({ help_type: 6 })" data-webtasks-id="1c0690ed-e1f8-4dfe"> HTML </li> |
|
<li data-tab="mdhelp-tables" data-gps-track="mdhelp.click({ help_type: 9 })" data-webtasks-id="9f2fd7a1-9d5a-409b"> Tables </li> <li class="float-right" data-webtasks-id="d3ba2cbb-cd68-4394"><a href="/editing-help" data-gps-track="mdhelp.click({ help_type: 7 })" target="_blank" data-webtasks-id="d40f4bc0-8759-424f">Advanced help</a></li> |
|
</ul> |
|
|
|
<div class="mdhelp-tab" id="mdhelp-links" data-webtasks-id="7f3020af-c1e6-44d7"> |
|
<div class="mdhelp-tab--content" data-webtasks-id="c130ea06-1d15-433e"> |
|
<p data-webtasks-id="dcb32e24-8209-4408"> |
|
In most cases, a plain URL will be recognized as such and automatically linked: |
|
</p> |
|
<pre data-webtasks-id="2b13e754-772e-4f1c">Visit https://area51.stackexchange.com/ regularly!
|
|
Use angle brackets to force linking: Have you seen <https://superuser.com>? |
|
</pre> |
|
<p data-webtasks-id="f7f78614-8c27-4489"> |
|
To create fancier links, use Markdown: |
|
</p> |
|
<pre data-webtasks-id="52ac642d-ec49-4e8c">Here's [a link](https://www.example.com/)! And a reference-style link to [a panda][1].
|
|
References don't have to be [numbers][question].
|
|
|
|
[1]: https://notfound.stackexchange.com/
|
|
[question]: https://english.stackexchange.com/questions/11481 |
|
</pre> |
|
<p data-webtasks-id="da1dc1ef-57ae-4bd1">You can add tooltips to links:</p> |
|
<pre data-webtasks-id="66440078-3003-41f5">Click [here](https://diy.stackexchange.com<span class="spaces" data-webtasks-id="59191e8f-7d9a-4ca2"> </span>"this text appears when you mouse over")! |
|
This works with [reference links][blog] as well. |
|
|
|
[blog]: https://stackoverflow.blog/<span class="spaces" data-webtasks-id="7774585a-1ec1-4000"> </span>"click here for updates" |
|
</pre> |
|
</div> |
|
</div> |
|
|
|
<div class="mdhelp-tab" id="mdhelp-images" data-webtasks-id="8b70d909-e569-4a16"> |
|
<div class="mdhelp-tab--content" data-webtasks-id="c22f6b0d-3bc8-4578"> |
|
<p data-webtasks-id="99d3c628-d970-4f92">Images are exactly like links, but they have an exclamation point in front of them:</p> |
|
<pre data-webtasks-id="8e4f687c-b8d5-4ce3">![a busy cat](https://cdn.sstatic.net/Sites/stackoverflow/Img/error-lolcat-problemz.jpg)
|
|
![two muppets][1]
|
|
|
|
[1]: https://i.imgur.com/I5DFV.jpg "tooltip" |
|
</pre> |
|
<p data-webtasks-id="7c8df19a-c42d-450c"> |
|
The word in square brackets is the alt text, which gets displayed if the browser
|
|
can't show the image. Be sure to include meaningful alt text for screen-reading
|
|
software. |
|
</p> |
|
|
|
</div> |
|
</div> |
|
|
|
<div class="mdhelp-tab" id="mdhelp-styles" data-webtasks-id="799d8681-db95-4cde"> |
|
<div class="mdhelp-tab--content" data-webtasks-id="5bf659c1-98cb-4d6c"> |
|
<div class="col1" data-webtasks-id="79b64271-ac53-4f7e"> |
|
<p data-webtasks-id="4506a7ac-b6e0-48c6">Be sure to use text styling sparingly; only where it helps readability.</p> |
|
<pre data-webtasks-id="35f30ef3-63ab-4458">*This is italicized*, and so
|
|
is _this_.
|
|
|
|
**This is bold**, just like __this__.
|
|
|
|
You can ***combine*** them
|
|
if you ___really have to___. |
|
</pre> |
|
|
|
</div> |
|
<div class="col2" data-webtasks-id="21106d91-f571-454e"> |
|
<p data-webtasks-id="f638a97b-cba1-4f54"> |
|
To break your text into sections, you can use headers: |
|
</p> |
|
<pre data-webtasks-id="d45294fc-7515-41c8">A Large Header
|
|
==============
|
|
|
|
Smaller Subheader
|
|
----------------- |
|
</pre> |
|
<p data-webtasks-id="29574698-d25f-4329"> |
|
Use hash marks if you need several levels of headers: |
|
</p> |
|
<pre data-webtasks-id="d2c445b5-81ac-4b2c"># Header 1 #
|
|
## Header 2 ##
|
|
### Header 3 ### |
|
</pre> |
|
</div> |
|
</div> |
|
</div> |
|
|
|
<div class="mdhelp-tab" id="mdhelp-lists" data-webtasks-id="b946ccb1-d1a0-46d7"> |
|
<div class="mdhelp-tab--content" data-webtasks-id="e025526b-6a82-4017"> |
|
<p data-webtasks-id="15e1b1ad-a8eb-495c">Both bulleted and numbered lists are possible:</p> |
|
<div class="col1" data-webtasks-id="325904df-41a6-45a5"> |
|
<pre data-webtasks-id="a743cae9-cc4e-4bfb">-<span class="spaces" data-webtasks-id="343257fc-78d2-4eff"> </span>Use a minus sign for a bullet |
|
+<span class="spaces" data-webtasks-id="dc5b097b-343d-4164"> </span>Or plus sign |
|
*<span class="spaces" data-webtasks-id="9254271c-a580-433e"> </span>Or an asterisk |
|
|
|
1.<span class="spaces" data-webtasks-id="26ed80f7-814a-4167"> </span>Numbered lists are easy |
|
2.<span class="spaces" data-webtasks-id="0fbd0692-2fd6-4dd4"> </span>Markdown keeps track of |
|
the numbers for you |
|
7.<span class="spaces" data-webtasks-id="0b11b3a0-94f3-4f50"> </span>So this will be item 3. |
|
</pre> |
|
</div> |
|
<div class="col2" data-webtasks-id="b045ae90-6e64-478c"> |
|
<pre data-webtasks-id="41e9f6db-e219-4408">1.<span class="spaces" data-webtasks-id="d1b701f6-32f2-423d"> </span>Lists in a list item: |
|
<span class="spaces" data-webtasks-id="1bca46bf-4431-4022"> </span>-<span class="spaces" data-webtasks-id="afa860ad-fcd0-47c7"> </span>Indented four spaces. |
|
<span class="spaces" data-webtasks-id="d5f88548-1ba2-4176"> </span>*<span class="spaces" data-webtasks-id="3d039c5b-3e16-4022"> </span>indented eight spaces. |
|
<span class="spaces" data-webtasks-id="92b62d3d-05f1-42e9"> </span>-<span class="spaces" data-webtasks-id="db354c16-84b0-47fe"> </span>Four spaces again. |
|
2.<span class="spaces" data-webtasks-id="3b810413-f99b-4791"> </span>You can have multiple |
|
<span class="spaces" data-webtasks-id="091405fb-c050-4d2b"> </span>paragraphs in a list items. |
|
<span class="spaces" data-webtasks-id="e1b6ad30-254c-4edd"> </span> |
|
<span class="spaces" data-webtasks-id="0f8d83fa-be04-4e13"> </span>Just be sure to indent. |
|
</pre> |
|
</div> |
|
</div> |
|
</div> |
|
|
|
<div class="mdhelp-tab" id="mdhelp-blockquotes" data-webtasks-id="d4067214-015a-4f33"> |
|
<div class="mdhelp-tab--content" data-webtasks-id="54846b57-bf53-452d"> |
|
<div class="col1" data-webtasks-id="c396da9a-9059-41f4"> |
|
<pre data-webtasks-id="4afab239-b21a-4abd">> Create a blockquote by
|
|
> prepending “>” to each line.
|
|
>
|
|
> Other formatting also works here, e.g.
|
|
>
|
|
> 1. Lists or
|
|
> 2. Headings:
|
|
>
|
|
> ## Quoted Heading ## |
|
</pre> |
|
</div> |
|
<div class="col2" data-webtasks-id="0db2148c-5cd7-4435"> |
|
<p data-webtasks-id="0501bb5e-ac77-4583"> |
|
You can even put blockquotes in blockquotes: |
|
</p> |
|
<pre data-webtasks-id="20059b9a-4b9f-48cc">> A standard blockquote is indented
|
|
> > A nested blockquote is indented more
|
|
> > > > You can nest to any depth. |
|
</pre> |
|
</div> |
|
</div> |
|
</div> |
|
|
|
<div class="mdhelp-tab" id="mdhelp-code" data-webtasks-id="fb534151-7590-4bac"> |
|
<div class="mdhelp-tab--content" data-webtasks-id="ae43167a-c140-4818"> |
|
<p data-webtasks-id="5c1d392a-37eb-4879"> |
|
To create preformatted text, indent by four spaces or surround with groups of backticks: |
|
</p> |
|
<pre data-webtasks-id="f7deb35f-09e8-4b79"><span class="spaces" data-webtasks-id="4b4151bc-d16c-4e23"> </span>This will be displayed in a monospaced font. The first four spaces |
|
<span class="spaces" data-webtasks-id="1c7a0b13-0311-405d"> </span>will be stripped off, but all other whitespace will be preserved. |
|
|
|
``` |
|
Markdown and HTML are turned off in code blocks: |
|
<i>This is not italic</i>, and [this is not a link](https://example.com) |
|
```</pre> |
|
<p data-webtasks-id="df7c07f1-40b0-42b3"> |
|
To create not a block, but an inline code span, use backticks: |
|
</p> |
|
<pre data-webtasks-id="405ca670-f34e-4d07">The `$` character is just a shortcut for `window.jQuery`. |
|
</pre> |
|
<p data-webtasks-id="2c8168c2-d1cf-476e"> |
|
If you want to have a preformatted block within a list, indent by eight spaces: |
|
</p> |
|
<pre data-webtasks-id="2c2a4ba8-d60f-4e9c">1. This is normal text. |
|
2. So is this, but now follows a code block: |
|
<span class="spaces" data-webtasks-id="c5e32d79-63bb-46ac"> </span> |
|
<span class="spaces" data-webtasks-id="d4773e56-4026-4787"> </span>Skip a line and indent eight spaces. |
|
<span class="spaces" data-webtasks-id="8326c52b-304f-41ce"> </span>That's four spaces for the list |
|
<span class="spaces" data-webtasks-id="1173093a-eb2b-4d03"> </span>and four to trigger the code block. |
|
</pre> |
|
</div> |
|
</div> |
|
|
|
<div class="mdhelp-tab" id="mdhelp-html" data-webtasks-id="cd265aea-321b-473d"> |
|
<div class="mdhelp-tab--content" data-webtasks-id="6250a66e-5b62-4a8d"> |
|
<p data-webtasks-id="94725868-a231-461f"> |
|
If you need to do something that Markdown can't handle, use HTML. Note that <a href="https://meta.stackexchange.com/questions/1777/what-html-tags-are-allowed" target="_blank" data-webtasks-id="ecd655be-09a9-4851">we only support a very strict subset of HTML!</a> |
|
</p> |
|
<pre data-webtasks-id="343d9384-f3ee-4a39">Strikethrough humor is <strike>funny</strike>. |
|
</pre> |
|
<p data-webtasks-id="c218e9f5-0515-4b45"> |
|
Markdown is smart enough not to mangle your span-level HTML: |
|
</p> |
|
<pre data-webtasks-id="65565589-ab1b-4709"><b>Markdown works *fine* in here.</b> |
|
</pre> |
|
<p data-webtasks-id="6c35593a-d575-4a42"> |
|
Block-level HTML elements have a few restrictions: |
|
</p> |
|
<ol data-webtasks-id="642f128a-2b55-47e1"> |
|
<li data-webtasks-id="72e4f0ee-0f48-49fd">They must be separated from surrounding text by blank lines.</li> |
|
<li data-webtasks-id="99f52e41-f344-4da1">The begin and end tags of the outermost block element must not be indented.</li> |
|
<li data-webtasks-id="60b2960d-7e32-48a0">Markdown can't be used within HTML blocks.</li> |
|
</ol> |
|
<br data-webtasks-id="ed723204-131d-4ad3"> |
|
<pre data-webtasks-id="9ea9e010-a54f-494d"><pre> |
|
You can <em>not</em> use Markdown in here. |
|
</pre> |
|
</pre> |
|
</div> |
|
</div> |
|
|
|
<div class="mdhelp-tab" id="mdhelp-tables" data-webtasks-id="b8904b73-fe93-4353"> |
|
<div class="mdhelp-tab--content" data-webtasks-id="49922570-37c2-42e3"> |
|
<p data-webtasks-id="473e7815-dac9-4309"> |
|
You can create tables using the <a href="https://github.github.com/gfm/#tables-extension-" data-webtasks-id="05224fda-2ad7-4269">GitHub-flavored markdown format</a>. |
|
</p> |
|
<pre data-webtasks-id="6061ef0d-61fe-40ba">| A header | Another header |
|
|
| -------- | -------------- |
|
|
| First | row |
|
|
| Second | row | |
|
</pre> |
|
|
|
<ul class="pl24 mt8" data-webtasks-id="d133deeb-9c38-4f22"> |
|
<li data-webtasks-id="f5ef9723-0c71-47d8">A header row is required and must be followed by a separator row with the same number of cells</li> |
|
<li data-webtasks-id="2c36f556-c8cc-4d01">Cells are separated by a pipe (<code data-webtasks-id="33f4186d-00a2-40e3">|</code>) symbol</li> |
|
</ul> |
|
|
|
<p data-webtasks-id="1479d984-a886-479b"> |
|
Set the <b data-webtasks-id="e55ac369-17bf-48e2">alignment</b> of a table column by placing a <code data-webtasks-id="a6e19898-a131-49bd">:</code> on the left, right, or both sides of a separator in the separator line. |
|
</p> |
|
<pre data-webtasks-id="98f7407a-07ca-4c83">| left | center | right |
|
|
|:---- |:------:| -----:|
|
|
| One | Two | Three | |
|
</pre> |
|
</div> |
|
</div> |
|
<div class="clear-both" data-webtasks-id="dafed241-1f88-465d"></div> |
|
</div> |
|
<div class="js-stacks-validation" data-webtasks-id="9de8f9f9-ff29-413e"> |
|
<div class="ps-relative" data-webtasks-id="9275830b-ac94-4a70"> |
|
<textarea id="wmd-input" name="post-text" class="wmd-input s-input bar0 js-post-body-field processed" data-editor-type="wmd" data-post-type-id="2" cols="92" rows="15" aria-labelledby="your-answer-header" tabindex="101" data-min-length="" data-webtasks-id="d43be7fa-4575-48c3"></textarea> |
|
<div class="grippie bbr-sm" data-webtasks-id="3b51bbfe-10a7-4ecc" style="margin-right: 0px;"></div></div> |
|
<div class="s-input-message mt4 d-none js-stacks-validation-message" data-webtasks-id="10d26185-95ff-4704"></div> |
|
</div> |
|
</div> |
|
</div> |
|
|
|
<aside class="d-flex ai-start jc-space-between js-answer-help s-notice s-notice__warning pb0 pr4 pt4 mb8 d-none" role="status" aria-hidden="true" data-webtasks-id="2840cc59-f272-46b2"> |
|
<div class="flex--item pt8" data-webtasks-id="bea4fe25-9b4d-489d"> |
|
<p data-webtasks-id="af049634-960a-451d">Thanks for contributing an answer to Travel Stack Exchange!</p><ul data-webtasks-id="5f1e6a89-4f11-4411"><li data-webtasks-id="1d0cae81-2316-47a8">Please be sure to <em data-webtasks-id="5067dd82-db8f-427d">answer the question</em>. Provide details and share your research!</li></ul><p data-webtasks-id="775afc98-64b5-494d">But <em data-webtasks-id="595019ce-808d-4165">avoid</em> …</p><ul data-webtasks-id="b90804be-0b55-4d5d"><li data-webtasks-id="c092ea63-ebc3-4aed">Asking for help, clarification, or responding to other answers.</li><li data-webtasks-id="b1dfa09c-d120-441d">Making statements based on opinion; back them up with references or personal experience.</li></ul><p data-webtasks-id="8f45de8c-5dd3-4c25">To learn more, see our <a href="/help/how-to-answer" data-webtasks-id="d6d0c90f-9976-421c">tips on writing great answers</a>.</p> |
|
</div> |
|
<button class="flex--item js-answer-help-close-btn s-btn s-btn__muted fc-dark" data-webtasks-id="0c057325-d6e9-45c4"> |
|
<svg aria-hidden="true" class="svg-icon iconClear" width="18" height="18" viewBox="0 0 18 18" data-webtasks-id="42dd5519-5793-43a7"><path d="M15 4.41 13.59 3 9 7.59 4.41 3 3 4.41 7.59 9 3 13.59 4.41 15 9 10.41 13.59 15 15 13.59 10.41 9 15 4.41Z" data-webtasks-id="c0de673f-03db-434a"></path></svg> |
|
</button> |
|
</aside> |
|
|
|
<aside class="d-flex ai-start jc-space-between js-answer-ban s-notice s-notice__danger pb0 pr4 pt4 mb8 d-none" role="status" aria-hidden="true" data-webtasks-id="60738ae6-73d8-4e99"> |
|
<div class="flex--item pt8" data-webtasks-id="c021c65f-85a0-4982"> |
|
<p data-webtasks-id="b1929b60-f477-487b">Some of your past answers have not been well-received, and you're in danger of being <a href="/help/answer-bans" data-webtasks-id="957f2ba1-8a10-41a8">blocked from answering</a>.</p><p data-webtasks-id="9e21fccd-51ab-4a06">Please pay close attention to the following guidance:</p><ul data-webtasks-id="efc5943c-690a-47a8"><li data-webtasks-id="ed89f2d2-92ed-4497">Please be sure to <em data-webtasks-id="63adf675-66ed-45a8">answer the question</em>. Provide details and share your research!</li></ul><p data-webtasks-id="68b15468-54ad-499d">But <em data-webtasks-id="2f1847da-3fe4-4d69">avoid</em> …</p><ul data-webtasks-id="a37092d2-6b83-457e"><li data-webtasks-id="f0aa482b-69f9-4230">Asking for help, clarification, or responding to other answers.</li><li data-webtasks-id="01cdd44c-02d3-4389">Making statements based on opinion; back them up with references or personal experience.</li></ul><p data-webtasks-id="7216f452-1f1d-4d15">To learn more, see our <a href="/help/how-to-answer" data-webtasks-id="b7f2ddea-b39f-441f">tips on writing great answers</a>.</p> |
|
</div> |
|
<button class="flex--item js-answer-ban-close-btn s-btn s-btn__muted fc-dark" data-webtasks-id="0f5f4b53-014b-4d5e"> |
|
<svg aria-hidden="true" class="svg-icon iconClear" width="18" height="18" viewBox="0 0 18 18" data-webtasks-id="43334b31-69ae-4249"><path d="M15 4.41 13.59 3 9 7.59 4.41 3 3 4.41 7.59 9 3 13.59 4.41 15 9 10.41 13.59 15 15 13.59 10.41 9 15 4.41Z" data-webtasks-id="9e628f59-3a67-4d35"></path></svg> |
|
</button> |
|
</aside> |
|
|
|
|
|
|
|
<div data-webtasks-id="ac9b804a-2b37-4212"> |
|
<div id="draft-saved" class="fc-success h24" style="display:none;" data-webtasks-id="8b5531f3-b2fa-47b7">Draft saved</div> |
|
<div id="draft-discarded" class="fc-error h24" style="display:none;" data-webtasks-id="1b0e6d7b-2bf0-4f21">Draft discarded</div> |
|
</div> |
|
|
|
|
|
<div id="wmd-preview" class="s-prose mb16 wmd-preview js-wmd-preview" data-webtasks-id="aa542417-85a9-4c12"></div> |
|
<div data-webtasks-id="941234de-b53a-4954"></div> |
|
|
|
<div class="edit-block" data-webtasks-id="80a77311-f95d-47f0"> |
|
<input id="fkey" name="fkey" type="hidden" value="fe76f3281bad2063ad30f357d65244f2508ca27d494a61fe42e4d6aea23fa56d" data-webtasks-id="403c2cf3-74a4-4661"> |
|
<input id="author" name="author" type="text" data-webtasks-id="1b369fb2-f12f-4695"> |
|
</div> |
|
|
|
</div> |
|
|
|
|
|
<div class="ps-relative" data-webtasks-id="1a6a076a-e770-4e99"> |
|
|
|
</div> |
|
|
|
<div class="form-submit clear-both d-flex gsx gs4" data-webtasks-id="51652857-fb6f-4f35"> |
|
<button id="submit-button" class="flex--item s-btn s-btn__primary s-btn__icon" type="submit" tabindex="120" autocomplete="off" data-webtasks-id="11c9d47e-9e83-4db5"> |
|
Post Your Answer |
|
</button> |
|
<button class="flex--item s-btn s-btn__danger discard-answer dno" data-webtasks-id="e81aeb80-7129-40c3"> |
|
Discard |
|
</button> |
|
</div> |
|
<div class="js-general-error general-error clear-both d-none" aria-live="polite" data-webtasks-id="a4445bc3-e1c7-4f5d"></div> |
|
</form> |
|
|
|
|
|
<h2 class="bottom-notice" data-loc="1" data-webtasks-id="ac4897e2-e3e7-4bac"> |
|
<div data-webtasks-id="9f21e147-315d-416b"> |
|
Not the answer you're looking for? Browse other questions tagged <ul class="ml0 list-ls-none js-post-tag-list-wrapper d-inline" data-webtasks-id="a508e3d3-9d5a-4c02"><li class="d-inline mr4 js-post-tag-list-item" data-webtasks-id="cfed2636-4a2b-4193"><a href="/questions/tagged/where-on-earth" class="post-tag" title="show questions tagged 'where-on-earth'" aria-label="show questions tagged 'where-on-earth'" rel="tag" aria-labelledby="tag-where-on-earth-tooltip-container" data-webtasks-id="4e007b3e-9956-4034">where-on-earth</a></li><li class="d-inline mr4 js-post-tag-list-item" data-webtasks-id="2bad993d-62d0-4885"><a href="/questions/tagged/bavaria" class="post-tag" title="show questions tagged 'bavaria'" aria-label="show questions tagged 'bavaria'" rel="tag" aria-labelledby="tag-bavaria-tooltip-container" data-webtasks-id="f6c6457e-2779-48bc">bavaria</a></li></ul> or <a href="/questions/ask" data-webtasks-id="107730a4-940a-4f0e">ask your own question</a>. </div> |
|
</h2> |
|
</div> |
|
</div> |
|
|
|
|
|
<div id="sidebar" class="show-votes" role="complementary" aria-label="sidebar" data-webtasks-id="89bb0998-7a09-4651"> |
|
|
|
|
|
|
|
|
|
<div class="s-sidebarwidget s-sidebarwidget__yellow s-anchors s-anchors__grayscale mb16" data-tracker="cb=1" data-webtasks-id="f9fcddcc-69bf-4bf0"> |
|
<ul class="d-block p0 m0" data-webtasks-id="12c87f83-121f-404b"> |
|
<li class="s-sidebarwidget--header s-sidebarwidget__small-bold-text d-flex fc-black-600 d:fc-black-900 bb bbw1" data-webtasks-id="74b462b3-8cc4-440c"> |
|
Featured on Meta |
|
</li> |
|
<li class="s-sidebarwidget--item d-flex px16" data-webtasks-id="dd81a9b3-7f6b-41ba"> |
|
<div class="flex--item1 fl-shrink0" data-webtasks-id="b7b10efc-8bca-4efc"> |
|
<div class="favicon favicon-stackexchangemeta" title="Meta Stack Exchange" data-webtasks-id="c8a584c3-c31e-43ea"></div> </div> |
|
<div class="flex--item wmn0 ow-break-word" data-webtasks-id="d27b24d2-d4b0-43cb"> |
|
<a href="https://meta.stackexchange.com/questions/390463/starting-the-prompt-design-site-a-new-home-in-our-stack-exchange-neighborhood?cb=1" class="js-gps-track" data-ga="["community bulletin board","Featured on Meta","https://meta.stackexchange.com/questions/390463/starting-the-prompt-design-site-a-new-home-in-our-stack-exchange-neighborhood",null,null]" data-gps-track="communitybulletin.click({ priority: 3, position: 0 })" data-webtasks-id="2478952a-6798-44f6">Starting the Prompt Design Site: A New Home in our Stack Exchange Neighborhood</a> |
|
</div> |
|
</li> |
|
<li class="s-sidebarwidget--item d-flex px16" data-webtasks-id="80467629-d5fc-4b99"> |
|
<div class="flex--item1 fl-shrink0" data-webtasks-id="d7782bf3-0e2c-4c9e"> |
|
<div class="favicon favicon-stackexchangemeta" title="Meta Stack Exchange" data-webtasks-id="861d3c7b-ae27-4fe5"></div> </div> |
|
<div class="flex--item wmn0 ow-break-word" data-webtasks-id="bb15f76a-b3f8-44ea"> |
|
<a href="https://meta.stackexchange.com/questions/390761/colors-update-a-more-detailed-look?cb=1" class="js-gps-track" data-ga="["community bulletin board","Featured on Meta","https://meta.stackexchange.com/questions/390761/colors-update-a-more-detailed-look",null,null]" data-gps-track="communitybulletin.click({ priority: 3, position: 1 })" data-webtasks-id="046e9581-c309-40b4">Colors update: A more detailed look</a> |
|
</div> |
|
</li> |
|
<li class="s-sidebarwidget--item d-flex px16" data-webtasks-id="f4109df2-3e14-4f67"> |
|
<div class="flex--item1 fl-shrink0" data-webtasks-id="6441fe0e-bf34-46cb"> |
|
<div class="favicon favicon-travelmeta" title="Travel Meta Stack Exchange" data-webtasks-id="847bb0a3-c945-4c7b"></div> </div> |
|
<div class="flex--item wmn0 ow-break-word" data-webtasks-id="685f7f65-dcbf-42ad"> |
|
<a href="https://travel.meta.stackexchange.com/questions/8524/july-2023-photo-competition-theme-of-pets-traveling?cb=1" class="js-gps-track" data-ga="["community bulletin board","Featured on Meta","https://travel.meta.stackexchange.com/questions/8524/july-2023-photo-competition-theme-of-pets-traveling",null,null]" data-gps-track="communitybulletin.click({ priority: 6, position: 2 })" data-webtasks-id="e0f0c071-e240-4bc3">July 2023 photo competition, theme of 'pets traveling'</a> |
|
</div> |
|
</li> |
|
</ul> |
|
</div> |
|
|
|
|
|
<div class="js-zone-container zone-container-sidebar" data-webtasks-id="6cee30bd-2c87-4e51"> |
|
<div id="dfp-tsb" class="everyonelovesstackoverflow everyoneloves__top-sidebar" data-dfp-zone="true" data-webtasks-id="a580bbf5-1e17-4707" style="min-height: auto; height: auto; display: none;" data-google-query-id="CJvU1t7Eg4ADFcvkcwEdVrkCUQ"><div id="google_ads_iframe_/248424177/travel.stackexchange.com/sb/question-pages_0__container__" data-webtasks-id="2e57bd6c-4d16-4cf2" style="border: 0pt none; width: 300px; height: 0px;"></div></div> |
|
<div class="js-report-ad-button-container " style="width: 300px" data-webtasks-id="05c51772-8717-4f23"></div> |
|
</div> |
|
|
|
|
|
|
|
|
|
|
|
<div class="module sidebar-related" data-webtasks-id="b5d32000-9eb3-47ed"> |
|
<h4 id="h-related" data-webtasks-id="01f9cb4c-90b4-4adc">Related</h4> |
|
<div class="related js-gps-related-questions" data-tracker="rq=1" data-webtasks-id="c6aff5b6-b481-44f2"> |
|
<div class="spacer" data-question-id="18524" data-webtasks-id="26df641f-bcd9-40ba"> |
|
<a href="https://travel.stackexchange.com/q/18524?rq=1" title="Question score (upvotes - downvotes)" data-webtasks-id="9691a120-9326-4e16"> |
|
<div class="answer-votes answered-accepted default" data-webtasks-id="3636a254-7b3f-41e9">5</div> |
|
</a> |
|
<a href="https://travel.stackexchange.com/questions/18524/which-place-in-middle-east-are-these-photos-taken-from?rq=1" class="question-hyperlink" data-webtasks-id="7d626d44-aaa6-4d27">Which place in Middle East are these photos taken from?</a> |
|
</div> |
|
<div class="spacer" data-question-id="60338" data-webtasks-id="e2b8ad29-9e35-423c"> |
|
<a href="https://travel.stackexchange.com/q/60338?rq=1" title="Question score (upvotes - downvotes)" data-webtasks-id="5fc3f90f-ef0e-452b"> |
|
<div class="answer-votes default" data-webtasks-id="8060ff7f-42b1-4dc6">23</div> |
|
</a> |
|
<a href="https://travel.stackexchange.com/questions/60338/where-are-these-old-yugoslavian-monuments?rq=1" class="question-hyperlink" data-webtasks-id="ab6a96ba-a3e7-405d">Where are these old Yugoslavian monuments?</a> |
|
</div> |
|
<div class="spacer" data-question-id="60782" data-webtasks-id="e4a8578a-5202-4c9b"> |
|
<a href="https://travel.stackexchange.com/q/60782?rq=1" title="Question score (upvotes - downvotes)" data-webtasks-id="d51f771f-6044-472a"> |
|
<div class="answer-votes answered-accepted default" data-webtasks-id="82a3a61d-3acd-43e2">18</div> |
|
</a> |
|
<a href="https://travel.stackexchange.com/questions/60782/where-are-these-unusual-rock-formations-rising-from-the-water?rq=1" class="question-hyperlink" data-webtasks-id="0678fede-4061-4f85">Where are these unusual rock formations rising from the water?</a> |
|
</div> |
|
<div class="spacer" data-question-id="79737" data-webtasks-id="26096afd-b644-4e1a"> |
|
<a href="https://travel.stackexchange.com/q/79737?rq=1" title="Question score (upvotes - downvotes)" data-webtasks-id="1b323df5-f99d-43ed"> |
|
<div class="answer-votes answered-accepted default" data-webtasks-id="3891956e-f5ec-4a02">12</div> |
|
</a> |
|
<a href="https://travel.stackexchange.com/questions/79737/what-city-are-these-buildings-in?rq=1" class="question-hyperlink" data-webtasks-id="397fb688-215c-48fd">What city are these buildings in?</a> |
|
</div> |
|
<div class="spacer" data-question-id="88049" data-webtasks-id="7a4de436-2728-4d22"> |
|
<a href="https://travel.stackexchange.com/q/88049?rq=1" title="Question score (upvotes - downvotes)" data-webtasks-id="174fe3e9-ae23-444c"> |
|
<div class="answer-votes default" data-webtasks-id="da330651-8c8f-4675">15</div> |
|
</a> |
|
<a href="https://travel.stackexchange.com/questions/88049/where-in-norway-are-these-massive-boulders?rq=1" class="question-hyperlink" data-webtasks-id="33070127-4a1d-402d">Where in Norway are these massive boulders?</a> |
|
</div> |
|
<div class="spacer" data-question-id="169220" data-webtasks-id="8d7e54ec-c4a3-441f"> |
|
<a href="https://travel.stackexchange.com/q/169220?rq=1" title="Question score (upvotes - downvotes)" data-webtasks-id="90249acb-936d-4b3a"> |
|
<div class="answer-votes answered-accepted default" data-webtasks-id="2c9091ff-c7db-49c5">11</div> |
|
</a> |
|
<a href="https://travel.stackexchange.com/questions/169220/where-are-these-cliffs?rq=1" class="question-hyperlink" data-webtasks-id="953f2fdc-676d-4bc7">Where are these cliffs?</a> |
|
</div> |
|
</div> |
|
</div> |
|
<script type="text/javascript" data-webtasks-id="c7c0046f-b281-4ac6"> |
|
$(document).ready(function() { |
|
$(".js-gps-related-questions .spacer").click(function () { |
|
fireRelatedEvent($(this).index() + 1, $(this).data('question-id')); |
|
}); |
|
|
|
function fireRelatedEvent(position, questionId) { |
|
StackExchange.using("gps", function() { |
|
StackExchange.gps.track('related_questions.click', |
|
{ |
|
position: position, |
|
originQuestionId: 181865, |
|
relatedQuestionId: +questionId, |
|
location: 'sidebar', |
|
source: 'Baseline' |
|
}); |
|
}); |
|
} |
|
}); |
|
</script> |
|
|
|
|
|
|
|
<div id="hot-network-questions" class="module tex2jax_ignore" data-webtasks-id="ff1d3379-6b4c-4586"> |
|
<h4 data-webtasks-id="e4e032d1-3719-4003"> |
|
<a href="https://stackexchange.com/questions?tab=hot" class="js-gps-track s-link s-link__inherit" data-gps-track="posts_hot_network.click({ item_type:1, location:11 })" data-webtasks-id="926cbefc-b189-4fb6"> |
|
Hot Network Questions |
|
</a> |
|
</h4> |
|
<ul data-webtasks-id="443beb31-8f25-40db"> |
|
<li data-webtasks-id="618b5321-10f4-4afc"> |
|
<div class="favicon favicon-movies" title="Movies & TV Stack Exchange" data-webtasks-id="987424a9-474f-430a"></div><a href="https://movies.stackexchange.com/questions/120450/why-did-indiana-jones-contradict-himself" class="js-gps-track question-hyperlink mb0" data-gps-track="site.switch({ item_type:11, target_site:367 }); posts_hot_network.click({ item_type:2, location:11 })" data-webtasks-id="72125375-ed19-4d7f"> |
|
Why did Indiana Jones contradict himself? |
|
</a> |
|
|
|
</li> |
|
<li data-webtasks-id="14e9afa4-529f-4027"> |
|
<div class="favicon favicon-latin" title="Latin Language Stack Exchange" data-webtasks-id="5f34030a-76b8-405d"></div><a href="https://latin.stackexchange.com/questions/21225/greek-datives-of-agent-in-latin-classical-prose" class="js-gps-track question-hyperlink mb0" data-gps-track="site.switch({ item_type:11, target_site:644 }); posts_hot_network.click({ item_type:2, location:11 })" data-webtasks-id="1df5118b-d685-4534"> |
|
Greek "datives of agent" in Latin classical prose? |
|
</a> |
|
|
|
</li> |
|
<li data-webtasks-id="286a4262-4f84-4640"> |
|
<div class="favicon favicon-stackoverflow" title="Stack Overflow" data-webtasks-id="cd53df4b-c280-40a7"></div><a href="https://stackoverflow.com/questions/76640378/python-zip-magic-for-classes-instead-of-tuples" class="js-gps-track question-hyperlink mb0" data-gps-track="site.switch({ item_type:11, target_site:1 }); posts_hot_network.click({ item_type:2, location:11 })" data-webtasks-id="76073324-c923-4702"> |
|
Python zip magic for classes instead of tuples |
|
</a> |
|
|
|
</li> |
|
<li data-webtasks-id="e079fb03-3836-4731"> |
|
<div class="favicon favicon-quant" title="Quantitative Finance Stack Exchange" data-webtasks-id="984fc4f1-3e56-4f8c"></div><a href="https://quant.stackexchange.com/questions/76070/different-maturities-but-same-tenor-to-obtain-the-yield" class="js-gps-track question-hyperlink mb0" data-gps-track="site.switch({ item_type:11, target_site:204 }); posts_hot_network.click({ item_type:2, location:11 })" data-webtasks-id="5160ebba-9d9c-4593"> |
|
Different maturities but same tenor to obtain the yield |
|
</a> |
|
|
|
</li> |
|
<li data-webtasks-id="022bb296-0ce2-472d"> |
|
<div class="favicon favicon-physics" title="Physics Stack Exchange" data-webtasks-id="9acc101f-42df-4c98"></div><a href="https://physics.stackexchange.com/questions/771285/relativistic-time-dilation-and-the-biological-process-of-aging" class="js-gps-track question-hyperlink mb0" data-gps-track="site.switch({ item_type:11, target_site:151 }); posts_hot_network.click({ item_type:2, location:11 })" data-webtasks-id="811abf5d-0857-4a7c"> |
|
Relativistic time dilation and the biological process of aging |
|
</a> |
|
|
|
</li> |
|
<li class="js-hidden" style="" data-webtasks-id="04b0282e-9480-47a4"> |
|
<div class="favicon favicon-puzzling" title="Puzzling Stack Exchange" data-webtasks-id="6cb594bc-8d52-4f08"></div><a href="https://puzzling.stackexchange.com/questions/121485/what-is-this-g-d-f" class="js-gps-track question-hyperlink mb0" data-gps-track="site.switch({ item_type:11, target_site:559 }); posts_hot_network.click({ item_type:2, location:11 })" data-webtasks-id="babf11f0-aff1-4eb0"> |
|
What is this: G D F? |
|
</a> |
|
|
|
</li> |
|
<li class="js-hidden" style="" data-webtasks-id="c9ed9425-9e27-4c93"> |
|
<div class="favicon favicon-tex" title="TeX - LaTeX Stack Exchange" data-webtasks-id="aff6a3ec-e0fc-48be"></div><a href="https://tex.stackexchange.com/questions/690685/sloped-label-on-arrows-in-tikz-not-parallel-to-its-arrow" class="js-gps-track question-hyperlink mb0" data-gps-track="site.switch({ item_type:11, target_site:85 }); posts_hot_network.click({ item_type:2, location:11 })" data-webtasks-id="02285cc8-3f15-4da9"> |
|
Sloped label on arrows in Tikz not parallel to its arrow |
|
</a> |
|
|
|
</li> |
|
<li class="js-hidden" style="" data-webtasks-id="4b946d7d-1850-41c2"> |
|
<div class="favicon favicon-latin" title="Latin Language Stack Exchange" data-webtasks-id="ea191115-0ad2-4f86"></div><a href="https://latin.stackexchange.com/questions/21223/minae-quibus-usque-ad-mortem-timeri-parum-est" class="js-gps-track question-hyperlink mb0" data-gps-track="site.switch({ item_type:11, target_site:644 }); posts_hot_network.click({ item_type:2, location:11 })" data-webtasks-id="39122b79-f04b-4332"> |
|
"minae quibus usque ad mortem timeri parum est." |
|
</a> |
|
|
|
</li> |
|
<li class="js-hidden" style="" data-webtasks-id="135007da-62e8-46cd"> |
|
<div class="favicon favicon-scifi" title="Science Fiction & Fantasy Stack Exchange" data-webtasks-id="e8150e9d-ec6f-4ccd"></div><a href="https://scifi.stackexchange.com/questions/277103/science-fiction-short-story-possibly-titled-hop-for-pop-about-life-ending-at" class="js-gps-track question-hyperlink mb0" data-gps-track="site.switch({ item_type:11, target_site:186 }); posts_hot_network.click({ item_type:2, location:11 })" data-webtasks-id="92ec9642-f685-46e7"> |
|
Science fiction short story, possibly titled "Hop for Pop," about life ending at age 30 |
|
</a> |
|
|
|
</li> |
|
<li class="js-hidden" style="" data-webtasks-id="684d1213-5e6a-43c2"> |
|
<div class="favicon favicon-worldbuilding" title="Worldbuilding Stack Exchange" data-webtasks-id="eb812c3f-18a7-4e70"></div><a href="https://worldbuilding.stackexchange.com/questions/247882/what-would-stop-a-large-spaceship-from-looking-like-a-flying-brick" class="js-gps-track question-hyperlink mb0" data-gps-track="site.switch({ item_type:11, target_site:579 }); posts_hot_network.click({ item_type:2, location:11 })" data-webtasks-id="6084f139-73ff-4b4f"> |
|
What would stop a large spaceship from looking like a flying brick? |
|
</a> |
|
|
|
</li> |
|
<li class="js-hidden" style="" data-webtasks-id="09d54eb7-d364-4231"> |
|
<div class="favicon favicon-math" title="Mathematics Stack Exchange" data-webtasks-id="15227162-af26-402b"></div><a href="https://math.stackexchange.com/questions/4733421/integral-domains-with-isomorphic-field-of-fractions" class="js-gps-track question-hyperlink mb0" data-gps-track="site.switch({ item_type:11, target_site:69 }); posts_hot_network.click({ item_type:2, location:11 })" data-webtasks-id="4230a912-2b22-4e40"> |
|
Integral domains with isomorphic field of fractions |
|
</a> |
|
|
|
</li> |
|
<li class="js-hidden" style="" data-webtasks-id="63135f55-acd3-432b"> |
|
<div class="favicon favicon-philosophy" title="Philosophy Stack Exchange" data-webtasks-id="12bdf5fb-796e-49fc"></div><a href="https://philosophy.stackexchange.com/questions/100587/is-the-seti-project-built-on-false-premises" class="js-gps-track question-hyperlink mb0" data-gps-track="site.switch({ item_type:11, target_site:265 }); posts_hot_network.click({ item_type:2, location:11 })" data-webtasks-id="a6735cff-75a8-4e52"> |
|
Is the SETI project built on false premises? |
|
</a> |
|
|
|
</li> |
|
<li class="js-hidden" style="" data-webtasks-id="850eca6e-c15c-4c14"> |
|
<div class="favicon favicon-academia" title="Academia Stack Exchange" data-webtasks-id="fb67703f-52f6-457f"></div><a href="https://academia.stackexchange.com/questions/198612/typo-in-cover-letter-of-the-journal-name-where-my-manuscript-is-currently-under" class="js-gps-track question-hyperlink mb0" data-gps-track="site.switch({ item_type:11, target_site:415 }); posts_hot_network.click({ item_type:2, location:11 })" data-webtasks-id="6e7878c1-871c-4db2"> |
|
Typo in cover letter of the journal name where my manuscript is currently under review |
|
</a> |
|
|
|
</li> |
|
<li class="js-hidden" style="" data-webtasks-id="afd9b049-e685-4af9"> |
|
<div class="favicon favicon-mathematica" title="Mathematica Stack Exchange" data-webtasks-id="cfbe228d-d5e2-4a4a"></div><a href="https://mathematica.stackexchange.com/questions/287430/coordinatestooloptions-copiedvaluefunction-weirdness" class="js-gps-track question-hyperlink mb0" data-gps-track="site.switch({ item_type:11, target_site:387 }); posts_hot_network.click({ item_type:2, location:11 })" data-webtasks-id="23cc0f14-a06c-4e24"> |
|
CoordinatesToolOptions -> CopiedValueFunction weirdness |
|
</a> |
|
|
|
</li> |
|
<li class="js-hidden" style="" data-webtasks-id="e8baf6da-b062-403a"> |
|
<div class="favicon favicon-mathoverflow" title="MathOverflow" data-webtasks-id="845d904a-63ab-4ebc"></div><a href="https://mathoverflow.net/questions/450428/does-every-banach-space-admit-a-continuous-not-necessarily-equivalent-strictly" class="js-gps-track question-hyperlink mb0" data-gps-track="site.switch({ item_type:11, target_site:504 }); posts_hot_network.click({ item_type:2, location:11 })" data-webtasks-id="d1bafdf3-3ca3-41b6"> |
|
Does every Banach space admit a continuous (not necessarily equivalent) strictly convex norm? |
|
</a> |
|
|
|
</li> |
|
<li class="js-hidden" style="" data-webtasks-id="2dfd43bf-947d-4db9"> |
|
<div class="favicon favicon-gamedev" title="Game Development Stack Exchange" data-webtasks-id="64a8f615-8562-4e4d"></div><a href="https://gamedev.stackexchange.com/questions/206387/why-objects-are-falling-equally-in-unity" class="js-gps-track question-hyperlink mb0" data-gps-track="site.switch({ item_type:11, target_site:53 }); posts_hot_network.click({ item_type:2, location:11 })" data-webtasks-id="e9018495-c478-43c2"> |
|
Why objects are falling equally in Unity |
|
</a> |
|
|
|
</li> |
|
<li class="js-hidden" style="" data-webtasks-id="d329afdf-5a4b-4382"> |
|
<div class="favicon favicon-puzzling" title="Puzzling Stack Exchange" data-webtasks-id="c23eee3f-dc48-4a1c"></div><a href="https://puzzling.stackexchange.com/questions/121481/how-far-on-average-does-the-grasshopper-need-to-travel-to-get-home-after-its-dru" class="js-gps-track question-hyperlink mb0" data-gps-track="site.switch({ item_type:11, target_site:559 }); posts_hot_network.click({ item_type:2, location:11 })" data-webtasks-id="0d4a8ac2-0ec1-44e6"> |
|
How far on average does the grasshopper need to travel to get home after its drunken jumping spree? |
|
</a> |
|
|
|
</li> |
|
<li class="js-hidden" style="" data-webtasks-id="6150e771-8063-4f6c"> |
|
<div class="favicon favicon-ell" title="English Language Learners Stack Exchange" data-webtasks-id="2b1c82a7-aeec-4ed5"></div><a href="https://ell.stackexchange.com/questions/338634/adjectives-ending-in-ical-vs-those-ending-in-ic" class="js-gps-track question-hyperlink mb0" data-gps-track="site.switch({ item_type:11, target_site:481 }); posts_hot_network.click({ item_type:2, location:11 })" data-webtasks-id="5bcad804-0e81-4d71"> |
|
Adjectives ending in -ical vs those ending in -ic |
|
</a> |
|
|
|
</li> |
|
<li class="js-hidden" style="" data-webtasks-id="c14e1ca7-02e1-4f60"> |
|
<div class="favicon favicon-hermeneutics" title="Biblical Hermeneutics Stack Exchange" data-webtasks-id="f311804f-8439-41b3"></div><a href="https://hermeneutics.stackexchange.com/questions/85450/what-is-the-grammatical-basis-for-understanding-%d7%90%d7%9c%d7%99-in-psalm-27-differently-tha" class="js-gps-track question-hyperlink mb0" data-gps-track="site.switch({ item_type:11, target_site:320 }); posts_hot_network.click({ item_type:2, location:11 })" data-webtasks-id="fc445340-6f0c-4293"> |
|
What is the grammatical basis for understanding אלי in Psalm 2:7 differently than Psalm 22:1? |
|
</a> |
|
|
|
</li> |
|
<li class="js-hidden" style="" data-webtasks-id="64a06292-b6f8-435e"> |
|
<div class="favicon favicon-langdev" title="Programming Language Design and Implementation Stack Exchange" data-webtasks-id="5b172e97-f85b-40b6"></div><a href="https://langdev.stackexchange.com/questions/2134/what-languages-give-you-access-to-the-ast-to-modify-during-compilation" class="js-gps-track question-hyperlink mb0" data-gps-track="site.switch({ item_type:11, target_site:716 }); posts_hot_network.click({ item_type:2, location:11 })" data-webtasks-id="d5687185-b583-4ed4"> |
|
What languages give you access to the AST to modify during compilation? |
|
</a> |
|
|
|
</li> |
|
<li class="js-hidden" style="" data-webtasks-id="042b9616-3ac5-4f81"> |
|
<div class="favicon favicon-softwareengineering" title="Software Engineering Stack Exchange" data-webtasks-id="3933f18c-1c31-4956"></div><a href="https://softwareengineering.stackexchange.com/questions/446403/is-password-hashing-bad" class="js-gps-track question-hyperlink mb0" data-gps-track="site.switch({ item_type:11, target_site:131 }); posts_hot_network.click({ item_type:2, location:11 })" data-webtasks-id="381e3fdd-6de8-4df3"> |
|
Is Password Hashing Bad? |
|
</a> |
|
|
|
</li> |
|
<li class="js-hidden" style="" data-webtasks-id="219d2d14-c363-42b9"> |
|
<div class="favicon favicon-academia" title="Academia Stack Exchange" data-webtasks-id="662ffc8b-ea09-4b28"></div><a href="https://academia.stackexchange.com/questions/198812/in-cv-how-to-mention-articles-published-only-in-arxiv" class="js-gps-track question-hyperlink mb0" data-gps-track="site.switch({ item_type:11, target_site:415 }); posts_hot_network.click({ item_type:2, location:11 })" data-webtasks-id="08b9231f-4a63-4be5"> |
|
In CV, how to mention articles published only in arxiv? |
|
</a> |
|
|
|
</li> |
|
<li class="js-hidden" style="" data-webtasks-id="d1809093-c2a6-43f7"> |
|
<div class="favicon favicon-codegolf" title="Code Golf Stack Exchange" data-webtasks-id="f7afeb59-77be-4e39"></div><a href="https://codegolf.stackexchange.com/questions/262553/landmine-number-ii" class="js-gps-track question-hyperlink mb0" data-gps-track="site.switch({ item_type:11, target_site:200 }); posts_hot_network.click({ item_type:2, location:11 })" data-webtasks-id="651e023a-e847-432f"> |
|
Landmine Number II |
|
</a> |
|
|
|
</li> |
|
<li class="js-hidden" style="" data-webtasks-id="1820eec0-e784-4942"> |
|
<div class="favicon favicon-electronics" title="Electrical Engineering Stack Exchange" data-webtasks-id="fd0b3f9a-d42a-4576"></div><a href="https://electronics.stackexchange.com/questions/673323/do-hard-ips-in-fpga-require-instantiation" class="js-gps-track question-hyperlink mb0" data-gps-track="site.switch({ item_type:11, target_site:135 }); posts_hot_network.click({ item_type:2, location:11 })" data-webtasks-id="02890f3d-cc35-4be4"> |
|
Do Hard IPs in FPGA require instantiation? |
|
</a> |
|
|
|
</li> |
|
</ul> |
|
|
|
|
|
</div> |
|
|
|
<div id="feed-link" class="js-feed-link" data-webtasks-id="acaf7071-2423-4193"> |
|
<a href="/feeds/question/181865" title="Feed of this question and its answers" data-webtasks-id="28f696f3-704f-4dfd"> |
|
<svg aria-hidden="true" class="fc-orange-400 svg-icon iconRss" width="18" height="18" viewBox="0 0 18 18" data-webtasks-id="38d4218e-fec6-4f27"><path d="M3 1a2 2 0 0 0-2 2v12c0 1.1.9 2 2 2h12a2 2 0 0 0 2-2V3a2 2 0 0 0-2-2H3Zm0 1.5c6.9 0 12.5 5.6 12.5 12.5H13C13 9.55 8.45 5 3 5V2.5Zm0 5c4.08 0 7.5 3.41 7.5 7.5H8c0-2.72-2.28-5-5-5V7.5Zm0 5c1.36 0 2.5 1.14 2.5 2.5H3v-2.5Z" data-webtasks-id="a1bcfd98-9ccc-419f"></path></svg> |
|
Question feed |
|
</a> |
|
</div> |
|
<aside class="s-modal js-feed-link-modal" tabindex="-1" role="dialog" aria-labelledby="feed-modal-title" aria-describedby="feed-modal-description" aria-hidden="true" data-webtasks-id="8780df5a-5716-4390"> |
|
<div class="s-modal--dialog js-modal-dialog wmx4" role="document" data-controller="se-draggable" data-webtasks-id="d7b969e4-ca88-469b"> |
|
<h1 class="s-modal--header fw-bold js-first-tabbable c-move" id="feed-modal-title" data-se-draggable-target="handle" tabindex="0" data-webtasks-id="306d7391-fd84-45f6"> |
|
Subscribe to RSS |
|
</h1> |
|
<div class="d-flex gs4 gsy fd-column" data-webtasks-id="b2fa8f62-e6a6-46e9"> |
|
<div class="flex--item" data-webtasks-id="42814764-04ec-44c6"> |
|
<label class="d-block s-label c-default" for="feed-url" data-webtasks-id="8b66ab00-ff42-4ec4"> |
|
Question feed |
|
<p class="s-description mt2" id="feed-modal-description" data-webtasks-id="e00a3387-1747-4624">To subscribe to this RSS feed, copy and paste this URL into your RSS reader.</p> |
|
</label> |
|
</div> |
|
<div class="d-flex ps-relative" data-webtasks-id="dd8704d9-74e8-4822"> |
|
<input class="s-input" type="text" name="feed-url" id="feed-url" readonly="readonly" value="https://travel.stackexchange.com/feeds/question/181865" data-webtasks-id="1e6ac16e-ada2-4764"> |
|
<svg aria-hidden="true" class="s-input-icon fc-orange-400 svg-icon iconRss" width="18" height="18" viewBox="0 0 18 18" data-webtasks-id="8e867c88-0c00-460f"><path d="M3 1a2 2 0 0 0-2 2v12c0 1.1.9 2 2 2h12a2 2 0 0 0 2-2V3a2 2 0 0 0-2-2H3Zm0 1.5c6.9 0 12.5 5.6 12.5 12.5H13C13 9.55 8.45 5 3 5V2.5Zm0 5c4.08 0 7.5 3.41 7.5 7.5H8c0-2.72-2.28-5-5-5V7.5Zm0 5c1.36 0 2.5 1.14 2.5 2.5H3v-2.5Z" data-webtasks-id="a38ec882-b541-40a7"></path></svg> |
|
</div> |
|
</div> |
|
<a class="s-modal--close s-btn s-btn__muted js-modal-close js-last-tabbable" href="#" aria-label="Close" data-webtasks-id="3222ff52-deb1-4d21"> |
|
<svg aria-hidden="true" class="svg-icon iconClearSm" width="14" height="14" viewBox="0 0 14 14" data-webtasks-id="5de4a4f6-125c-4131"><path d="M12 3.41 10.59 2 7 5.59 3.41 2 2 3.41 5.59 7 2 10.59 3.41 12 7 8.41 10.59 12 12 10.59 8.41 7 12 3.41Z" data-webtasks-id="4da129b4-553f-44fb"></path></svg> |
|
</a> |
|
</div> |
|
</aside> |
|
|
|
</div> |
|
|
|
</div> |
|
<script data-webtasks-id="275a5f48-7b3e-4df6">StackExchange.ready(function(){$.get('/posts/181865/ivc/04fc?prg=1d0ebe39-0589-461d-ab0d-a300a79a17d3');});</script> |
|
<noscript data-webtasks-id="b0306821-92dd-4cf0"><div><img src="/posts/181865/ivc/04fc?prg=1d0ebe39-0589-461d-ab0d-a300a79a17d3" class="dno" alt="" width="0" height="0"></div></noscript></div> |
|
|
|
<script data-webtasks-id="7a606c58-8e29-4659"> |
|
$('#wmd-input').one("keypress", function () { |
|
$.ajax({ |
|
url: '/accounts/email-settings-form', |
|
cache: false, |
|
success: function (data) { |
|
$('#submit-button').parent().prepend(data); |
|
} |
|
}); |
|
}); |
|
|
|
</script> |
|
|
|
|
|
</div> |
|
</div> |
|
|
|
|
|
<script type="text/javascript" data-webtasks-id="72219e8e-5708-46e8"> |
|
var cam = cam || { opt: {} }; |
|
var clcGamLoaderOptions = cam || { opt: {} }; |
|
var opt = clcGamLoaderOptions.opt; |
|
|
|
opt.refresh = !1; |
|
opt.refreshInterval = 0; |
|
opt.sf = !1; |
|
opt.hb = !1; |
|
opt.ll = !0; |
|
opt.tlb_position = 0; |
|
opt.personalization_consent = !1; |
|
opt.targeting_consent = !1; |
|
opt.performance_consent = !1; |
|
|
|
opt.targeting = {Registered:['true'],'nontech-ron':['where-on-earth','bavaria'],NumberOfAnswers:['3']}; |
|
opt.adReportEnabled = !0; |
|
opt.adReportUrl = '/ads/report-ad'; |
|
opt.adReportText = 'Report this ad'; |
|
opt.adReportFileTypeErrorMessage = 'Please select a PNG or JPG file.'; |
|
opt.adReportFileSizeErrorMessage = 'The file must be under 2 MiB.'; |
|
opt.adReportErrorText = 'Error uploading ad report.'; |
|
opt.adReportThanksText = 'Thanks for your feedback. We’ll review this against our code of conduct and take action if necessary.'; |
|
opt.adReportLoginExpiredMessage = 'Your login session has expired, please login and try again.'; |
|
opt.adReportLoginErrorMessage = 'An error occurred when loading the report form - please try again'; |
|
opt.adReportModalClass = 'js-ad-report'; |
|
|
|
opt.requestGuid = '1d0ebe39-0589-461d-ab0d-a300a79a17d3'; |
|
|
|
|
|
opt.targeting.TargetingConsent = ['False_Active']; |
|
|
|
const urlParams = new URLSearchParams(window.location.search); |
|
if (urlParams.has('dfptestads')) { |
|
const dfptestads = urlParams.get('dfptestads'); |
|
opt.targeting.DfpTestAds = dfptestads; |
|
} |
|
</script> |
|
<script data-webtasks-id="89306c7d-3d62-452d">;(()=>{"use strict";var __webpack_modules__={23:(e,t,o)=>{o.d(t,{Z7:()=>l,eq:()=>r,kG:()=>n});const s=/^\/tags\//.test(location.pathname)||/^\/questions\/tagged\//.test(location.pathname)?"tag-pages":/^\/$/.test(location.pathname)||/^\/home/.test(location.pathname)?"home-page":"question-pages";let a=location.hostname;const i={slots:{lb:[[728,90]],mlb:[[728,90]],smlb:[[728,90]],bmlb:[[728,90]],sb:e=>"dfp-tsb"===e?[[300,250],[300,600]]:[[300,250]],"tag-sponsorship":[[730,135]],"mobile-below-question":[[320,50],[300,250]],msb:[[300,250],[300,600]],"talent-conversion-tracking":[[1,1]],"site-sponsorship":[[230,60]]},ids:{"dfp-tlb":"lb","dfp-mlb":"mlb","dfp-smlb":"smlb","dfp-bmlb":"bmlb","dfp-tsb":"sb","dfp-isb":"sb","dfp-tag":"tag-sponsorship","dfp-msb":"msb","dfp-sspon":"site-sponsorship","dfp-m-aq":"mobile-below-question"},idsToExcludeFromAdReports:["dfp-sspon"]};function n(){return Object.keys(i.ids)}function r(e){return i.idsToExcludeFromAdReports.indexOf(e)<0}function l(e){var t=e.split("_")[0];const o=i.ids[t];let n=i.slots[o];return"function"==typeof n&&(n=n(t)),{path:`/248424177/${a}/${o}/${s}`,sizes:n,zone:o}}},865:(e,t,o)=>{function s(e){return"string"==typeof e?document.getElementById(e):e}function a(e){return!!(e=s(e))&&"none"===getComputedStyle(e).display}function i(e){return!a(e)}function n(e){return!!e}function r(e){return/^\s*$/.test(s(e).innerHTML)}function l(e){const{style:t}=e;t.height=t.maxHeight=t.minHeight="auto",t.display="none"}function d(e){const{style:t}=e;t.height=t.maxHeight=t.minHeight="auto",t.display="none",[].forEach.call(e.children,d)}function c(e){const{style:t}=e;t.height=t.maxHeight=t.minHeight="auto",t.removeProperty("display")}function p(e){const t=document.createElement("script");t.src=e,document.body.appendChild(t)}function g(e){return o=e,(t=[]).push=function(e){return o(),delete this.push,this.push(e)},t;var t,o}function h(e){let t="function"==typeof HTMLTemplateElement;var o=document.createElement(t?"template":"div");return e=e.trim(),o.innerHTML=e,t?o.content.firstChild:o.firstChild}o.d(t,{$Z:()=>c,Bv:()=>h,Gx:()=>p,Nj:()=>s,QZ:()=>g,cf:()=>l,pn:()=>i,wo:()=>d,xb:()=>r,xj:()=>a,yb:()=>n})},763:(__unused_webpack_module,__webpack_exports__,__webpack_require__)=>{__webpack_require__.d(__webpack_exports__,{t:()=>AdReports});var _common_helper__WEBPACK_IMPORTED_MODULE_2__=__webpack_require__(865),_console__WEBPACK_IMPORTED_MODULE_1__=__webpack_require__(276),_ad_units__WEBPACK_IMPORTED_MODULE_0__=__webpack_require__(23);class AdReports{constructor(e,t){if(this.googletag=e,this.cam=t,this.allowedFileTypes=["image/png","image/jpg","image/jpeg"],this.ignoreValidation=!1,_console__WEBPACK_IMPORTED_MODULE_1__.cM("Ad reporting init"),this.cam=t,this.callOnButtonClick=e=>this.onButtonClick(e),this.googletag.pubads().addEventListener("slotRenderEnded",e=>this.handleSlotRendered(e)),Array.isArray(t.slotsRenderedEvents)){_console__WEBPACK_IMPORTED_MODULE_1__.cM("Adding report button to "+t.slotsRenderedEvents.length+" events that have transpired");for(var o=0;o<t.slotsRenderedEvents.length;o++)this.handleSlotRendered(t.slotsRenderedEvents[o])}}handleSlotRendered(e){if(e&&e.slot&&!e.isEmpty&&(e.creativeId||e.lineItemId||!e.isEmpty)){var t=e.slot.getSlotElementId();if(t){var o=document.getElementById(t);if(o)if((0,_ad_units__WEBPACK_IMPORTED_MODULE_0__.eq)(t)){var s=o?.closest(".js-zone-container")?.querySelector(".js-report-ad-button-container");s.innerHTML="",s.append(this.createButton(e)),s.style.height="24px",_console__WEBPACK_IMPORTED_MODULE_1__.cM("Added report button to the bottom of "+t)}else _console__WEBPACK_IMPORTED_MODULE_1__.cM("Not adding report button to the bottom of "+t+": shouldHaveReportButton = false");else _console__WEBPACK_IMPORTED_MODULE_1__.cM("Not adding report button to the bottom of "+t+": resolved invalid adUnit element")}else _console__WEBPACK_IMPORTED_MODULE_1__.cM("Not adding report button to the bottom of element: invalid adUnitElementId")}else _console__WEBPACK_IMPORTED_MODULE_1__.cM("Not adding report button to the bottom of element: invalid SlotRenderEndedEvent")}async onButtonClick(e){e.preventDefault();let t=e.target;const o=t.dataset.modalUrl,s=t.dataset.googleEventData;return await this.loadModal(o,t,s),!1}createButton(e){let t=document.createElement("button");var o=JSON.stringify(e);return t.dataset.googleEventData=o,t.dataset.modalUrl=this.cam.opt.adReportUrl,t.dataset.adUnit=e.slot.getSlotElementId(),t.classList.add("js-report-ad","s-btn","s-btn__link","fs-fine","mt2","float-right"),t.append(document.createTextNode(this.cam.opt.adReportText)),t.removeEventListener("click",this.callOnButtonClick),t.addEventListener("click",this.callOnButtonClick),t}async loadModal(url,$link,googleEventData){try{await window.StackExchange.helpers.loadModal(url,{returnElements:window.$($link)}),this.initForm(googleEventData)}catch(e){var message="",response=e.responseText?eval(`(${e.responseText})`):null;message=response&&response.isLoggedOut?this.cam.opt.adReportLoginExpiredMessage:this.cam.opt.adReportLoginErrorMessage,window.StackExchange.helpers.showToast(message,{type:"danger"})}}removeModal(){window.StackExchange.helpers.closePopups(document.querySelectorAll("."+this.cam.opt.adReportModalClass),"dismiss")}initForm(e,t=!1){this.ignoreValidation=t,this.$form=document.querySelector(".js-ad-report-form"),this.$googleEventData=this.$form.querySelector(".js-json-data"),this.$adReportReasons=this.$form.querySelectorAll(".js-ad-report-reason"),this.$adReportReasonOther=this.$form.querySelector(".js-ad-report-reason-other"),this.$fileUploaderInput=this.$form.querySelector(".js-file-uploader-input"),this.$imageUploader=this.$form.querySelector(".js-image-uploader"),this.$clearImageUpload=this.$form.querySelector(".js-clear-image-upload"),this.$imageUploaderText=this.$form.querySelector(".js-image-uploader-text"),this.$imageUploaderPreview=this.$form.querySelector(".js-image-uploader-preview"),this.$fileErrorMessage=this.$form.querySelector(".js-file-error");const o=this.$form.querySelector(".js-drag-drop-enabled"),s=this.$form.querySelector(".js-drag-drop-disabled");this.$googleEventData.value=e,this.$adReportReasons.forEach((e,t)=>e.addEventListener("change",e=>{this.$adReportReasonOther.classList.toggle("d-none","3"!==e.target.value)})),this.$fileUploaderInput.addEventListener("change",()=>{this.validateFileInput()&&this.updateImagePreview(this.$fileUploaderInput.files)}),this.$clearImageUpload.addEventListener("click",e=>{e.preventDefault(),this.clearImageUpload()});try{this.$fileUploaderInput[0].value="",this.$imageUploader.addEventListener("dragenter dragover dragleave drop",this.preventDefaults),this.$imageUploader.addEventListener("dragenter dragover",this.handleDragStart),this.$imageUploader.addEventListener("dragleave drop",this.handleDragEnd),this.$imageUploader.addEventListener("drop",this.handleDrop)}catch(e){o.classList.add("d-none"),s.classList.remove("d-none")}this.$form.removeEventListener("",this.handleDragEnd),this.$form.addEventListener("submit",async e=>(e.preventDefault(),this.submitForm(),!1))}clearImageUpload(){this.$fileUploaderInput.value="",this.$imageUploaderPreview.setAttribute("src",""),this.$imageUploaderPreview.classList.add("d-none"),this.$clearImageUpload.classList.add("d-none"),this.$imageUploaderText.classList.remove("d-none"),this.$imageUploader.classList.add("p16","ba","bas-dashed","bc-black-100")}preventDefaults(e){e.preventDefault(),e.stopPropagation()}handleDragStart(e){this.$imageUploader.classList.remove("bas-dashed"),this.$imageUploader.classList.add("bas-solid","bc-black-100")}handleDragEnd(e){this.$imageUploader.classList.remove("bas-solid","bc-black-100"),this.$imageUploader.classList.add("bas-dashed")}handleDrop(e){var t=e.originalEvent.dataTransfer.files;FileReader&&t&&1===t.length&&(this.$fileUploaderInput.files=t,this.validateFileInput()&&this.updateImagePreview(t))}setError(e){this.$fileErrorMessage.parentElement.classList.toggle("has-error",e)}updateImagePreview(e){this.$imageUploader.classList.remove("p16","ba","bas-dashed","bc-black-100"),this.$clearImageUpload.classList.remove("d-none"),this.$imageUploaderText.classList.add("d-none");var t=new FileReader;t.onload=e=>{null!=e.target&&(this.$imageUploaderPreview.setAttribute("src",e.target.result),this.$imageUploaderPreview.classList.remove("d-none"))},t.readAsDataURL(e[0])}validateFileInput(){if(this.ignoreValidation)return!0;const e=this.cam.opt.adReportFileTypeErrorMessage,t=this.cam.opt.adReportFileSizeErrorMessage;if(null==this.$fileUploaderInput.files)return!1;var o=this.$fileUploaderInput.files[0];return null==o?(this.setError(!0),!1):this.allowedFileTypes.indexOf(o.type)<0?(this.$fileErrorMessage.textContent=e,this.$fileErrorMessage.classList.remove("d-none"),this.setError(!0),!1):o.size>2097152?(this.$fileErrorMessage.textContent=t,this.$fileErrorMessage.classList.remove("d-none"),this.setError(!0),!1):(this.$fileErrorMessage.classList.add("d-none"),this.setError(!1),!0)}async gatherDiagnosticInfo(){return{BrowserVersion:await this.getBrowserVersion()}}getElementSource(e){return e.outerHTML}getNestedIFrameElement(e){var t=e.querySelector("iframe");return t.contentDocument?t.contentDocument.documentElement:t.contentWindow.document.documentElement}async getBrowserVersion(){return await navigator.userAgentData.getHighEntropyValues(["fullVersionList"]).then(e=>JSON.stringify(e.fullVersionList))}async submitForm(){if(!this.validateFileInput())return!1;this.$form.querySelector("[type=submit]").setAttribute("disabled","true");var e=JSON.parse(this.$googleEventData.value||"{}");e.Reason=parseInt(this.$form.querySelector(".js-ad-report-reason:checked").value,10),e.Description=this.$adReportReasonOther.value,this.$googleEventData.value=JSON.stringify(e);var t=new FormData(this.$form);if("1"===t.get("shareDiagnosticInfo")){var o=await this.gatherDiagnosticInfo();Object.keys(o).forEach(e=>t.append(e,o[e]))}try{const e=await window.fetch(this.$form.getAttribute("action"),{method:this.$form.getAttribute("method"),body:t,cache:"no-cache"}),o=e.headers.get("content-type")||"",a=await e.text();if(!e.ok)throw new Error("response not valid");if(0===o.indexOf("text/html")){var s=(0,_common_helper__WEBPACK_IMPORTED_MODULE_2__.Bv)(a);const e=s?s.querySelector(".js-modal-content"):null;if(_console__WEBPACK_IMPORTED_MODULE_1__.cM("$popupContent"),_console__WEBPACK_IMPORTED_MODULE_1__.cM(e),!e)throw new Error(`Could not find .js-modal-content in response from ${this.$form.getAttribute("action")}`);document.querySelector(".js-modal-content").replaceWith(e)}else window.StackExchange.helpers.showToast(this.cam.opt.adReportThanksText,{type:"success"}),this.removeModal()}catch(e){window.StackExchange.helpers.showToast(this.cam.opt.adReportErrorText,{type:"danger"})}finally{let e=this.$form.querySelector("[type=submit]");e&&e.removeAttribute("disabled")}}}},276:(e,t,o)=>{function s(...e){}function a(...e){}o.d(t,{cM:()=>s,vU:()=>a})}},__webpack_module_cache__={};function __webpack_require__(e){var t=__webpack_module_cache__[e];if(void 0!==t)return t.exports;var o=__webpack_module_cache__[e]={exports:{}};return __webpack_modules__[e](o,o.exports,__webpack_require__),o.exports}__webpack_require__.d=(e,t)=>{for(var o in t)__webpack_require__.o(t,o)&&!__webpack_require__.o(e,o)&&Object.defineProperty(e,o,{enumerable:!0,get:t[o]})},__webpack_require__.o=(e,t)=>Object.prototype.hasOwnProperty.call(e,t);var __webpack_exports__={};(()=>{var e=__webpack_require__(276),t=(e=>(e[e.Above=0]="Above",e[e.Below=1]="Below",e))(t||{});const o=Object.assign({},{"lib":"https://cdn.sstatic.net/clc/js/bundles/gam_loader_script/gam_loader_script.bundle.741.17faee8eba9a98950467.min.js","style":null,"u":null,"wa":true,"kt":2000,"tto":true,"h":"clc.stackoverflow.com","allowed":"^(((talent\\.)?stackoverflow)|(blog\\.codinghorror)|(.*\\.googlesyndication)|(serverfault|askubuntu)|([^\\.]+\\.stackexchange))\\.com$","wv":true,"al":false,"abd":true,"cpa_liid":[5882654614],"cpa_cid":[138377597667],"dp":false});var s=__webpack_require__(23),a=__webpack_require__(865),i=__webpack_require__(763);class n{constructor(t,o){this.googletag=t,this.interval=o,e.cM("Ad refresh init. interval: "+o),this.googletag.pubads().addEventListener("impressionViewable",e=>this.onImpressionViewable(e)),e.cM("done enabling ad refresh")}onImpressionViewable(t){var o=t.slot;e.cM("ad refresh - slot "+o.getSlotElementId()+" is viewable, initializing refresh"),this.scheduleRefresh(o)}scheduleRefresh(e){setTimeout(()=>this.refreshAdSlot(e),1e3*this.interval)}refreshAdSlot(t){var o=t.getSlotElementId();this.isElementVisibleInBrowser(o)?(e.cM("refreshing ad slot "+o),googletag.pubads().refresh([t])):(e.cM("refresh skipped this time; ad slot not viewable:"+o),this.scheduleRefresh(t))}isElementVisibleInBrowser(e){var t=document.getElementById(e);if(null!==t){var o=t.getBoundingClientRect();if(o.top>=0&&o.left>=0&&o.bottom<=(window.innerHeight||document.documentElement.clientHeight)&&o.right<=(window.innerWidth||document.documentElement.clientWidth))return!0}return!1}}window.cam=new class{constructor(){this.gptImported=!1,this.collapsed={},e.cM("constructor"),this.clc_options=o,window.clcGamLoaderOptions?Object.assign(this,window.clcGamLoaderOptions):void 0===this.opt&&(this.opt=window.opt)}init(){if(e.cM("init"),void 0===this.opt)throw new Error("opt not set, required by GAM Loader");e.cM("setup message handler"),window.addEventListener("message",e=>{this.onmessage(e)}),this.opt.targeting&&this.opt.targeting.ProductVariant&&"SystemDefault"===this.opt.targeting.ProductVariant[0]&&(window.matchMedia&&window.matchMedia("(prefers-color-scheme: dark)").matches?this.opt.targeting.ProductVariant="Dark":this.opt.targeting.ProductVariant="Light")}handleSlotRenderedNoAdReport(){if(googletag.pubads().addEventListener("slotRenderEnded",e=>this.applyExtraMarginBottom(e)),Array.isArray(this.slotsRenderedEvents))for(var e=0;e<this.slotsRenderedEvents.length;e++)this.applyExtraMarginBottom(this.slotsRenderedEvents[e])}onmessage(t){let o="omni";if(t.data&&("string"==typeof t.data||t.data instanceof String)){if(0===t.data.indexOf("get-omni-")){e.cM("Recevied get-omni message, sending back omni");var s=t.source,i=this.opt.omni,n="string"==typeof i?i:"";s.postMessage([o,n,this.opt.requestGuid].join("|"),"*")}else if(0===t.data.indexOf("collapse-")){e.cM("Recevied collapse message, collapse ad iframe"),e.cM(t);for(var r=t.source.window,l=document.getElementsByTagName("IFRAME"),d=0;d<l.length;d++){var c=l[d];if(c.contentWindow==r)return void(0,a.wo)(c.parentElement.parentElement.parentElement)}}else if(0===t.data.indexOf("resize|")){e.cM("Recevied resize message, resize ad iframe"),e.cM(t);let o=this._getFrameByEvent(t),s=t.data.indexOf("|")+1,a=t.data.slice(s),i=parseFloat(a)+.5;e.cM("New iframe height "+i),o.height=i.toString(),o.parentElement.style.height=i.toString()+"px"}else if(0===t.data.indexOf("getmarkup|")){let o=t.data.indexOf("|")+1,s=t.data.slice(o);e.cM("Recevied get markup message: "+s);let a=this._getFrameByEvent(t).closest(".everyonelovesstackoverflow");const i=document.createElement("script");i.dataset.adZoneId=a.id,i.src=s,document.body.appendChild(i)}else if(0===t.data.indexOf("window-location|")){let o=t.data.indexOf("|")+1,s=t.data.slice(o);e.cM("Recevied window location message: "+s),s.startsWith("/")||(s="/"+s),window.open(window.location.protocol+"//"+window.location.host+s,"_blank")}}else e.cM("Received unhandled message")}_getFrameByEvent(e){return Array.from(document.getElementsByTagName("iframe")).filter(t=>t.contentWindow===e.source)[0]}classifyZoneIds(e){const t=e.map(a.Nj).filter(a.yb);return{eligible:t.filter(a.xb).filter(a.pn),ineligible:t.filter(a.xj)}}applyExtraMarginBottom(t){if(t&&t.slot&&!t.isEmpty&&(t.creativeId||t.lineItemId||!t.isEmpty)){var o=t.slot.getSlotElementId();if(o){var a=document.getElementById(o);if(a)if((0,s.eq)(o)){var i=a?.closest(".js-zone-container");i.style.marginBottom="24px",e.cM("Applied extra margin to the bottom of "+o)}else e.cM("Not applying extra margin to the bottom of "+o+": shouldHaveReportButton = false");else e.cM("Not applying extra margin to the bottom of "+o+": resolved invalid adUnit element")}else e.cM("Not applying extra margin to the bottom of element: invalid adUnitElementId")}else e.cM("Not applying extra margin to the bottom of element: invalid SlotRenderEndedEvent")}load(o=(0,s.kG)()){const r=this.opt.tlb_position===t.Above?["dfp-mlb","dfp-smlb"]:["dfp-mlb","dfp-smlb","dfp-tlb"];if(!this.isGptReady())return e.cM("Initializing..."),this.initGpt(),void googletag.cmd.push(()=>this.load(o));this.opt.adReportEnabled?(e.cM("Ad reporting enabled"),this.adReports=new i.t(googletag,this)):(e.cM("Ad reporting not enabled"),this.handleSlotRenderedNoAdReport()),this.opt.refresh?(e.cM("Ad refresh enabled"),this.adRefresh=new n(googletag,this.opt.refreshInterval)):e.cM("Ad refresh not enabled"),e.cM("Attempting to load ads into ids: ",o);const{eligible:l,ineligible:d}=this.classifyZoneIds(o);if(this.initDebugPanel(googletag,l.concat(d)),l.forEach(e=>(0,a.cf)(e)),d.forEach(a.wo),0===l.length)return void e.cM("Found no ad ids on page");e.cM("Eligible ids:",l),this.opt.abd&&this.appendAdblockDetector();var c=googletag.pubads().getSlots().filter(e=>o.indexOf(e.getSlotElementId())>=0);googletag.destroySlots(c),this.opt.sf&&(googletag.pubads().setForceSafeFrame(!0),googletag.pubads().setSafeFrameConfig({allowOverlayExpansion:!0,allowPushExpansion:!0,sandbox:!0})),e.cM("Targeting consent: Checking...");let p=!1,g=!1;void 0!==this.opt.targeting_consent&&(g=!0,e.cM("Targeting consent: Parameter set"),e.cM("Targeting consent: Consent given? ",this.opt.targeting_consent),p=this.opt.targeting_consent),void 0!==this.opt.personalization_consent&&(e.cM("Personalization consent: Parameter set"),e.cM("Personalization consent: Consent given? ",this.opt.personalization_consent),p=p&&this.opt.personalization_consent),p=p&&g,googletag.pubads().setRequestNonPersonalizedAds(p?0:1),p||googletag.pubads().setPrivacySettings({limitedAds:!0}),this.opt.ll||googletag.pubads().enableSingleRequest(),cam.sreEvent||(googletag.pubads().addEventListener("slotRenderEnded",e=>this.onSlotRendered(e)),cam.sreEvent=!0),this.setTargeting(googletag);var h=l.filter(e=>!this.opt.ll||r.indexOf(e.id)<0),m=l.filter(e=>!!this.opt.ll&&r.indexOf(e.id)>=0);e.cM("Up front ids:",h),e.cM("Lazy loaded ids:",m),h.forEach(t=>{e.cM(`Defining ad for element ${t.id}`),this.defineSlot(t.id,googletag),t.setAttribute("data-dfp-zone","true")}),googletag.enableServices(),h.forEach(t=>{e.cM(`Displaying ad for element ${t.id}`),googletag.cmd.push(()=>googletag.display(t.id))}),this.opt.ll&&(e.cM("Enabling lazy loading for GAM"),googletag.pubads().enableLazyLoad({fetchMarginPercent:0,renderMarginPercent:0}),e.cM("Setting up lazy loaded ad units"),m.forEach(t=>{e.cM(`Lazy loading - Defining Slot ${t.id}`),this.defineSlot(t.id,googletag)}),m.forEach(t=>{e.cM(`Lazy loading - Displaying ad for element ${t.id}`),googletag.cmd.push(()=>googletag.display(t.id))}))}setTargeting(t){let o=this.opt.targeting;if(!o)throw new Error("Targeting not defined");Object.keys(o).forEach(s=>{e.cM(`-> targeting - ${s}: ${o[s]}`),t.pubads().setTargeting(s,o[s])})}appendAdblockDetector(){const e=document.createElement("div");e.className="adsbox",e.id="clc-abd",e.style.position="absolute",e.style.pointerEvents="none",e.innerHTML=" ",document.body.appendChild(e)}onSlotRendered(o){try{const n=o.slot.getSlotElementId();let r=[];n||r.push("id=0");const l=document.getElementById(n);if(n&&!l&&r.push("el=0"),0!==r.length)return void this.stalled(r.join("&"));const{path:d,sizes:c,zone:p}=(0,s.Z7)(n);if(this.collapsed[p]&&o.isEmpty)return e.cM(`No line item for the element #${l.id}... collapsing.`),void(0,a.wo)(l);if(this.slotsRenderedEvents.push(o),o.lineItemId||o.creativeId||!o.isEmpty){e.cM(`Rendered ad for element #${l.id} [line item #${o.lineItemId}]`),e.cM(o);var i=l.parentElement;if(i.classList.contains("js-zone-container")){switch((0,a.cf)(i),n){case"dfp-tlb":this.opt.tlb_position===t.Above?i.classList.add("mb8"):i.classList.add("mt16");break;case"dfp-tag":i.classList.add("mb8");break;case"dfp-msb":i.classList.add("mt16");break;case"dfp-mlb":case"dfp-smlb":case"dfp-bmlb":i.classList.add("my8");break;case"dfp-isb":i.classList.add("mt24");break;case"dfp-m-aq":i.classList.add("my12"),i.classList.add("mx-auto")}(0,a.$Z)(i),(0,a.$Z)(l)}else e.cM(`No ad for element #${l.id}, collapsing`),e.cM(o),(0,a.wo)(l)}}catch(e){this.stalled("e=1")}}stalled(e){(new Image).src=`https://${this.clc_options.h}/stalled.gif?${e}`}defineSlot(t,o){"dfp-isb"===t&&(e.cM("-> targeting - Sidebar: Inline"),o.pubads().setTargeting("Sidebar",["Inline"])),"dfp-tsb"===t&&(e.cM("-> targeting - Sidebar: Right"),o.pubads().setTargeting("Sidebar",["Right"]));const{path:a,sizes:i,zone:n}=(0,s.Z7)(t);e.cM(`Defining slot for ${t}: ${a}, sizes: ${JSON.stringify(i)}`),o.defineSlot(a,i,t).addService(o.pubads())}importGptLibrary(){this.gptImported||(this.gptImported=!0,void 0===this.opt.targeting_consent||this.opt.targeting_consent?(0,a.Gx)("https://securepubads.g.doubleclick.net/tag/js/gpt.js"):(0,a.Gx)("https://pagead2.googlesyndication.com/tag/js/gpt.js"))}isGptReady(){return"undefined"!=typeof googletag&&!!googletag.apiReady}initGpt(){"undefined"==typeof googletag&&(window.googletag={cmd:(0,a.QZ)(()=>this.importGptLibrary())})}initDebugPanel(t,o){e.cM("initDebugPanel"),e.cM("Not showing debug panel")}},window.clcGamLoaderOptions&&(cam.init(),cam.load())})()})();</script><script src="https://pagead2.googlesyndication.com/tag/js/gpt.js" data-webtasks-id="ceea60ab-322c-4ffc"></script> |
|
|
|
<footer id="footer" class="site-footer js-footer" role="contentinfo" data-webtasks-id="f7d7d78a-2f02-4f78"> |
|
<div class="site-footer--container" data-webtasks-id="46add8bd-3561-493a"> |
|
<nav class="site-footer--nav" data-webtasks-id="e8cd4858-0d84-416c"> |
|
<div class="site-footer--col" data-webtasks-id="8b22f507-082d-46cf"> |
|
<h5 class="-title" data-webtasks-id="61b4f51e-e16d-471f"><a href="/" data-webtasks-id="605bb756-782f-4932">Travel</a></h5> |
|
<ul class="-list js-primary-footer-links" data-webtasks-id="25d0a8a3-9c91-4c6b"> |
|
<li data-webtasks-id="a93a060a-c20e-4939"><a class="js-gps-track -link" data-gps-track="footer.click({ location: 2, link: 2 })" href="/tour" data-webtasks-id="cbce05f4-a250-4e10">Tour</a></li> |
|
<li data-webtasks-id="60bf84c9-757c-43fa"><a href="/help" class="js-gps-track -link" data-gps-track="footer.click({ location: 2, link: 3 })" data-webtasks-id="479a238e-c24f-46f9">Help</a></li> |
|
<li data-webtasks-id="61e6f935-0540-46b9"><a class="js-gps-track -link" data-gps-track="footer.click({ location: 2, link: 5 })" href="https://chat.stackexchange.com?tab=site&host=travel.stackexchange.com" data-webtasks-id="c0bf1d4f-ab2e-4bb2">Chat</a></li> |
|
<li data-webtasks-id="e104c1dc-b272-41bb"><a class="js-gps-track -link" data-gps-track="footer.click({ location: 2, link: 13 })" href="/contact" data-webtasks-id="48af378e-094a-4f98">Contact</a></li> |
|
<li data-webtasks-id="bb184e13-bad6-4c13"><a class="js-gps-track -link" data-gps-track="footer.click({ location: 2, link: 14 })" href="https://travel.meta.stackexchange.com" data-webtasks-id="e462d5a2-5d28-4d24">Feedback</a></li> |
|
</ul> |
|
</div> |
|
<div class="site-footer--col" data-webtasks-id="56bfdf12-6cff-4850"> |
|
<h5 class="-title" data-webtasks-id="0f3d51cb-7494-493d"><a class="js-gps-track" data-gps-track="footer.click({ location: 2, link: 1 })" href="https://stackoverflow.co/" data-webtasks-id="6b673afe-aa60-485c">Company</a></h5> |
|
<ul class="-list" data-webtasks-id="527d8a5d-73e7-44a5"> |
|
<li data-webtasks-id="17fe8841-d4ef-4383"><a href="https://stackoverflow.com" class="js-gps-track -link" data-gps-track="footer.click({ location: 2, link: 15})" data-webtasks-id="0e9a9813-6041-4049">Stack Overflow</a></li> |
|
<li data-webtasks-id="5fd30b33-5535-4a35"><a href="https://stackoverflow.co/teams/" class="js-gps-track -link" data-gps-track="footer.click({ location: 2, link: 29 })" data-webtasks-id="39bde7dc-b1a5-4954">Teams</a></li> |
|
<li data-webtasks-id="08e28990-97d9-4ca3"><a href="https://stackoverflow.co/advertising/" class="js-gps-track -link" data-gps-track="footer.click({ location: 2, link: 21 })" data-webtasks-id="1cf9c2e4-b3d4-45de">Advertising</a></li> |
|
<li data-webtasks-id="cb03bb0e-289f-4cb7"><a href="https://stackoverflow.co/collectives/" class="js-gps-track -link" data-gps-track="footer.click({ location: 2, link: 40 })" data-webtasks-id="bae227e1-34d4-48cd">Collectives</a></li> |
|
<li data-webtasks-id="a0c82a80-4bf4-4a02"><a href="https://stackoverflow.co/talent/" class="js-gps-track -link" data-gps-track="footer.click({ location: 2, link: 20 })" data-webtasks-id="0fafee12-8ce2-4ca9">Talent</a></li> |
|
<li data-webtasks-id="8ec55b53-15b9-49fb"><a class="js-gps-track -link" data-gps-track="footer.click({ location: 2, link: 1 })" href="https://stackoverflow.co/" data-webtasks-id="838c0d50-a1d5-47c2">About</a></li> |
|
<li data-webtasks-id="dea6fb01-549f-4acc"><a class="js-gps-track -link" data-gps-track="footer.click({ location: 2, link: 27 })" href="https://stackoverflow.co/company/press/" data-webtasks-id="ed67e34b-c01b-40fe">Press</a></li> |
|
<li data-webtasks-id="0be46101-43fc-45a6"><a class="js-gps-track -link" data-gps-track="footer.click({ location: 2, link: 7 })" href="https://stackoverflow.com/legal" data-webtasks-id="20a3728d-4de8-4afd">Legal</a></li> |
|
<li data-webtasks-id="420fb185-31ab-4f04"><a class="js-gps-track -link" data-gps-track="footer.click({ location: 2, link: 8 })" href="https://stackoverflow.com/legal/privacy-policy" data-webtasks-id="7d7308ec-d499-4a9e">Privacy Policy</a></li> |
|
<li data-webtasks-id="e30a47ee-827c-4daa"><a class="js-gps-track -link" data-gps-track="footer.click({ location: 2, link: 37 })" href="https://stackoverflow.com/legal/terms-of-service" data-webtasks-id="f82d3f23-7ecd-4734">Terms of Service</a></li> |
|
<li class="" id="consent-footer-link" data-webtasks-id="1430d690-070a-4014"><a class="js-gps-track -link js-cookie-settings" data-gps-track="footer.click({ location: 2, link: 38 })" href="#" data-consent-popup-loader="footer" data-webtasks-id="4ac6641f-4f67-4485">Cookie Settings</a></li> |
|
<li data-webtasks-id="ad1b042c-aee8-40be"><a class="js-gps-track -link" data-gps-track="footer.click({ location: 2, link: 39 })" href="https://stackoverflow.com/legal/cookie-policy" data-webtasks-id="d489605f-8a7e-49e7">Cookie Policy</a></li> |
|
</ul> |
|
</div> |
|
<div class="site-footer--col site-footer--categories-nav" data-webtasks-id="642f7c29-3d15-4035"> |
|
<div data-webtasks-id="d343da80-c8c6-4097"> |
|
<h5 class="-title" data-webtasks-id="55091a61-6d54-4fa1"><a href="https://stackexchange.com" data-gps-track="footer.click({ location: 2, link: 30 })" data-webtasks-id="fd2d8317-e7ea-42fc">Stack Exchange Network</a></h5> |
|
<ul class="-list" data-webtasks-id="ba218f6e-5ef3-4074"> |
|
<li data-webtasks-id="3671152c-0cc8-407b"> |
|
<a href="https://stackexchange.com/sites#technology" class="-link js-gps-track" data-gps-track="footer.click({ location: 2, link: 24 })" data-webtasks-id="0fb13cb3-ba10-44c8"> |
|
Technology |
|
</a> |
|
</li> |
|
<li data-webtasks-id="7a801ea5-add3-4977"> |
|
<a href="https://stackexchange.com/sites#culturerecreation" class="-link js-gps-track" data-gps-track="footer.click({ location: 2, link: 24 })" data-webtasks-id="6d26262a-35c9-4f63"> |
|
Culture & recreation |
|
</a> |
|
</li> |
|
<li data-webtasks-id="bd3f8a4e-c36a-41e0"> |
|
<a href="https://stackexchange.com/sites#lifearts" class="-link js-gps-track" data-gps-track="footer.click({ location: 2, link: 24 })" data-webtasks-id="775a4c5f-8d60-480b"> |
|
Life & arts |
|
</a> |
|
</li> |
|
<li data-webtasks-id="e8714e2a-73f6-45ab"> |
|
<a href="https://stackexchange.com/sites#science" class="-link js-gps-track" data-gps-track="footer.click({ location: 2, link: 24 })" data-webtasks-id="f9207bf6-cfe2-4075"> |
|
Science |
|
</a> |
|
</li> |
|
<li data-webtasks-id="ce1ff674-8d8c-438e"> |
|
<a href="https://stackexchange.com/sites#professional" class="-link js-gps-track" data-gps-track="footer.click({ location: 2, link: 24 })" data-webtasks-id="ed5572ad-3efa-49b5"> |
|
Professional |
|
</a> |
|
</li> |
|
<li data-webtasks-id="473c6a01-e27e-4f50"> |
|
<a href="https://stackexchange.com/sites#business" class="-link js-gps-track" data-gps-track="footer.click({ location: 2, link: 24 })" data-webtasks-id="430ef2f5-e399-4884"> |
|
Business |
|
</a> |
|
</li> |
|
|
|
<li class="mt16 md:mt0" data-webtasks-id="6a883e4a-b518-46cd"> |
|
<a href="https://api.stackexchange.com/" class="-link js-gps-track" data-gps-track="footer.click({ location: 2, link: 24 })" data-webtasks-id="61056338-da82-49f4"> |
|
API |
|
</a> |
|
</li> |
|
|
|
<li data-webtasks-id="2d971d1f-9ba3-4fcb"> |
|
<a href="https://data.stackexchange.com/" class="-link js-gps-track" data-gps-track="footer.click({ location: 2, link: 24 })" data-webtasks-id="0ae45ab5-4705-4013"> |
|
Data |
|
</a> |
|
</li> |
|
</ul> |
|
</div> |
|
</div> |
|
</nav> |
|
<div class="site-footer--copyright fs-fine md:mt24" data-webtasks-id="0ce1576e-21c0-43f4"> |
|
<ul class="-list -social md:mb8" data-webtasks-id="ecd85c21-d595-4804"> |
|
<li data-webtasks-id="e551b6fd-9653-415c"><a class="js-gps-track -link" data-gps-track="footer.click({ location: 2, link:4 })" href="https://stackoverflow.blog?blb=1" data-webtasks-id="3718f4a4-02e2-4869">Blog</a></li> |
|
<li data-webtasks-id="cbd186be-d12f-44d7"><a href="https://www.facebook.com/officialstackoverflow/" class="-link js-gps-track" data-gps-track="footer.click({ location: 2, link: 31 })" data-webtasks-id="928749c7-fb7e-4011">Facebook</a></li> |
|
<li data-webtasks-id="a8a73e9c-ccb2-4c9c"><a href="https://twitter.com/stackoverflow" class="-link js-gps-track" data-gps-track="footer.click({ location: 2, link: 32 })" data-webtasks-id="8d142fdd-c9ed-44f6">Twitter</a></li> |
|
<li data-webtasks-id="1ec2c2c6-1fc2-47a2"><a href="https://linkedin.com/company/stack-overflow" class="-link js-gps-track" data-gps-track="footer.click({ location: 2, link: 33 })" data-webtasks-id="4e0081ee-6d33-4773">LinkedIn</a></li> |
|
<li data-webtasks-id="07b891d1-85dc-469d"><a href="https://www.instagram.com/thestackoverflow" class="-link js-gps-track" data-gps-track="footer.click({ location: 2, link: 36 })" data-webtasks-id="f298bc9e-3542-40c9">Instagram</a></li> |
|
</ul> |
|
|
|
<p class="md:mb0" data-webtasks-id="601f4ff4-11d0-4975"> |
|
Site design / logo © 2023 Stack Exchange Inc; user contributions licensed under <span class="td-underline" data-webtasks-id="ae5f6382-ad8c-4328"><a href="https://stackoverflow.com/help/licensing" data-webtasks-id="4eacab29-b50c-406d">CC BY-SA</a></span>. <span id="svnrev" data-webtasks-id="cebae780-8510-4117">rev 2023.7.7.43526</span> |
|
</p> |
|
</div> |
|
</div> |
|
|
|
</footer> |
|
|
|
|
|
|
|
|
|
|
|
<script async="" src="https://www.googletagmanager.com/gtag/js?id=G-S812YQPLT2" data-webtasks-id="499dcdaa-de16-4812"></script> |
|
<script data-webtasks-id="95d462e5-7a93-417b"> |
|
window.dataLayer = window.dataLayer || []; |
|
function gtag() { dataLayer.push(arguments); } |
|
</script> |
|
<script data-webtasks-id="da61b649-9983-4653"> |
|
StackExchange.ready(function() { |
|
|
|
var ga3Settings = { |
|
autoLink: ["stackoverflow.blog","info.stackoverflowsolutions.com","stackoverflowsolutions.com"], |
|
sendTitles: true, |
|
tracker: window.ga, |
|
trackingCodes: [ |
|
'UA-108242619-5' |
|
], |
|
checkDimension: 'dimension42' |
|
}; |
|
|
|
var customGA4Dimensions = {}; |
|
|
|
|
|
var ga4Settings = { |
|
tracker: gtag, |
|
trackingCodes: [ |
|
'G-S812YQPLT2' |
|
], |
|
consentsToPerformanceCookies: "denied", |
|
consentsToTargetingCookies: "denied", |
|
eventParameters: customGA4Dimensions, |
|
checkForAdBlock: true, |
|
sendTitles: true, |
|
trackClicks: false, |
|
}; |
|
|
|
StackExchange.ga.init({ GA3: ga3Settings, GA4: ga4Settings }); |
|
|
|
StackExchange.ga.setDimension('dimension1', '28364646'); |
|
|
|
StackExchange.ga.setDimension('dimension2', '|where-on-earth|bavaria|'); |
|
|
|
|
|
StackExchange.ga.setDimension('dimension3', 'Questions/Show'); |
|
|
|
|
|
StackExchange.ga.setDimension('dimension7', "1688971389.1266069309"); |
|
|
|
StackExchange.ga.trackPageView(); |
|
}); |
|
</script><iframe src="https://8504380c80e9008cd1298766e93e74a9.safeframe.googlesyndication.com/safeframe/1-0-40/html/container.html" style="visibility: hidden; display: none;" data-webtasks-id="5fb2ad39-c005-4e45"></iframe> |
|
|
|
|
|
<div id="onetrust-consent-sdk" class="d-none" data-webtasks-id="c8273683-a809-4dd3"></div> |
|
<div id="onetrust-banner-sdk" data-controller="s-modal" data-webtasks-id="8dbe8f34-7d67-4030"></div> |
|
<div id="ot-pc-content" class="d-none" data-webtasks-id="9009e0f8-97a0-4e16"></div> |
|
<div id="onetrust-style" class="d-none" data-webtasks-id="3f183654-ae0a-4b51"> </div> |
|
<div class="d-none js-consent-banner-version" data-consent-banner-version="1" data-webtasks-id="6809864c-37c6-4d4a"></div> |
|
|
|
|
|
|
|
|
|
|
|
</body><iframe name="goog_topics_frame" src="https://securepubads.g.doubleclick.net/static/topics/topics_frame.html" data-webtasks-id="e7e391a2-23b5-43f6" style="display: none;"></iframe> |